Warum ist das Universum dreidimensional?

BLOG: Das Zauberwort

Physik und die Liebe zur Welt
Das Zauberwort

Begann alles mit einem kosmischen Kabelsalat?

“Das Universum ist dreidimensional weil es nach dem Urknall total verknotet war”, berichtete das US-Magazin Newsweek im vergangenen Monat.

Nun, der faktische Ton ist hier zweifellos ein wenig übertrieben, aber der wahre Kern der Nachricht ist, dass das European Physical Journal kürzlich einen Artikel veröffentlicht hat, in dem wir tatsächlich die These vom kosmischen Kabelsalat diskutieren und als eine mögliche Erklärung für die offensichtliche Dreidimensionalität des Universums vorschlagen.

Kabelsalat im Universum! (Keith Wood/Vanderbilt)

“Wir” – das sind in diesem Fall die Kosmologen Arjun Berera und João Rosa von den Universitäten in Edinburgh (Schottland) und Aveiro (Portugal), die eher mathematisch ausgerichteten Theoretiker Tom Kephart von der Vanderbilt University (Nashville USA) und Roman Buniy (Chapman University, Kalifornien/USA) und ich. Berera, Buniy und ich waren alle zu verschiedenen Zeiten Postdocs bei Tom Kephart an der Vanderbilt University, und Rosa war wiederum Bereras Postdoc in Edinburgh. Unser Projekt hatte also ein bisschen was von einem Klassentreffen.

Was genau wollen wir dabei erklären? Nun, jeden Ort im Raum können wir durch drei Zahlen oder Koordinaten beschreiben, nämlich Höhe, Länge und Breite. Das zugehörige Koordinatensystem hat folglich drei Achsen, deshalb sprechen wir davon, dass das Universum drei Raumdimensionen aufweist. Zusammen mit der Zeit bilden sie die 3+1-dimensionale Raumzeit der Relativitätstheorie.

Aber warum gerade 3? Kurioserweise ist die Dreidimensionalität unseres Universums ein Problem, das eher selten diskutiert wird, und das obwohl populäre Entwürfe für Quantengravitationstheorien oft annehmen, die Welt habe eigentlich mehr oder weniger Raumdimensionen. So postuliert die Stringtheorie [1], gelegentlich schon als heißer Kandidat für eine “Theorie für Alles” gehandelt, 6-7 zusätzliche Raumdimensionen. Diese überflüssigen Dimensionen sind dann deshalb nicht beobachtbar, weil sie entweder mikroskopisch klein aufgerollt sind, oder weil wir und unsere Umgebung auf einer niederdimensionalen Fläche, einer sogenannten “Brane”, kleben. In der Loop-Quantengravitation wiederum wird die Raumzeit durch Schleifen eindimensionaler Objekte erzeugt, sogenannte Spin-Netzwerke. Und in der “kausalen dynamischen Triangulation” erscheint die Raumzeit auf sehr kleinen Skalen plötzlich zweidimensional oder fraktal.

Die Dreidimensionalität ergibt sich also keineswegs zwingend aus unseren Theorien, aber während man z.B. Hunderte wenn nicht gar Tausende wissenschaftliche Veröffentlichungen finden kann, die diskutieren, warum die Elementarteilchen gerade die Massen besitzen, die sie nun einmal haben, wird die doch deutlich offensichtlichere Frage des Ursprungs der Dimensionalität des Universum kaum diskutiert. Tatsächlich sind bislang nur eine Hand voll Szenarien auf dem Markt:

So hatten die Stringtheoretiker Robert Brandenberger und Cumrun Vafa argumentiert, um Raumvolumina herumgeschlungene Strings könnten die Expansion des Universums aufhalten. Allerdings könnten herumgeschlungene Strings in ungewundene zerstrahlen, falls sie sich treffen, was aber zu unwahrscheinlich in mehr als drei Dimensionen sei. Dieser Mechanismus wurde später von Brian Greene, Daniel Kabat und Stefanos Marnerides weiter ausgearbeitet. Ein ähnliches Argument, ebenfalls im Rahmen der Stringtheorie, stammt von Ruth Durrer, Martin Kunz und Mairi Sakellariadou: Hier sind es ganze Universen (die oben erwähnten Branes), die sich bei zu hoher Dimensionalität entwirren und zerstrahlen sollen.

Ein ganz anderes Szenario hatten David Hochberg (ein weiterer ehemaliger Vanderbilt-Postdoc) und James Wheeler vorgeschlagen: Bei ihnen ist die Raumzeitdimension eine dynamische, also sich zeitlich entwickelnde Variable, deren Wert sich aus Wechselwirkungen ergibt. Bei diesem und ähnlichen Vorschlägen ist es natürlich stark modellabhängig, ob die Variable dann auch wirklich beim Wert drei endet.

Das besondere an unserem Modell ist dabei, dass es sowohl „dynamisch“ als auch „topologisch“ ist. “Dynamisch” soll dabei heißen, dass es einen konkreten Mechanismus entwickelt, der gerade genau drei Dimensionen aufbläht, nämlich eine neue Variante der kosmischen Inflation [2], der Phase beschleunigter Expansion, von der die meisten Kosmologen annehmen, dass sie im sehr frühen Universum stattgefunden hat (siehe auch “Gab es den Urknall?”).

Topologie ist ein Teilgebiet der Mathematik, das sich Eigenschaften von Dingen beschäftigt, die unter stetigen Verformungen erhalten bleiben. So sind ein Würfel und eine Kugel topologisch gleich oder äquivalent, weil man sie ineinander verformen kann, ohne Löcher hineinzupieksen. Anders sieht es bei einer Kaffeetasse mit Henkel aus, die aber wiederum topologisch äquivalent zu einem Donut oder Reifen ist. Topologische Argumente sind ja/nein-Argumente. Gebäck kann entweder ein Loch haben wie ein Donut, oder kein Loch, wie ein Berliner. Halbe Löcher gibt es nicht. Die Knotentheorie ist wiederum ein Teilgebiet der Topologie. Sie diskutiert z.B. welche Knoten ineinander verformt werden können, ohne sie zu lösen oder das Seil zu zerschneiden. Knoten lassen sich überhaupt nur in drei Dimensionen knüpfen: In zwei oder weniger Dimensionen kann man Seilstücke nicht übereinander legen. In drei oder mehr Dimensionen kann man den Knoten durch einen Weg in der zusätzlichen Dimension auflösen. Das kann man sich z.B. plausibel machen, indem man einen Kreis auf ein Blatt Papier zeichnet und dann mit dem Kugelschreiber dessen Mitte markiert: Kugelschreiber und Kreis sehen aus wie verknotete Seilstücke, und genau wie beim Knoten können sie den Kugelschreiber nicht aus dem Kreis herausbewegen, ohne den Kreis zu überqueren und ohne mit der Stiftspitze das Papier zu verlassen, also eine zusätzliche Dimension einzuführen. Sollte nun also die Energie, die die Inflation antreibt, aus einem Kabelsalat von eindimensionalen Objekten bestehen, könnte die Inflation überhaupt nur drei Dimensionen zu einer makroskopischen Größe aufblasen.

Das Kabel kann nicht aus der Schlinge befreit werden, ohne die Bildebene in eine Extradimension zu verlassen.

Unser Projekt hat dabei eine längere Geschichte: Vor gut sieben Jahren hatte ich auf einem Neutrino-Workshop am CERN einen Vortrag gehalten, in dem ich unorthodoxe Ideen zur Erklärung der Neutrino-Massenspektren diskutiert hatte, unter Anderem Modelle mit zusätzlichen Dimensionen oder Modelle, die – inspiriert durch Arbeiten von Buniy und Kephart – auf den mathematischen Eigenschaften von Knoten beruhten. Einer der Zuhörer war Hitoshi Murayama, der mich fälschlicherweise so interpretierte, dass auch die Knotenmodelle mehr als drei Raumdimensionen annahmen und kritisierte, dass Knoten nur in drei Dimensionen stabil seien. Das Missverständnis konnte ich schnell ausräumen, allerdings setzte Murayamas Kommentar bei mir einen Denkprozess in Gang, der darauf abzielte, ob diese interessante Eigenschaft von Knoten nicht zu irgendetwas anderem nütze sein könnte. Also begann ich mit Kephart und Berera zu diskutieren, die ihrerseits wieder Buniy und Rosa ins Boot holten. Auf einem Workshop am Isaac-Newton-Institute der Universität Cambridge, den Kephart zusammen mit anderen Kollegen im Dezember 2012 organisierte, trafen wir uns und begannen, die Details der Idee auszuarbeiten. Im August 2015 konnten wir schließlich eine erste Beschreibung auf dem arXiv posten, wobei es dann noch einmal über zwei Jahre dauerte, bis das Paper schließlich zur Veröffentlichung akzeptiert wurde. Vielleicht tröstet es die Vertreter alternativer Theorien ja etwas, dass auch Angehörige des wissenschaftlichen Establishments manchmal etwas länger brauchen, um ihre Kollegen von der Qualität ihrer unorthodoxen Ideen zu überzeugen (oder, so die weniger wohlwollende Interpretation der Geschichte, man jeden Mist veröffentlichen kann, wenn man nur hartnäckig bei der Sache bleibt).

Aber zurück zum Thema. Woher sollten in unserem Modell die Knoten stammen? Ursprünglich hatte ich dabei Superstrings im Sinn, die schwingenen Fäden, aus denen in der Stringtheorie die Elementarteilchen bestehen sollen, aber die schienen nicht stabil genug. Kephart schlug schließlich sogenannte “Flux-Tubes” vor, Flussröhren, die in bestimmten Teilchentheorien wie der Quantenchromodynamik, oder kurz QCD,  auftreten können. Die QCD beschreibt die Kräfte, die zwischen Quarks wirken und diese z.B. in Protonen und Neutronen, den Bestandteilen des Atomkerns, binden. Dabei tragen die Kraftfelder im Gegensatz z.B. zu elektromagnetischen Feldern selbst Ladungen, sie wirken also auf sich selbst. Bei der QCD führt das dazu, dass die Kraft mit zunehmenden Abstand wächst. Dass sich die Kraft z.B. zwischen Quarks und Anti-Quarks vergrößert, wenn man versucht, sie auseinander zu reißen. Das Kraftfeld, das sich dabei zwischen Quarks und Anti-Quarks bildet, ist eine Flux-Tube. Bei kleineren Abständen oder hohen Energien sind die Kräfte dagegen klein. Dieses Verhalten ist gerade umgekehrt zu der uns vertrauten Eigenschaft von Ladungen oder Massen, weniger starke elektrische oder Gravitationskräfte zu bewirken, je entfernter sie sind.

Ein Netzwerk aus verknoteten Flux-Tubes könnte die kosmische Inflaton angetrieben haben (Roman Buniy/Chapman University).

Folglich waren die Quarks und Anti-Quarks bei den hohen Energien im sehr frühen Universum nahezu frei von Kräften, erst als die Energie des Universums so weit sank, dass die QCD-Kräfte groß wurden, bildeten sich Flux-Tubes aus. Dabei ist die Chance, dass diese Flux-Tubes schön parallel aufgereiht waren, vernachlässigbar klein. Wie jedes Kabel oder jeder Faden haben auch Flux-Tubes die Neigung, sich zu verknoten. In unserem Modell verursacht also eine neue, der QCD ähnliche Kraft (in der QCD selbst zerfallen die Flux-Tubes zu schnell) einen riesigen Kabelsalat, der das ganze Universum ausfüllte, und dessen Energie die Inflation antrieb. Da sich der Kabelsalat überhaupt nur in drei Dimensionen bilden kann, können nur dreidimensionale Universen (oder Baby-Universen) auf diese Weise “inflatieren”. Sollte der von uns beschriebene Mechanismus also das einzig mögliche (oder das letzte, bei den niedrigsten Energien funktionierende) Inflations-Szenario sein, würde das erklären, warum die Welt dreidimensional ist.

Als ich unsere Idee zum ersten mal auf dem Internationalen Symposium über Teilchen, Strings und Kosmologie (PASCOS) am ICTP im italienischen Triest vorstellte, gab es genau eine Frage. Und die kam von Andrei Linde, einem der Begründer der kosmischen Inflation. Leider war Linde ausgesprochen kritisch, genau genommen hielt er das Projekt für pure Zeitverschwendung. Sein Argument geht dabei einerseits von der Gültigkeit der Stringtheorie aus, die nach gegenwärtigem Wissensstand 10500 verschiedene Universen mit verschiedenen physikalischen Gesetzen erlaubt (die sogenannte “Cosmic Landscape” [3]), von denen nach seinem eigenen Modell der Ewigen Inflation (oder “chaotischen Inflation”), das er ebenfalls voraussetzte, auch alle in irgendwelchen Baby-Universen entstanden sein sollen. Mit diesen Voraussetzungen kann man sich nun auf das sogenannte “Anthropische Prinzip” berufen, das aussagt, das wir Menschen uns nicht wundern sollten, wenn wir ein Universum beobachten, in dem unsere Existenz möglich ist. Denn wenn es anders wäre, hätten wir keine Gelegenheit, es zu beobachten.

Anthropische Argumente für die Dreidimensionalität der Welt (Max Tegmark, arXiv:gr-qc/9702052).

Die anthropischen Argumente für drei Dimensionen hat der am MIT tätige schwedische Kosmologe Max Tegmark in einem schönen Artikel zusammengefasst: So diskutiert er zuerst eine Untersuchung, die der österreichische Physiker Paul Ehrenfest – ein enger Freund Einsteins und Mitstreiter bei der Entwicklung der Allgemeinen Relativitätstheorie mit einem tragischen Schicksal (1933 tötete Ehrenfest sich und seinen am Down-Syndrom leidenden Sohn) –  bereits 1917 angestellt hatte. So hatte Ehrenfest gezeigt, dass in einem Raum mit mehr als drei Dimensionen weder die Umlaufbahnen der Planeten noch Atome stabil sein könnten: Die Planeten und Hüllenelektronen würden entweder in das All oder die Umgebung katapultiert oder in die Sonne oder den Atomkern stürzen. Für Räume mit weniger als drei Dimensionen – also Flächen oder Linien – hatte Gerald James Whitrow 1955 argumentiert, dass sie keine Entwicklung komplexer Lebewesen erlauben würden, z.B. weil sich Nervenbahnen nicht kreuzen könnten.

Nun kann man Linde auf verschiedenen Ebenen kritisieren. Zum einen ist keinesfalls gesichert, dass Stringtheorie und Ewige Inflation wirklich die korrekte Beschreibung der Natur liefern, beides sind bis jetzt vollkommen unbestätigte Hypothesen und sollten deshalb, wenngleich zweifellos interessant, keinesfalls als alternativlos betrachtet werden. Ein weiteres Problem betrifft das Anthropische Prinzip, dass von den meisten Physikern mit Argwohn betrachtet wird. So versuchte ich Linde zu überzeugen, dass das Anthropische Prinzip zwar Delfinen plausibel machen würde, warum sie sich im Wasser befinden und deshalb nicht auf dem Mond leben, aber keineswegs erklärt, warum es Wasser auf der Erde gibt, auf dem Mond aber nicht. Anders ausgedrückt: Das Anthropische Prinzip kann zwar ein hilfreiches Plausibilitätsargument liefern, kann aber auch, wenn es zu früh eingesetzt wird, den Fortschritt der Wissenschaft bremsen, indem es die Frage nach einer wirklichen Erklärung des beobachteten Sachverhalts gar nicht erst aufkommen lässt. Obwohl ich also eigentlich ein Freund von Lindes Ewiger Inflation und anderen Multiversums-Szenarien bin, bezweifle ich, dass das Multiversum eine Entschuldigung sein darf, sich vorschnell solcher Argumente zu bedienen. Ich konnte Linde nicht überzeugen. Ich könne ja durchaus meine Zeit verschwenden, ich sei ja verbeamtet, aber er würde keinem Nachwuchswissenschaftler empfehlen, sich mit einem solchen Projekt zu befassen, gab er mir zum Abschied mit.

Allerdings ist Lindes Argument, ganz unabhängig von der Gültigkeit von Stringtheorie, Ewiger Inflation und Anthropischem Prinzip noch aus ganz anderen Gründen fraglich. Man muss sich tatsächlich fragen, ob das Anthropische Prinzip hier überhaupt greift.

Denn zum Einen sind Sonnensysteme wie das unsere zwar in mehr als drei Dimensionen instabil, aber wir könnten ja auch vierdimensionale Wesen auf einer vierdimensionalen Erde sein, die eine vierdimensionale Sonne in einem 5-dimensionalen Universum umkreist. Oder 5, 6, 7, usw.-dimensionale Wesen in 6,7,8-dimensionalen Welten…. Zum Beispiel haben Cliff Burgess, Patrick Martineau, Fernando Quevedo und Raul Rabadan die Möglichkeit “Branonium” genannter Bindungszustände zweier Branes, also beliebig-dimensionaler Volumina in der Stringtheorie, betrachtet, und dabei geschlossene Umlaufbahnen gefunden.

Aber auch an niederdimensionalen Alternativen herrscht kein Mangel: So haben der niederländische Physiknobelpreisträger Gerard t’Hooft und Lenny Susskind, einer der Begründer der Stringtheorie, spekuliert, dass das Universum ein Hologramm sein könnte [4], also so etwas wie ein zweidimensionales Bild, das eine dreidimensionale Illusion erzeugt (mehr zum “Holografischen Prinzip” gibt es bei Spektrum).

Schuld an diesen Spekulationen ist wieder einmal das Zwillingsparadox der Relativitätstheorie, genauer gesagt dessen Verallgemeinerung auf Schwarze Löcher. Wir erinnern uns: Zwei gleichmäßig zueinander bewegte Zwillinge gehen jeweils davon aus, beim anderen Bruder vergehe die Zeit langsamer und er sei folglich jünger. Das geht so lange, bis einer der Zwillinge umkehrt, also beschleunigt oder angebremst wird und zum anderen Zwilling zurückkehrt. Beim Wiedersehen sind sich beide Zwillinge einig, dass der Bruder, der beschleunigt wurde, der jüngere ist. Für einen Beobachter, der außerhalb eines Schwarzen Lochs in dessen Gravitationsfeld ruht, erfordert es nun eine ungeheure Beschleunigung, um nicht in das Schwarze Loch zu fallen. Für ihn vergeht die Zeit langsamer, auf dem Ereignishorizont bleibt sie gar stehen. Aus seiner Perspektive fällt also niemals irgendetwas ins Schwarze Loch, alles friert auf seinem Horizont ein. Andererseits wähnt sich ein frei ins Schwarze Loch fallender Beobachter in Schwerelosigkeit, er kann – ohne irgendwelche Sonderbarkeiten festzustellen – durch den Horizont ins Schwarze Loch stürzen. Darüber, wie diese zwei Beschreibungsebenen zusammen passen, rätseln Physiker bis heute, das Problem ist unter dem Namen “Informationsparadox” bekannt. T’Hooft und Susskind gingen ganz grob gesagt davon aus, dass beide Beschreibungsebenen das gleiche beschreiben, das also alles, was im dreidimensionalen Innern des Schwarzen Lochs passiert genauso gut durch eine zweidimensionale Physik auf dessen Oberfläche beschreiben werden kann. Prinzipiell lässt sich diese Idee nun auf das ganze Universum verallgemeinern, das dann kein dreidimensionales Gebilde mehr wäre, sondern nur eine dreidimensionale Illusion erzeugt durch Information auf der zweidimensionalen Fläche, die den prinzipiell beobachtbaren Bereich begrenzt.

Andere Forscher wie der Oxford-Philosoph Nick Bostrom gehen wiederum davon aus, dass wir sowieso in einer Computersimulation leben, so wie die verkabelten, in Tanks dahinvegetierenden Protagonisten des Science-Fiction Thrillers Matrix, denen über eine Schnittstelle in ihre Gehirne eine virtuelle Realität vorgegaukelt wird. Eine solche Computersimulation muss natürlich keineswegs auf einem dreidimensionalen Computer laufen. Tatsächlich kann eine Turingmaschine, also ein Schreibkopf, der ein unendlich langes, eindimensionales Speicherband entlangfährt, ausliest und entsprechend seinen Speicherzustand ändert, nach der sogenannten Church-Turing-These der theoretischen Informatik, die gleichen Rechnungen durchführen wie ein beliebig komplizierter Computer. Soll heißen, falls unser Universum nur eine Computersimulation ist, kann die Wirklichkeit auch aus einer eindimensionalen Turingmaschine bestehen.

Wenn man diesen Argumenten folgt, wissen wir also eigentlich so genau gar nicht, ob das Universum wirklich dreidimensional ist. Aber wie fast alles im Leben kann man das positiv sehen: Die nervige Tatsache, dass Ihre Kopfhörerkabel sich so gern verheddern, sorgt dann dafür, dass es ein klein wenig weniger wahrscheinlich ist, dass wir in einer Computersimulation oder einem Hologramm leben. Und vielleicht, ja ganz vielleicht ist Ihr Kabelsalat sogar direkt verknüpft mit der Entstehung des Universums.

Die Welt ist wunderbar!

[1] Brian Greene: Das elegante Universum, Goldmann Verlag, München 2000, 2006

[2] Alan Guth: Die Geburt des Kosmos aus dem Nichts, Droemer Knaur, München 1999.

[3] Leonard Susskind: The Cosmic Landscape, Time Warner, New York 2006.

[4] Leonard Susskind: Der Krieg um das Schwarze Loch: Wie ich mit Stephen Hawking um die Rettung der Quantenmechanik rang, Suhrkamp Berlin, 2010.

Avatar-Foto

Heinrich Päs ist Professor für Theoretische Physik an der TU Dortmund und forscht über Neutrinos, Teilchenphysik und Kosmologie. Er hat sich aber auch schon als Zeitmaschinenentwickler und Philosoph versucht, ein Buch ("Neutrinos - Die perfekte Welle") geschrieben und mehrere Science-Fiction Romane inspiriert. Er war Postdoc in Hawaii. Wenn er nicht forscht oder liest ist er gern in der Natur, beim Segeln, Surfen, Wandern, Skifahren oder Laufen. Und noch mehr als die Welt liebt er seine Frau Sara.

295 Kommentare

  1. Der Buddhismus (seit 2500 Jahren) und Bischof Augustinus (vor 1600 Jahren, in Bekenntnisse, Buch 11, Kap. 13-29) gehen davon aus, dass Zukunft und Vergangenheit in der Realität nicht existieren – und dass die Gegenwart nur eine imaginäre Grenze des Übergangs ohne Dauer ist. D.h. die Zeit-DAUER als 4. Dimension gibt es in der Realität nicht. Die Empfindung von Zeit-Dauer ist nur ein Ergebnis unserer Vorstellungskraft/Phantasie.

    Dies ist seit langem bekannt. Physiker gehen aber von der realen Möglichkeit einer 4D-Raumzeit aus (in der man möglicherweise Zeitreisen durchführen kann). D.h. die Idee einer 4D-Raumzeit ist eine esoterische Idee ohne belegbare Grundlage. Aber wenn Physiker schon an der 4. Dimension kläglich scheitern – dann sind Überlegungen über weitere Dimensionen 5,6,7,… als fragwürdig zu betrachten.

    Wenn die Stringtheorie 10hoch500 ohne nachvollziehbare Belege mögliche Universen erlaubt, dann ist nicht ausgeschlossen dass es auch Osterhasen und das Christkind wirklich gibt – welches Weihnachtsgeschenke bringt: Jedem Physiker kann man nur wünschen, dass dieses Jahr das Reclam-buch ISBN 978 315 002 792 9 (Augustinus: Bekenntnisse) dabei ist. Ein Quellenstudium zum Thema ZEIT wäre eine gute Idee.

    • Sehr gut,

      Wenn das Universum so kompliziert wehre wie es die Physiker und einige selbsternannte Experten im Internet machen, dann könnte es gar nicht existieren,

      Die Physiker erfinden immer eine neue Kraft damit sie die vorhergehende erklären können, das geht so bis in die Unendlichkeit. Eine klare Aussage so ist es, haben die nicht.

      Formeln passen immer, weil die nur ein Abbild sind von dem was man beobachtet hat.

  2. @KRichard:

    Sie setzen es also als “bekannt” und damit wohl soviel wie “wahr” voraus, dass die buddhistischen Ideen und Gedanken von Augustinus von vor einigen Hundert Jahren die Welt korrekt(er) beschreiben, als es heutige Theorien tun? Was hält Sie davon ab, die damaligen Weltbilder genau so wie die heutigen zu hinterfragen? Etwa, dass es sich für Sie irgenwie “richtig anfühlt”? Gleichzeitig meinen Sie, dass aktuelle Weltbilder bzgl. der Raumzeit esoterisch seien? Wie hat denn der Erkenntnisgewinn bei den Buddhisten und Augustinus funktioniert und warum genau ist er dem heutigen Weg der Wissenschaft überlegen?
    Besonders wissenschaftlich ist es jedenfalls nicht, einfach alte Aussagen (dass sie alt sind, macht sie übrigens weder automatisch richtig noch falsch) als wahr hinzunehmen und im Gegenzug das Aktuelle kritisch zu hinterfragen und zu verwerfen, wenn es nicht dazupasst. Mangels eigener wissenschaftlicher Herangehensweise ist es auch nicht besonders zweckmäßig, andere (also die heutigen) Wissenschaftler für ihre Gedanken zu kritisieren bzw. zu belächeln.

    Mal davon abgesehen ist es auch ein eher trivialer Gedanke, zu sagen, dass die Gegenwart von (bzw. fast: siehe Planck-Zeit) beliebig kurzer und damit infinitesimaler (also die Naturgesetze und damit die Planck-Zeit mal außen vor gelassen) Länge ist, weil alles, was beliebig kurz davor, Vergangenheit ist und alles, was beliebig kurz danach passiert, zur Zukunft gehört.

    Vielleicht war ihr Beitrag ja aber auch nur Satire, die ich nicht verstanden habe. Andernfalls könnten Sie bei Gelegenheit mal über die Widersprüche nachdenken, die ich in Ihrem Beitrag zu sehen glaube und im ersten Absatz hier erwähnt habe.

  3. Naja, mich irritiert einfach schon die “Kinderfrage” am Anfang dieses Aufsatzes: “Warum ist das Universum 3 Dimensional” – da könnte man meinen, der Autor hätte die Fähigkeit / das Bewusstsein – keine weiteren Dimensionen zuzulassen – (nur die Frage eben) – Wissen können wir Lebewesen wie der Autor es beschreibt nur dass, was wir eben “sehen, hören, fühlen, riechen, wahrnehmen” usw. mit unseren vorhandenen Sinnesorganen die da sind. Das “schwarze” Universum hat eben die Planeten drin – und was alles rumschwirrt – das sieht der Mensch mit seinem Bewusstsein. Klar, das ist 3D, jedoch könnten es auch 11 oder Millionen Dimensionen sein.
    Nur weil wir es nicht erkennen (sehen usw.) heißt das noch lange nicht, dass das Universum (in dem wir uns befinden) 3dimensional ist.

    Kleines philosophisches Gedankenmuster:
    Die Tastatur hat das Alphabet drauf (und noch paar weitere Tasten) und unsere vom Menschen entwickelten Zahlen mit denen wir hier kommunizieren. Die Buchstaben sind begrenzt. Das Alphabet hat halt 26 (25) Buchstaben – mit denen wir kommunizieren. Wir sind schon in diesem System “gefangen” dass wir uns “nur” mit diesen Buchstaben unterhalten können. Was wäre, wenn wir 10000 Buchstaben hätten ?(klar, auch neue Wörter wären für eine erweiterte Ausdrucksweise möglich) – Vielleicht könnten wir uns dann noch besser über das Universum dank neuer Buchstaben und Zahlen unterhalten. Und vielleicht sogar besser (und “noch exakter”) über neue Dimensionen sprechen…

    Je mehr das gesteigert wird, desto mehr Informationen. Ich behaupte, dass andere Lebensformen, welche im Universum leben (könnten) und so “intelligent” wie wir Menschen sind oder noch intelligenter 😉 dass die keinen einzigen Buchstaben wie wir hier tippen und lesen entziffern können. Die haben mit Sicherheit nicht den Duden vor sich, um übersetzen zu können.

    So, das war mein Kommentar – Rechtschreibfehler könnt ihr gerne bei meinem damaligen Deutschlehrer abgeben. Der freut sich…

    • Nur weil wir etwas in begrenzter Zeichenzahl codieren, heißt das nicht, dass wir begrenzt Informationen austauschen können (da gibt es andere Limits). Falls Ihnen das nicht klar ist, informieren Sie sich einfach mal darüber, wie ihre Zeichen auf dem Computer verarbeitet werden …

      • Ein Computer hat ein zweistelliges Alphabeth (0 und 1) und trotzdem kann er jede Information mit der wir ihn aus unserem 26 stelligen (+ Zahlen, Zeichen usw.) füttern wunderbar verabeiten. Der Vergleich hinkt also etwas.

        Nichts desto trotz steckt Wahrheit in Ihren Aussagen, nur weil wir nur 3 Dimensionen wahrnehmen können heißt es nicht, dass es nur 3 Dimensionen gibt. Ich als Nichtphysiker habe für mich selber die gewagte Hypothese aufgestellt, dass die sogenannte dunkle Materie genau die gleichen Eigenschaften hat wie die “normale Materie” sie aber einfach in einer anderen dimensionalen Hyperebene lebt die wir einfach -nicht wahrnehmen können. Ist nur eine Idee. Evtl völliger Unsinn… aber ich mag sie 🙂

      • Ich denke, sie nehmen an, dass Buchstaben erst in Zeiten des Computers entwickelt wurden und codiert werden müssen. 😉
        Buchstaben sind für Ausdrucksweise und dem Verstehen da, unabhängig ob sie digital verarbeitet werden oder in der Zeitung stehen. Dem Leser und Verfasser, welcher das digital übergibt ist dies egal. Keiner macht sich Gedanken, wie welcher Buchstabe und welche Hintergrundaufgaben gemacht werden müssen, damit eine Nachricht da steht. Wichtig ist immer, dass der eigentliche Empfänger das Lesbare mit den Augen annehmen und damit was anfangen kann.

  4. KRichard,
    Zukunft und Vergangenheit existieren in der Realität nicht.
    Also , wenn ich in den Spiegel schaue, sehe ich mein Alter und manche Leute sehen einem Menschen an, dass er nicht mehr lange zu leben hat.
    Also, wie ist diese Behauptung gemeint?

  5. @MarcGyver @Robert: Dass es die Zeit-Dauer als 4. Dimension in der Realität nicht gibt und dass die Empfindung von Zeit-Dauer nur ein Ergebnis unserer subjektiven Wahrnehmung ist – ist keine Idee, die man vernachlässigen kann. Denn bei Richtigkeit sind Ideen wie 4D-Raumzeit, Zeitreisen, Wurmloch, Multiversum, Zeitumkehr-berechnungen – fragwürdig. Es wäre also sinnvoll, darüber nachzudenken, was passiert, wenn Zeit vergeht.

    Beispiel Uhr: Eine Uhr ist im Prinzip eine Maschine. Fügt man potentielle Energie zu (indem man eine Batterie einlegt oder ein Federwerk aufzieht), dann beginnt sie zu laufen. Potentielle Energie wird in Form von kinetischer Energie das Uhrwerk antreiben, so dass man an den Zeigern die Uhrzeit ablesen kann. Ist die zugefügte Energie verbraucht, bleibt die Uhr stehen. D.h. Uhr-/Zeit vergeht nur deshalb, weil ein einseitig gerichteter Fluss von Energie stattfand. Diese Energieveränderung in der Uhr erzeugt Zeit/Uhrzeit. Oder anders gesagt: Das Wesen von Zeit ist Energie.

    Jeder Physiker sollte wissen wie eine Maschine funktioniert. Oder anders gesagt: eine ´Knotentheorie´, mit der man dynamische (zeitliche!) Veränderungen im Universum beschreiben will, muss auch in der Lage sein, die Arbeitsweise einer Uhr zu beschreiben: das Vergehen von Zeit. Das Thema ´Zeit´ ist also alles andere als trivial.

    off topic @Robert: Wie fehlerhaft unsere Wahrnehmung ist, kann man in jedem Kinofilm sehen: Wir sehen bewegte Objekte, obgleich der Film nur aus Fotos besteht.

    • Ich glaube nicht, daß das Wesen der Zeit Energie ist. Energie ist das Wesen der Zeit-Messung. Wenn eine Uhr stoppt, wird nur die Zeit-Messung gestoppt, andere Uhren laufen weiter.
      Im Prinzip ist jedes Ding -sogar kappute Uhren-, das Energie verbraucht, in der Lage, Zeit zu messen. Warum? Weil die Zeit für aus mehreren Teilchen bestehende Systeme immer vergehen wird, Dinge altern.
      Damit bleibt für das Wesen der Zeit nur 2 Optionen: eine “echte” Dimension zu sein oder einfach die Enthropie verwirklichende Relation zwischen den Bestanteilen von Systemen, die aus mehreren (Hunderte oder Milliarden) Teilchen bestehen. Für mich ist die 2. Option eher wahr.

  6. KRichard,
    Vom Wesen und Sein der Begriffe,
    ich denke, dass die Sprache eine Abbildung der Wirklichkeit ist. Je treffender und diffizieler, desto genauer können wir unsere Umwelt wahrnehmen. Die Inuit haben etwa 50 Bezeichnungen für “weiß” und jede unterscheidet sich geringfügig von der anderen.
    Wenn man keinen Begriff für eine Sache hat, dann existiert sie für uns nicht, weil wir sie nicht benennen können. Gefühlsmäßig vielleicht schon, wie die Mystiker versuchen durch kultische Handlung Erkenntnisgewinn zu bekommen.
    Umgekehrt darf man aber nicht sicher sein, wenn man einen begriff hat, das da eine Entsprechung in der Wirklichkeit hat. Ich erinnere an den Wärmebegriff des 19. Jahrhunderts, der sich als falsch herausgestellt hat.

    Das die Zeit mit der Energie gekoppelt ist, ist mir klar, weil Zeit immer Bewegung voraussetzt. Das was zwischen vorher und nachher ist, ist die Zeit. Diese Vorstellung scheint mir sehr sinnvoll.
    Wenn man wie bei Achilles mit der Schildkröte die Zeit einfach stillstehen lässt, dann kommt man zu der logischen Erkenntnis, dass Achilles die Schildkröte nicht erreichen kann. Die Realität macht uns da aber einen Strich durch die Rechnung. Slange wir denken können, so lange verfliegt die Zeit. Man könnte jetzt formuliern, Denken ist mit der Zeit gekoppelt.

    Wenn man meditiert, hat man das Gefühl die Zeit stehe still. Dieses subjektive Empfinden zur Grundlage einer Kosmologie zu machen, ist erlaubt und bringt auch ungeahnte Einsichten!
    Aber realistisch ist das nicht.

    • Zitat “weil Zeit immer Bewegung voraussetzt.”
      Ist es nicht eher umgekehrt? Dass Bewegung Zeit voraussetzt? Zeit vergeht auch , wenn ich mich nicht bewege, bewegen kann ich mich nur durch die Zeit.

  7. Patrick,
    Zeichen und Information,
    Computer können mit nur 2 Zeichen den selben Inhalt abspeichern wie bei 26 Buchstaben oder 6000 Schriftzeichen der Chinesen. Daran liegt es nicht.
    Es liegt an der elektromagnetischen Brille, die wir aufhaben und die unsere Erkenntnis filtert. Wir sehen nur was elektromagnetisch ist, under Gehirn arbeitet elektromagnetisch, ja unsere Zellen arbeiten elktromagnetisch, weil die chemischen Substanzen elektromagnetisch sind.
    Wären wir aus Neutrinos, dann wären wir viel größer , vielleicht 1 Lichtjahr groß und würden mit Lichtgeschwindigkeit durch das All rasen. Wir könnten die Sonnen nicht sehen, weil die Neutrinos nur schwach mit der Materie wechselwirken. Aber wir könnten denken. Und wir hätten ganz andere Einsichten über die Dimensionen.

  8. Heinrich Päs schrieb (29. November 2017):
    > Knoten lassen sich überhaupt nur in drei Dimensionen knüpfen: In zwei oder weniger Dimensionen kann man Seilstücke nicht übereinander legen. In drei

    … vier? …

    > oder mehr Dimensionen kann man den Knoten durch einen Weg in der zusätzlichen Dimension auflösen.

    Ganz rigoros scheint das allerdings nicht zu gelten, denn offenbar lassen sich (auch) 1+1-dimensionale Kopfhörerkabel im 3+1-dimensionalen Inneren einer Hosentasche verknoten.

    > die These vom kosmischen Kabelsalat […] Woher sollten in unserem Modell die Knoten stammen? […] Kephart schlug schließlich sogenannte “Flux-Tubes” vor, Flussröhren, die in bestimmten Teilchentheorien wie der Quantenchromodynamik, oder kurz QCD, auftreten können.

    Wenn man der Vorstellung folgt, dass (in beliebig, aber hinreichend, vielen Dimensionen)

    sich zwischen zwei […] Quarks […] Gluonen zu einem Strang verbinden

    — könnten mehrere derartige einzelne “Stränge” auch zwischen je einem bestimmten Quark und mehreren anderen Quarks bestehen,
    so dass neben oder anstatt “einfacher Flux-Tube-[[Linien]] oder –[[Kreislinien]]
    auch “mehr oder weniger kompliziert verzweigte Flux-Tube-Graphen” in Betracht zu ziehen wären?

    > Da sich der Kabelsalat überhaupt nur in drei Dimensionen bilden kann

    Gibt es wirklich auch keine hinreichend kompliziert verzweigten Graphen, die in Dimensionen größer als 3(+1) “unentwirrbar verhedderten Salat” bilden könnten?

  9. Hi,
    ich fand das Beispiel mit dem Spiegel nicht schlecht !

    Ist es nicht so das wenn man(n) in den Spiegel schaut, in dem Moment das “Licht” die “Zeit” benötigt bis wir den Spiegel sehen und zu uns zurück “reflektiert” wird bzw wahrnehmen ?
    Heißt, da das “Licht” “Zeit” benötigt, dass man alles nur in der Vergangenheit sieht ?
    Alles was man betrachtet, betrachtet man in der Vergangenheit, auch wenns nur ein Bruchteil unserer Zeitrechnung ist ?!

    Daher finde ich Déjà-vu Erlebniss ganz interessant:
    Nehmen wir mal an, so ein Ereigniss geschieht ( und wäre/ist real)
    Dann müsste man sich doch die Frage stellen ob man Gegenwart ( was ja eigentlich ein Blick in die Zukunft wäre) und oder Vergangenheit und oder Zukunft gesehen hat !
    Oder je nachdem beides ?

  10. Gerada,
    déjà-vu ist richtig unheimlich oder auch erfreuend, je nachdem. Ich hatte früher öfters welche und da fragt man sich ganz spontan, bin ich noch im richtigen Film?
    Auch verwirrend. Ich fuhr auf der Autobahn nachts zwischen Mannheim und Hockenheim. Da gibt es rechts eine Abfahrt nach Speyer. Ich fuhr mit etwa 80 km/h mit meiner Ente, da kamen mir zwei Autos entgegen rückwärts
    eines von der Abfahrt nach Speyer und eines aus gerader Richtung Hockenheim. Auch sehr langsam.
    Mir kam es vor, als ob die Zeit rückwärts läuft.
    Ich wechselte auf die linke Spur um keinen Zusammenstoß zu riskieren.
    Darüber musste ich noch tagelang nachdenken. Das war nicht mehr real.

  11. Ich habe ja schon einmal gesagt dass ich Laie bin (bei https://scilogs.spektrum.de/das-zauberwort/gab-es-den-urknall/ am 3.11 um 17:26). So auch im (Fach-) Verständnis dieses threads. Aus den zahlreichen (interessanten!) Punkten greife ich daher Mal zwei heraus, zu denen ich relativ leicht etwas anmerken kann.

    Das Anthropische Prinzip: Sehr “naheliegend“ dass wir die Dinge so sehen wie wir sie sehen (aus unserer Sicht, gemäß unserem “Bezugssystem“; unseren Prägungen) – da wir eben so sind wie wir jeweils sind [*jeweils* i.S.v. dass wir uns ändern (“evolutionieren“), dazulernen (Verständnis vermehren)]. Das mit dem dass wenn das A.P. zu früh eingesetzt wird es den Fortschritt bremsen kann (indem es die Frage nach einer wirklichen Erklärung des beobachteten Sachverhalts gar nicht erst aufkommen lässt), verstehe ich nicht so recht.

    Zu dass bei der QCD die Kraft mit zunehmendem Abstand wächst (bzw.) dass sich die Kraft [..] vergrößert wenn man versucht sie auseinanderzureißen, kam mir der Gedanke, dass hier also zwei Kräfte im Spiel sind. Eine die zieht und eine die dagegenhält. Welche gewinnt am Ende (zerreißt das Kraftfeld oder nicht)? Denkbar, dass sich beide “ewig“ die Waage halten. Je mehr das eine zieht, desto mehr hält das andere dagegen (bzw. umgekehrt). Oder auch Mal Expansion, Mal Kontraktion (“bzw. umgekehrt“). Das ist von mir aber auch mit etwas “Augenzwinkern“ gemeint.

    Wegen den (vielen) schwierigen Punkten/Aspekten [die vielen Links – aus denen man wiederum aber ja auch Interessantes lernen kann (bzw. sie sorgfältig lesen muss/müsste, um besser zu verstehen worum es Ihnen geht)], verlinke ich jetzt auch (und noch zusätzlich) etwas (heute Vormittag im Internet drauf gestoßen). Sean Carroll [den ich in dem o.a. post ebenfalls erwähnt habe (kleine Korrektur dazu: Das Buch hatte ich nicht kostenlos runtergeladen, sondern gekauft)]:

    https://www.youtube.com/watch?v=2JsKwyRFiYY (Vortrag) und https://www.youtube.com/watch?v=KEt5XQuE_cY (Q&A zum Vortrag).

    Da sind auch einige (auch “psychologisch-philosophisch”) interessante Aussagen. Z.B. ab 5:40 (der roadrunner). Auch eine Art von A.P. Erst wenn wir uns von Etwas bewusst werden (es messen) “kollabiert“ es. *Robert* schreibt heute um 12:12:

    Wenn man keinen Begriff für eine Sache hat, dann existiert sie für uns nicht, weil wir sie nicht benennen können […] Umgekehrt darf man aber nicht sicher sein, wenn man einen Begriff hat, das da eine Entsprechung in der Wirklichkeit hat.

    Stimmt. Es kann aber sein dass man eine Vorstellung (ein Bild) im Kopf hat – und einem bloß (gerade) das Wort/der Begriff dazu nicht einfällt. Und: Nur weil wir etwas nicht wissen (überhaupt kein Bild davon haben), heißt das ja noch lange nicht dass es das auch nicht gibt. Abstrakte (Ober-) Begriffe (zusammenfassende B.) haben keine (materiellen) Entsprechungen in der Realität (sind jedoch hilfreich/zeitsparend bei der Kommunikation). Ein z.B. Auto an sich gibt es nicht; es ist ein “Konglomerat“ (eine “Zusammenballung“) seiner Bestandteile (Konstituenten) – der Veränderung unterliegend; übrigens. Im Vortragsvideo (9:10) sagt Carroll auch that the natural state is not rest, but constant motion.

    Zwar negiert er häufiger, dass bei den fundamental laws (d.h. in der Quantenphysik) Ursache und Wirkung gibt (sondern Wahrscheinlichkeiten/Zufall). Bei 13:30 sagt er aber auch if this then that and ice versa (was doch Ursache und Wirkung ist).

    Bei 39:30 sagt er there will literally be nothing but empty space. Leerer Raum ist aber doch auch noch Etwas. Na ja: Andererseits nur ein (abstrakter – s.o.) Begriff. _Bestehen_ tut (konstituiert ist) er aus nothing. Art paradox. Ab so 52:20 sagt er (m.o.w.), dass Magnetfelder entstehen when I think a thought. Gedanken an sich sind immateriell/abstrakt (nur ein Begriff); trotzdem aber real (wirkungsmächtig).

    Bei 45:45 there is this wonderful thing (that kicks in) called evolution. Erinnerte mich an Ihr “Motto” *Die Welt ist wunderbar!* Nicht nur aber – es geschehen (z.B. unter Menschen) viele Dinge, die nicht wunderbar sind. Doch solche Fragen (Ethik, Moral – good/bad usw.) sind nicht das (eigentliche) Thema der sciences [sagt er sinngemäß (am Anfang von Video zwei bei 1:00 – als Antwort auf die erste Frage)].

    Beste Grüße.

    • Hallo Herr Krüger, sorry irgendwie landen Ihre Beiträge öfters mal in der Quarantäne, so dass ich sie nachträglich genehmigen muss. Keine Ahnung, wieso, ich finde sie nämlich richtig gut. Beste Grüße! Heinrich Päs

      • Keine Ahnung, wieso

        Ich vermute, weil in Herrn Krügers Kommentar drei Links enthalten waren (und zwei außerhalb von Scilogs). Das mögen viele Spamfilter nicht.

        Dass die Links auf Videos von Sean Carroll verweisen und damit weit entfernt von Spam angesiedelt sind, dafür sind heutige Spamfilter halt noch nicht intelligent genug. 😉

      • Das liegt an den Dimensionen, bei mehr als zwei links gibt es einen Knoten 😉
        hat man wohl so gemacht, daß nicht zuviel (Eigen-)Werbung reinkommt.

        Hallo Prof Päs,
        das wollte ich um 12:25 sagen, da war der Kommentar 9:39 von Spritkopf mit IE
        nicht zu sehen (wieso?), Vorschau ging nicht, dafür meine Meinung ins Nirvana.
        Ausweg genommen, und auf Opera gibt es die heutigen Kommentare gar nicht.

      • Das liegt an den Dimensionen, bei mehr als zwei links gibt es einen Knoten 😉
        hat man wohl so gemacht, daß nicht zuviel (Eigen-)Werbung reinkommt.

        Hallo Prof Päs,
        das wollte ich um 12:25 sagen, da war der Kommentar 9:39 von Spritkopf mit IE
        nicht zu sehen (wieso?), Vorschau ging nicht, dafür meine Meinung ins Nirvana.
        Ausweg genommen, und auf Opera gibt es die heutigen Kommentare gar nicht,
        auch der Bezugskommentar von A.K. vom 30.11. aus dem Spam fehlt dort.
        Das ist jetzt der 3. Versuch, der 2. von 12:40 ist auch nicht angekommen.

        • @ Herr Senf
          Das hab ich (auch) schon zweimal moniert. Es musss an der Aufpaltung der Kohärenz in die verschiedenen Wirklichkeiten liegen !! Dennn sehhhhr wahrscheinlich funktioniert die Dekohärenz in der Matrix direkt über uns auf genau dieselbe Weise wie auf unserer Ebene. Was natürlich zu erwarten ist, weil die Naturgesetze ja universell gültig sind .
          Aber weil Ihre Apelle an die Administration direkt da oben ja nichts genützt haben , möchte ich Sie ermunteren, sich noch weiter oben zu beschweren.
          Denn mir ist überschnell zu Ohren gekommen, dass DIE dort noch etwas weiter hinter dem Urknall sitzen.
          Ich wünsche Ihnen jetzt schon viel Glück. Kann aber dauern, da auch Sie ja erklärt haben, dass “Information” auch dem Gesetz einer gewissen Maximalgeschwindigkeit unterliegt. Macht aber nichts. Als Pensionist habe zumindest ich in MEINEM Koordinatensystem (fast) alle Zeit der wirklich ganzen Welt.

          (Dafür will ich aber jetzt die Anwartschaft auf den Nobelpreis)

          • Nachtrag:
            Da ich ein unverbeserlicher Optimist zu sein scheine, scheue ich die Etwas endgültige Hypothese des “Nichts-Nirwana” wie der Teufel die Nihilisten.
            Denn das mit dem unendlichen “Etwas” namens “Matrix” ist mir halt viel angenehmer. Hab schon als Kind im Keller immer Angst vor dem dunklen “Nichts- Nirwana” dort gehabt.
            Und mir dann gewünscht: Advent, Advent – ein Lichtlein brennt.

      • @Axel Krüger, @ Karl Mistelberger: In der Quantenfeldtheorie ist das Vakuum keineswegs „nichts“ oder „leer“ sondern nur der Zustand mit der niedrigsmöglichen Energie. Das beeindruckendste Beispiel ist meiner Meinung nach der Unruh-Effekt, bei dem ein beschleunigter Beobachter im Vakuum Teilchen misst. „Leer“ oder „nicht-leer“ ist dann eine Eigenschaft des Koordinatensystems.

    • ” Ab so 52:20 sagt er (m.o.w.), dass Magnetfelder entstehen when I think a thought. Gedanken an sich sind immateriell/abstrakt (nur ein Begriff);”

      Diese Aussage hat mich doch sehr stutzig gemacht, dass Sean Carroll behauptet haben könnte, Gedanken seien “immateriell”.
      Also habe ich mir das Video an der Stelle nochmal angeschaut, und natürlich, behauptet Sean Carroll gerade nicht, dass Gedanken immateriell seien, sondern dass das exakte Gegenteil zutrifft: 52.30: “Thoughts are associated with real physical things. happening in your brain. They are not abstract things outside our physical bodies. ”

      Wie sollte es auch anders sein.

      • Hallo Sherfolder,
        einen spannenden Blog haben Sie als Webseite, ist das Ihrer?
        Was „materiell“ versus „nicht-materiell“ angeht, muss man genau sagen, was man eigentlich meint. Ich würde z.B. auch Gedanken eher als „nicht-materiell“ bezeichnen, damit meine ich aber keineswegs „nicht-physikalisch“. Denn Gedanken können ja auf verschiedenen Medien archiviert und verbreitet werden, vom Buch über den USB-Stick bis zum Blog. Das teilen Sie mit vielen Dingen, bei denen der Informationsgehalt wichtiger ist als das materielle Trägermedium (Gesetze, Musik, Mathematik, Aktiengresellschaften, ja sogar biologische Organismen). Tatsächlich denke ich, dass die Beziehung von Materie und Information eines der wichtigsten Themen für die Naturwissenschaft des 21.Jahrhunderts sein wird.

        Beste Grüße, Heinrich Paes

        Ps: Genau zu diesem Thema habe ich übrigens von Gerhard Roth, den Sie ja auch populär zitieren, ziemlich haarsträubendes gelesen….

  12. Interesssante Fiktion, aber schon arg weit von dem entfernt, was in der Quantenwelt so festgestellt wird. Und mit vielen Prämisse-Fehlern.

  13. Habe leider gerade keine Zeit, mir den Artiekel durchzulesen. Die Überschrift finde ich aber schon anmaßend. Wer sagt denn, dass das Universum dreidimensional wäre?? Etwas, was angeblich unendlich sein soll, kann meiner Auffassung nach gar nicht dreidimensional sein. Wo sollen diese drei Dimensionen den stecken??? – Lol — Entweder es ist ein dreidimensionaler Raum, dann muss dieser aber auch endlich sein oder es ist gar kein Raum, so wie wir Menschen es uns mit useren Spatzenhirnen vorstellen.

  14. Hallo, mir als Laie ist es immer wieder unklar, weshalb Wissenschaftler das anthropische Prinzip überhaupt ernsthaft heranziehen. Schließlich macht es uns doch in keinster Weise schlauer. Wenn wir jedesmal auf die Frage, weshalb das Universum so ist, wie es ist, mit der fast schon scherzhaft anmutenden Antwort kämen: “Weil es sonst keine Naseweise wie dich gäbe, die solche Fragen stellen könnten!”, wo bliebe da der Erkenntnisgewinn? Propagiert man da nicht eine Es-ist-halt-so-Physik, die in Wirklichkeit doch gar keine sein kann?

  15. Dafür muss man Wissentschaflter sein um festzstellen das das Universersum keine Scheibe ist.

    Wow … ich denke das es genausoviel Dimensionen gibt wie es immer kleinere Objekte gibt die Entdeckt werden (Gottesteilchen).

    Somit ist jede Festlegung nur solange haltbar wie die Gegenwart andauert.

  16. Hallo ihr Gelehrten (hier schreibt euch ein Experimentalphysiker, d.h. ich kann euch leider nur so ca. auf Delphinniveau fragen, obwohl ich mir die Frage “warum 3 Raumdimensionen” selbst oft gestellt habe)

    Könnt ihr mir also sagen, ob nicht als einfachstes Argument gegen mehr als drei Raumdimensionen das Kreuzprodukt dann physikalisch nicht mehr funktionieren würde? A = Ursache -> B = Vermittlung -> C = Wirkung (ortogonal auf A und auf B) Ich bin jetzt nämlich mathematisch nicht bewandert genug, um mir ein vierdimensionales Vektorprodukt aufschreiben und veranschaulichen zu können, aber vielleicht geht das ja formal auf…

    Das Kreuzprodukt ist doch z.B. Basis von Drehimpuls und damit auch dessen Erhaltung. Oder Lorentz-Kraft. MaW. Bewegungen in Gravitations- und elektromagnetischen Feldern würden in höheren Dimensionen unvorhersagbar “auseinanderfallen”, weil die Richtung der Wirkung im Vektorprodukt nicht mehr eindeutig definiert wäre.

    Dieses simple Argument scheint mir sehr in Richtung von Ehrenfests Überlegungen zu gehen. War natürlich klar, dass da vorher jemand drauf gekommen ist.

  17. Zeit existiert nicht? Kommt wohl drauf an, was man unter ‘Zeit’ versteht. Die einseitige Bewegung des Universus (thermodynamischer Zeitpfeil), der alles unterliegt, existiert sehr wohl. Zukunft und Vergangenheit aber sind nützliche Konzepte, für die es in der physikalischen Welt allerdings meines Wissens nach keinen einzigen Beleg gibt. Was wir sehen ist immer, ausschließlich die Gegenwart. Die Gegenwart verwandelt sich nicht plötzlich in Vergangenheit und wird dann in einer Schublade verstaut. Die Gegenwart verändert sich aber unaufhörlich, diese Veränderung ist physikalische Wirklichkeit.

    • Zeit-DAUER als 4. Dimension (DAUER) existiert in der Realität nicht, sondern nur in unserer Vorstellung/Phantasie – ein wichtiger Unterschied. Deswegen können wir uns Zeiträume vorstellen und zeitabhängige Phänomene berechnen (Geschwindigkeit).

      Wenn es aber Physiker gibt, die Zeitreisen berechnen können – obwohl diese in der Realität nicht möglich sind, sondern nur in der Vorstellung – dann haben wir ein Problem: Mit dieser physikalischen Mathematik kann man Phantasie-Ideen berechnen. D.h. diese Mathematik/Physik hat eine Qualität, über die es sich einmal nachzudenken lohnt.

  18. Eigentlich darf ich nichts hier schreiben, da ich bis heute nicht begreife, wie unsere Begriffe “alles” und “nichts” beide vorstellbar sein sollen. Ich kann mir diese in ihrer Absolutheit und dabei gegenseitigen Aufhebung einfach nicht vorstellen.

    Entsprechend verstehe ich die gesamten mathematischen und physikalischen Gedankenmodelle nicht.

    Ich habe nur eine Frage: wenn ich mir eine Kugel (oder einen Würfel) als lochfreies Gebilde vorstelle, räumlich in drei Dimensionen, die NICHT zusammengesetzt, sondern materialisiert per se als Kugel räumlich materiell existiert, dann hätte ich damit zwei räumlich-materiell vollständig getrennte Räume. Könnten sich nun in diesen unterschiedliche Dimensionen ausbilden, z.B. indem unterschiedliche Zeiten in diesen Räumen herrschen bzw. sich entwickeln? Aber es muss ja nicht die Zeit sein, wenn ja noch weitere Dimensionen gedacht werden.

    Wie kann ich mir eine solche Absolutheit vorstellen – die Räume voneinander materiell trennt. Oder gibt es Dimensionen, die jegliche materiell-räumliche Trennung oder Absolutheit aufheben?

    Wie kann ich in einem solchen Fall überhaupt nur bis zu einem Urknall denken?
    Könnte dieser nicht einfach ein Moment sein, der eine Bewegungsänderung verursachte, wie beim Pulsen sich dehnen und kontrahieren einen Umkehrpunkt haben, der zeitlich aber vielleicht gar nicht auf einen einzigen Moment beschränkt werden kann, da sich konträre Bewegungen zeitweilig überlagern?

    Ist demzufolge der Gedanke an DEN einen Urknall nicht eine ziemlich hilflose Konstruktion, an der sich alle aufhängen – weil sie eben nicht weiter blicken können?
    Gab es “vor” dem Urknall keine Kraftfelder?

    Gab es vor dem Urknall keine Ideen? Oder sind Ideen und Gedanken nur materiell gebunden?

    Das sind so meine Fragen.
    Entschuldige mich für die völlige Unwissenheit in naturwissenschaftlicher Hinsicht!

  19. Hallo Herrschaften ! Hat sich schon JEMAND Gedanken über eine Paraboloide Möbiusschleife gemacht deren Segmente fortlaufend pulsieren den umlaufenden Rcci-Fluß außenherum und den Corolli Effekt innerhalb der Möbiusschleife darstellen, letzt endlich die Trennung des Tochteruniversums vom Mutteruniversum interpretieren.
    Wie findet der Kopier Prozess statt. Wie hoch ist die Lebenserwartung des Mutteruniversums.? Noch weitere Fragen. Euer QuerdenkerNorbert

  20. Diskussionen um Spekulationen von Menschen, die allermeist durch neurose-bedingte “Wahrnehmungs- / Erkenntnis-Störung” an der Wahrnehmung / Erkenntnis der vollen Wirklichkeit gehindert sind.

    Die Wirklichkeit besteht aus ZWEI Ebenen / “Hälften”:
    1. Der bewirkenden – feinstofflichen (Seins- / Bewußtseins)-Kraft / -Energie
    und
    2. der bewirkten – grobstofflichen – Materie.

    (Auch) Der Mensch gehört zu BEIDEN: Sein Körper ist aus erschaffener Materie und seine Seele aus feinstofflicher Seins- / Bewußtseins-Energie.

    Die materielle, scheinbar 3+1-dimensionale Welt, die wir hier vorfinden, wenn wir auf die Welt kommen bzw. die wir wahrnehmen, wenn unser grobstoffliches Bewußtsein entprechend entwickelt ist, haben nicht wir eerschaffen, sondern das göttliche Sein / Bewußtsein, welches meiner Meinung nach aus unzähligen kleinsten Energieeinheiten (evtl. “Quarks” oder “Strings”) besteht. Die Bibel nennt z.B. die “göttlichen / himmlischen Heerscharen”. Die ebenfalls genannte “Kraft aus der Höhe” ist “universelle feinstoffliche (Seelen-)Energie” / “höhere Bewußtseins-Energie”.

    Zwischen dem “universellen” Gott und dem Gott im Menschen gibt es eine Art “Zuständigkeits-” / “Verantwortungs-Teilung:
    2. Der Mensch erschafft sich selbst als höheres Bewußtsein und sein Leben;
    1. Der universelle Gott erschafft die “Rahmenbedingungen”.

    Eine “Computer-Simulation” kann nicht mehr tun / erschaffen als ein schöpferisches Bewußtsein – und das wir IN und MIT einem solchen Leben, kann der Neurose-freie, zum wahren Menschsein Aufgestiegene erkennen.
    “Jeder Mensch ist ein Gedanke Gottes” heißt es irgendwo in den Heiligen Schriften. Und: “Kein Mensch bin ich, sondern ein Gott; heilig in meiner MITTE” (Buch Amos).
    Die “Mitte” deute ich als das wahre Zentrum des Menschen, seine göttliche Seele.

    Auch Jesus fragt die Menschen:
    “Haben euch eure Väter nicht gesagt, daß ihr Götter seid? Sagt es nicht euer Buch / Gesetz?”

    Der normale, mehrheitliche, typische, Wissenschaftler ignoriert, leugnet, “verdrängt” diese “ewigen Wahrheiten”. Meinen 70-jährigen Erfahrungen nach aufgrund der “Kollektiven Zivilisations-Neurose” (KZN), um die und deren Heilung man sich viel mehr kümmern sollte. Denn ohne grundlegende, natürliche, Heilung drohen Untergang und Aussterben.

  21. Heureka 47,
    du sprichst die Ursachen an, die Subjekt-Objekt Spaltung.
    Viele Kommentatoren hier machen keinen Unterschied zwischen dem Subjekt und seinen begriffen und dem Objekt mit seinen Gesetzmäßigkeiten.
    Michael,
    du kannst dir viele Fragen sparen, wenn du akzeptierst, dass unser Denken nur Modelle produziert und wir nur nach deren inneren Widersprüchen suchen, anstatt in der Wirklichkeit selbst zu suchen.
    Mystiker probieren das, Künstler versuchen das, Praktiker tuen das.

    • Danke, Robert!)

      Wobei probieren, versuchen, tun sich meines Erachtens in nichts unterscheiden – außer den effektiven Ergebnissen in der materiellen Welt. Da kann ich den Naturwissenschaften nur bewundernd zusprechen, dass sie extrem viel verändert haben, sowohl stofflich, als auch geistig.

      Doch wenn es so ist, wie Sie es beschreiben, dann bleibt auch die Naturwissenschaft gefangen, ist mithin die Beschäftigung mit ihr gewiss eine erfüllende Herausforderung für alle daran Beteiligten. Aber Ihr Ergebnis würde nach dieser Überlegung über die ersten unbeantworteten Fragen nicht hinauskommen.

      Somit stellt sich die Frage nach dem Zweck, nach dem Sinn, und damit danach, was wir Menschen aus den zwischenzeitlich erhaltenen (neuen) Erkenntnissen ganz profan machen.

      Und da würde ich weder die Mystiker, noch die Künstler außen vor lassen, auch wenn diese nur spielen wollen. Wollen die Naturwissenschaftler ja auch nur)

  22. Norbert Kowalski,
    es ist schon göttlich, wenn man ein Paralleluniversum in Händen halten kann und wenn du dann noch das Möbiusband in einen 4-dimensionalen Raum einbettest, bekommst du eine Erleuchtung, die einer Lampe mit 1000 Lumen entspricht. Das ist ein Beitrag zum Energiesparen.

  23. Es tut mir leid wenn ich eure welt ein wenig auf den Kopfstelle, das Universum ist nicht 3 dimensional! Ich bezeichne es als O dimensional, leider kann ich jedoch nicht wirklich mit worten oder formeln seine form darstellen oder zum ausdruck bringen. um das Universum zu begreifen Muss man all sein wissen erst mal über bord werfen, mit unserer heutigen Logig wäre es nicht erfassbar. Also am besten weiterforschen denn darin fängt die erkenntnis an.

  24. Hallo!

    Ist “Zeit” in dem Sinne, dass etwas eine (zwei) Ursache(n) und eine Wirkung hat, und somit etwas passiert – quasi eine Zustandsänderung, wofür es eben ein “Zeitvergehen” braucht aus unserer Sicht – ist das nicht einfach nur wie als wenn man einen Stein nimmt und wirft ihn ins Wasser…es entstehen Wellen um ein Zentrum. Diese “Wellen” tragen meiner Meinung nach die Zeitkomponente schon in sich.
    Ich will sagen, ich denke, dass Zeit nichts anderes ist, als eine Eigenschaft der Materie ansich und nicht unabhängig davon existiert. Quasi wie eine Welle, die sich eben ausbreitet. Materie ist wiederum Bewußtsein. Oh jetzt wirds esoterisch.. 😀

    @Robert: Die Mystiker machen doch keine kultischen Handlungen um Erkenntnisgewinn zu bekommen. Der Mechanismus ist über die Herstellung einer Verbindung zum Innersten, das wiederum mit dem Ganzen verbunden ist.
    Aber wer weiss schon, was Bewußtsein ist…

    Grüße!

    Christoph

  25. @ Vizero und seiner These:
    “…Dass Bewegung Zeit voraussetzt? Zeit vergeht auch , wenn ich mich nicht bewege, bewegen kann ich mich nur durch die Zeit.… (Zitat- Ende)

    Ich weiß noch nicht mal, wie Sie genau (!) feststellen wollen, ob Sie sich in Bezug auf ein anderes “System” (eine Funktionseinheit) wirklich nicht bewegen, oder ob dieses Bezugssystem wirklich absolut unbewegt ist.
    Ganz sicher ist aber, dass SIE ein “System” sind, das intern niemals ohne Bewegung auskommen kann. Denn dann stehen Ihre Funktionen still und Sie funktionieren nicht mehr. Und obwohl eventuell die Subsysteme, aus denn Sie bestehen, noch “weiterleben”, sind jedenfalls SIE ziemlich tot.
    Sie bezahlen also zumindest Ihre interne Bewegungslosigkeit dann halt mit ihrem Verschwinden aus der Welt.
    Ob es das wert ist?

  26. Anthropisches Argument für die 3-Dimensiolaität der Welt? Der neben Einstein wohl herausforderndste Theoretiker der Neuzeit, Sepp Herberger, hat postuliert: „der Ball ist rund“. Okay, könnte auch ein Kreis oder eine mehrdimensionale Rundheit sein, aber der Alt-Bundestrainer, ein ausgewiesener Taktik-Fuchs, wusste natürlich genau, dass das Spielfeld zwar zweidimensional ist, die Kugel aber durchaus auch 3D oben im Torwinkel versenkt werden kann. Unerwarteten Beistand erhielt er von Francis Picabia, einem Pionier der modernen Kunst, und dessen Diktum: der Kopf ist rund, damit das Denken die Richtung wechseln kann. Übrigens versetzte Herberger ganz nebenbei auch dem thermodynamischen Zeitpfeil den Todesstoß, indem er feststellte: „nach dem Spiel ist vor dem Spiel“.

    Ich finde es jedenfalls faszinierend, wenn äußerst spezifische und komplexe Dinge so zur Darstellung kommen, in Bildern etwa, dass es auch für Nicht-Naturwissenschaftler möglich ist, darauf assoziierend oder mit Analogien zu reagieren. Dank dafür!

    Zu den hypothetischen Knoten: Das Universum als Gewebe ist eine uralte Metapher – erst 2016 veranstaltete die Textile Society of America das Symposion The Woven Cosmos: Visualizing the Invisible through Textile, Modern Science, and Ancient Worldviews, an der Kunstwissenschaftler, Ethnologen und ebenso ein Experte in computational astrophysics beteiligt waren. Das ist Gewebe. Knoten kommen beim Teppichknüpfen ins Spiel, der persische Keshan-Knoten etwa wurde aufgrund seiner Komplexität und Schönheit in das UNESCO Weltkulturerbe aufgenommen. Und Schifffahrt lehrt, dass es unglaublich viele Knoten mit den unterschiedlichsten Eigenschaften gibt, etwa solche, die eine Leine ein stückweit laufen lassen und an einem definierten Punkt dann stoppen. Ob Tubes oder Strings da mitspielen? Wer weiß? Womöglich sind das alles gordische Knoten.

    • @ anonymus
      Ist mir völlig unerfindlich, von was für einem runden Objekt Sie da philosophiert haben. Muss einer mir völlig fremden Aufspaltung zur Realität entstammen. Seltsam und: “faszinierend !” Da krieg ich vor lauter Aufregung immer gleich einen Knoten in den Hals. Zur Entspannung schau ich mir deswegen morgen nochmal was total Realistisches an, das heute nacht um 03:10 im wirklich Allerersten (nochmal) wiederholt wird:
      “The Fountain – Quell des Lebens” . Dort kann man wenigstens alles überleben.
      Denn ich überleb so manche Talkshow manchmal nur mit Mühe. Noch schlimmer wirds, wenn ich die Zeitung les, oder zur besseren Verdauung bei Tisch den Deutschlandfunk hör.
      Na dann bis morgen. Wir wollen dann darüber philosophieren, ob und wie ich es fertig gebracht habe, etwas in der Vergangenheit schon Abgeschlossenes in einer nicht allzu fernen Zukunft nochmal zu sehen und sogar viel später noch mit endlich minus X Geschwindigkeit fast instantan mit fernen Bezugssystemen genau darüber zu plappern.

  27. Catchme76,
    …Dimensionen,
    ganz sicher hat die Welt auch noch eine 4. Dimension.
    Denk doch mal an die Schwarzen Löcher. Wohin verschwinden die Himmelskörper? Wir sagen ganz einfach sie werden singular.
    Wohin verschwindet eine elektromagnetische Welle im dem Augenblick , wenn sie als elektrisches Feld verschwindet und als magnetisches Feld wieder auftaucht. Für einen sehr kurzen Moment ist sie verschwunden.

  28. Hallo Herr Päs,

    leider hat Ihre Idee mit dem Kabelsalat entscheidende Nachteile:

    a) sie basiert auf formalen, statt auf rein physikalischen Überlegungen (aus mathematischen Stukturen kann man viel herauslesen, je nach Brille),

    b) vor allem aber ist sie sehr kompliziert.

    Wenn ich Sie richtig verstehe: die Dreidimensionaliät steckt schon im Kabelsalat, welcher durch energetische Prozesse (?) die Inflation verursacht. Dann wäre die Dreidimensionalität in dem äußerst komplizierten, nicht anschaulich erklärbaren Gemengelage vor der Inflation schon angelegt, und nicht erst durch die Inflation erzeugt. Die Inflation würde dann schon im Dreidimensionalen mit Überlichtgeschwindigkeit stattfinden.

    Oder findet die Bildung der Dreidimensionalität gleichzeitig im Kabelsalat u n d der Inflation statt ?

    Oder In der Infaltion ? Dann ist der Kabelsalat nicht die Ursache (jedenfalls nicht die einzige).

    Sie sind doch Fan der Physik nicht zuletzt wegen Ihrer Klarheit und Einfachheit. Soll dieses komplizierte, formal-mathematische, kaum deutbare Modell der Schlüssel zur Wahrheit der Dimensionaliät unserer Welt sein ? Da kann ich eigentlich nur leicht drüber lächeln. Nun gut, jedem sein Steckenpferd.

    Es gibt doch viel einfachere Modelle, schon seit vielen Jahren, die sehr schlüssig und vor allem mit physikalischen Argumenten arbeiten. Sogar ich kann mir spontan ein funktionierendes vorstellen. Eins haben Sie am Rande erwähnt: nämlich die Spin-Netzwerke. Diese erklären die Dreidimensionaliät der Welt auf sehr, sehr einfache Weise – eben auch physikalisch und sehr verständlich, sogar für Laien. Daß sich über die Dreidimensionalität kaum einer Gedanken macht, ist also nicht richtig. Ganze Generationen von Physikern machen sich Gedanken über die Spin-Netzwerke, die nicht nur die Dreidimensionaliät erklärt, sondern einer der Kandidaten für eine Theorie der Quantengravitation ist.

    Um die Dreidimensionaliät zu erklären ist es im übrigen sehr wichtig, die beiden Fragen zu unterscheiden: warum ist die Welt dreidimensional ? – und wie ist die Dreidimensionaliät entstanden ? Welche Dimensionaliät entsteht ist nämlich eine Prozeßfrage, warum sie so ist eine Strukturfrage. Am besten man behandelt diese Fragen getrennt – dann versteht man alles besser.

    Eine allgemeine Antwort kann ich Ihnen vorab schon geben: der einfachste Prozeß führt zur einfachsten Struktur, die Basis für den Rest ist. Wobei wir wieder bei den Schwächen Ihres Modells wären.

    Grüße
    Fossilium

    • Sind wir halt dort, wo wir schon immer waren:
      Die Welt ist so, wie sie so ist – ein Problemfall ihrer Anfangsbedingungen im Multiversum.
      Vielleicht sind konkret nur die Hauptsätze der Thermodynamik schuld.
      Die Energie der Inflation hat gereicht für 3 Dimensionen, dann war’s zu kalt für mehr.
      Nur gut das was übrig war für das was wir heute in den 3 Dimensionen drin haben.
      Erklärt nur nicht die Frage, warum entstehen zuerst Dimensionen, und warum überhaupt.
      http://iopscience.iop.org/article/10.1209/0295-5075/113/40006 März 2016
      “Is the (3 + 1)-d nature of the universe a thermodynamic necessity?”

  29. Vielleicht hatte der Alte doch recht mit seiner Ahnung , die er einem Freund in einem seiner letzten Briefe angeblich mitgeteilt haben soll:

    ” Ich erachte es aber durchaus für möglich, dass die Physik nicht auf den Feldbegriff gegründet werden kann, das heißt auf kontinuierliche Gebilde. Dann bleibt von meinem ganzen Luftschloss inklusive Gravitationstheorie, nichts bestehen.”

  30. Guten Abend Herr Päs,
    Überschrift Ihres Artikels:
    Warum ist das Universum dreidimensional?
    Zitat:
    Wenn man diesen Argumenten folgt, wissen wir also eigentlich so genau gar nicht, ob das Universum wirklich dreidimensional ist. Aber wie fast alles im Leben kann man das positiv sehen
    =
    Meine Meinung:
    Man vergleicht das Universum gerne mit einem aufgeblasenen Luftballon und die
    Galaxien bewegen sich sinngemäß an der Oberfläche des Luftballons, sprich: Universums.
    Ein Luftballon wird aufgeblasen mit Luft oder Gas, allerdings nicht von jetzt auf sofort.
    Zum Aufblasen benötigt man eben eine gewisse Zeit und ein Medium.

    Im Universum gab es lt. Wikipedia zum Zeitpunkt der Entkopplung noch keine
    >Dunkle EnergieDunkle EnergieZeitDunkler Energie<.
    Demnach ist das Universum nicht 3- sondern 4-dimensional.

    Hinzu kommen 2 weitere Dimensionen: Die Eigendrehung und die Bahndrehung der
    kleinsten Bausteine des Universums, welche letztlich auch für die Krümmung des
    ganzen Universums mitverantwortlich sind.
    Als Laie bin ich sehr gespannt auf fundierte und nachvollziehbare Gegenargumente.
    W. B.

    • Guten Abend Herr Päs,
      Überschrift Ihres Artikels:
      Warum ist das Universum dreidimensional?
      Zitat:
      Wenn man diesen Argumenten folgt, wissen wir also eigentlich so genau gar nicht, ob das Universum wirklich dreidimensional ist. Aber wie fast alles im Leben kann man das positiv sehen
      Meine Meinung:
      Man vergleicht das Universum gerne mit einem aufgeblasenen Luftballon und die
      Galaxien bewegen sich sinngemäß an der Oberfläche des Luftballons, sprich: Universums.
      Ein Luftballon wird aufgeblasen mit Luft oder Gas, allerdings nicht von jetzt auf sofort.
      Zum Aufblasen benötigt man eben eine gewisse Zeit und ein Medium.

      Im Universum gab es lt. Wikipedia zum Zeitpunkt der Entkopplung noch keine
      “Dunkle Energie”.
      Derzeit beträgt der Anteil am Universum ca. 72%
      Verglichen mit dem Luftballon (Luft, Gas) muss die “Dunkle Energie” irgendwie
      in das Universum hineinkommen und das nicht auf einmal, sondern kontinuierlich.

      Erforderlich ist somit eine weitere Dimension, nämlich die “Zeit”, gekoppelt
      mit der Zufuhr “Dunkler Energie”.
      Demnach ist das Universum nicht 3- sondern 4-dimensional.

      Hinzu kommen 2 weitere Dimensionen: Die Eigendrehung und die Bahndrehung der
      kleinsten Bausteine des Universums, welche letztlich auch für die Krümmung des
      ganzen Universums mitverantwortlich sind.
      Als Laie bin ich sehr gespannt auf fundierte und nachvollziehbare Gegenargumente.
      W. B.
      Sorry, da hat etwas nicht funktioniert, ><?

  31. @L.Schaber … Quell des Lebens – also realiter geht das ja meistens weitaus schiefer als im Film. Der erste Kaiser Chinas Qin Shihuangdi, dem der Gedanke seiner Sterblichkeit ein einziger Graus war, unternahm so ziemlich alles, um selbiger zu entgehen, rüstete Expeditionen zu fernen Inseln der Unsterblichkeit aus, setzte seine Ärzte unter Druck und wandte sich schließlich den Schamanen zu. Diese verabreichten ihm – dem Cheffe konnte man schlechterdings keinen Wusch abschlagen, sonst sonst tat er ebendies mit den Häuptern der Heiler – quecksilberhaltiges Gebäck, damals der letzte Schrei auf dem Gebiet; mit dem Erfolg des baldigen himmelssöhnlichen Ablebens. Das soll Sie aber keinesfalls beunruhigen.

    @Heinrich Päs, L.Schaber, zur Erwähnung der Matrix. Ich mag die Wachowskis, aber die Sache mit der Ungewissheit über Wirklichkeit geht auch einfacher, ohne Computersimulation und Schnittstellen im Hirn. Gern verweise ich auf den Sketch Mir hat draamt i wär a Antn (Mir träumte ich wäre eine Ente), in dem Liesl Karlstadt den schlafenden Karl Valentin weckt. Der ist sauer, denn er hatte gerade geträumt, eine Ente zu sein im Begriff, einen leckeren fetten Wurm zu verspeisen. Den Einwand, er sei in Wirklichkeit doch der Valentin Karl und möge keine fetten Würmer, akzeptiert er nicht, denn im Traum eben war er doch die Ente, und Enten mögen eben … usw. Das ist insofern verblüffend, da K.V. meines Wissens kein Kenner des alten Chinas war, genau dort aber, im 4./3. Jt. V. u. Z., der daoistische Weise Zhuangzi in seinem Schmetterlingstraum das nämliche Szenario exakt abgearbeitet hat, in etwas anderer Besetzung.

    @Axel Krüger – sehr schön.

    • Ät Nonymus
      Danke . Den Karl kannte ich zwar schon lange, seinen Traum allerdings noch nicht. Wusste aber ,dass gewöhlich der Liebe Gott sich mit dem selben Problem heruzuschlagen hatte. Bezüglich einer Lösung ist mir aber (obwohl katholisch verbandelt) bisher noch nichts zu Ohren gekommen. Ich sollte mal die Margot fragen, die jemand in der Nähe mal live vor Ort ziemlich im Zentrum des hiesigen Koordinatensystems erlebt hat.
      Aber mal ganz nebenbei: Sind Sie aber gebildet! Mit Ihnen kann man ja sogar bei smalltalk noch was lernen. Ich glaub sogar in Nachbarblogs. Wie war doch gleich noch mal der Name?

  32. Ich glaub hier spinnen alle außer mir. Wo bleibt der Donnerblitz von halb da oben?
    Die reine Anarchie. Wo kommen wir denn da noch hin? Oder kommen wir am Ende von da grad her? Dann wärs kein Wunder.

  33. Das kommt von diesen vertrackten Knoten. Der weiter oben vorgebrachte Einwand, der Knotenansatz zum Handeln der Inflation sei etwas kompliziert – kann nicht beurteilen, ob das wirklich so ist. Man liest ja immer mal wieder, dass bei Physikern Konzepte/Theorien/Gleichungen sich aufgrund ihrer Einfachheit besonderer Beliebtheit erfreuen. Erstaunlich, und auch wiederum nicht, dass die Großmeister von Naturwissenschaft und Mathematik dabei regelrecht ins Schwärmen geraten und von “schönen” Gleichungen sprechen. Mich freut das immer über die Maßen, dass da auf einmal ästhetische Kategorien reinrutschen. Das hat entschieden was.

    Nochwas zum Feldbegriff. Hat nicht der Alte selbst das Konzept vom (Gravitations-)feld losgelassen, in der Allgemeinen Relativitätstheorie, indem er sagte, dieses Feld erstrecke sich nicht irgendwie im Raum, sei vielmehr Raum (/Zeit) selbst?

    • Ja, ja, ich weiß: Mal soll “Ockhams Razor” gelten und mal nicht. Meine Gattin sagt auch immer “das ist was Anderes” wenn ich auf Gleichbehandlung bestehe.
      Zu: “Feldbegriff als Raumzeit selbst” :
      O Gott, nicht schon wieder. Schauen Sie mal in die diesbezüglichen oder auch weniger bezüglichen Blogs (nicht nur hier und nicht nur der) letzten Monate.
      Obwohl, wenn ichs genau betrachten tu: Eigentlich macht das Welträtsel- Lösen sogar ein wenig Spaß.

      • Keine Gefahr, tummle mich ansonsten nur noch in einem Fußball-Blog. Obwohl…

        Im übrigen hat der Altmeister diese ganzen Problematiken so wunderbar runtergebrochen – ein ungewöhnlicher Plural, schon passts: Da er klang ein schmerzlich Ach! / Als das All, mit Machtgebärde, / In die Wirklichkeiten brach.

    • @Ah, nonymous:

      Sehr schöne Beiträge, unterhaltsam und lehrreich. Vielleicht sogar ein Grund, diese Kommentarspalte nicht einfach zuzumachen. Zu Ihren Fragen

      Hat nicht der Alte selbst das Konzept vom (Gravitations-)feld losgelassen, in der Allgemeinen Relativitätstheorie, indem er sagte, dieses Feld erstrecke sich nicht irgendwie im Raum, sei vielmehr Raum (/Zeit) selbst?

      Ja, in der ART ist die Raumzeit selbst eine Art Feld. Gravitation ist Krümmung der Raumzeit. Das heißt aber ja keineswegs, dass der Feldbegriff aufgegeben wurde, vielmehr wird er noch universeller benutzt. Und daneben gibt es natürlich nach wie vor Felder, die sich in der Raumzeit ausbreiten wie z.B. das elektromagnetische Feld.

      À propos. Zurzeit geistern ja Nachrichten herum, der Urknall habe ein für allemmal ausgeknallt. Frage an die Wissenden: ist das so? Und was passiert dann mit unseren Knoten?

      Erst einmal muss man sich klarmachen, was man überhaupt mit dem Wort “Urknall” meint, siehe meinen Beitrag Gab es den Urknall ?. Die momentan in der Presse diskutierten Argumente zweifeln an der Inflation, allerdings treffen nicht alle Kritikpunkte auf das Knotenszenario zu, wie z.B. die Kritik an zu viel Finetuning. Und die vertretenen Alternativvorschläge sind auch nicht unproblematisch, z.B. ist bei einer Vor-Urknall-Kontraktions-Phase mit anschließendem “Big Bounce” die Frage, ob unser klassisches Konzept der Zeit dort funktioniert, es gibt nämlich Argumente, dass Zeit immer in Richtung der kosmischen Expansion laufen sollte.

      Die Kazz oder die Nachbarin?

      Wurde das jemals beantwortet?

  34. Seit dem 7./8. Jahrhundert haben indische Mathematiker festgelegt, dass eine Division durch Null nicht sinnvoll/zulässig ist. Und ab dem 17.Jhdt. wurde von Mathematikern die Idee des Grenzwertes verwendet.

    D.h. wenn man einen Punkt als kleinstes Objekt definiert, dann muss dieser immer dreidimensional sein – wobei Länge, Breite, Höhe sich einem Grenzwert gegen Null, (aber größer als Null) annähern. Und angenähert an Euklids Beispiel, kann man daraus Linien, Flächen bzw. Körper zusammensetzen.

    Mathematisch ist genug definiert – man muss vorhandenes Wissen bloß nutzen. (Übrigens: Weil auch der kleinste Punkt dreidimensional sein muss – ist die Idee eindimensionaler Strings mathematisch nicht zulässig.)

    • Wenn schon Formalismus, dann gilt
      Die Frage ‘Was ist ein Punkt?’ mit der Frage ‘Was ist eine Dimension?’ bzw. ‘Was ist eine Gerade’ wechselseitig zu erklären ist formal ein Zirkelschluß.
      Euklidische Geometrie ist und bleibt immer eine reine Flächenbetrachtung.

      Alle drei Fragen sind im mathematischen Sinne gerade _nicht_ definiert. Ich verweise nochmals auf den Link zu Farhad Ghaboussi.

      Ganz so simpel ist genug definiert halt nicht:

      Was ist Null?

  35. @ Franz Maria
    Vielleicht haben sie die Präzision der Definitinen ja deswegen nicht erhöht, weil dann eventuelle Paradoxien zu offensichtlich zu Tage getreten wären. Deswegen bedeutete vielleicht ihre Definition einfach: Wir beschließen, ab hier einfach nicht weiter in die Tiefe zu denken, sondern nur noch seitwärts.
    Vieleicht war es aber auch nur eine bewusste Abstraktion (von) der Wirklichkeit aus erkenntnisökonomischen Gründen. (Reduktionismus)

    Zur Mathematik: Manche Theoretiker glauben ja, dass Mathematik das einzig sinnvolle Werkzeug zur Erforschung und korrekten Erkenntnis der Realität (Welt) sei, weil die Realität aus nichts anderem als purer Mathematik bestehe. (“Platonisten”). Das gefällt dann wiederum machen Mächtigen in der Welt Denn daraus kann man dann auch folgern: Am Anfang war das Wort (des Herrn!) und sonst gar nichts. Dann wird aber auch wieder durch Definition des Begriffes “Gott der Herr” festgelegt: Das ist der, der aus sich heraus besteht und deswegen (was jetzt wirklich einmalig ist) keines (Selbst-)
    Verursachers bedarf. Somit ist Gott sowas Ähnliches wie das Universum ohne Urknall.
    Wer Maria heißt, sollte sich da ja auskennen.

    Man sieht: Mit Weiterdenkverboten durch anderen gewaltsam auferlegte Eigendefinitionen kann man viel erreichen. Nicht nur in Philosophie und den Wissenschaften. Vor allem dann auch in der Politik.
    Und jetzt mal etwas skurril zynisch: Manche nennen das
    “kontrollierte weitgehende Zerstören von zuvor als ziemlich ungefährlich erkannten Päckchen mit einer Blechdose mit ein paar Nägeln ,ein paar Drähten und einem Chinaböller”
    ja auch nicht “Vernichtung von Beweismitteln” , sondern “Kontrollierte Sprengung zur Abwendung einer unmittelbahren Gefährdung der Bevölkerung” .
    Wobei der Eine gestern Abend erzählte, das Päckchen sei gesprengt worden, der andere aber heute morgen im DLF es sei mit einen Wasserstrahl aus einem Feuerwehrschlauch bearbeitet worden .
    Wie dem auch sei. Vielleicht alles nur mediale Verzerrungen. Ich kann ja kaum verifizieren, was dort ganau passiert ist. Heute wird von offizieller Seite ja nicht mal mehr kommuniziert, was ganz genau zu welchem genauen Zeitpunkt wo und von wem getan wurde/ passiert ist.

    Himmel Herrgott, schon wieder vom Thema abgekommen. Ich sollte jetzt endlich die Zeitung lesen. Ich hab nämlich noch sowas altmodisches. Sonst weiß man ja nicht mal,
    was die Regierung will, dass man glauben soll.

    • Ich denke auch, dass wir Menschen uns zu sehr in die Mathematik vertiefen und diese als “allwisendes für das Erklären des Universums” hernehmen. Es gibt momentan aber keine andere Konkurrenz zur Mathematik. Wir haben diese in der Schule gelernt, studiert und nie gefragt: “Ist die Mathematik so etwas wie ein Glaube?” In Religion musste ich viel schmunzeln über dass, was man lernen musste, nur um bei der Schulaufgabe gute Noten zu schreiben.

      Das Universum in dem wir uns befinden gibt die Antwort in keinem Fach wieder. Philosophisch frage ich oft: “Warum überhaupt etwas Seiendes und nicht absolut nichts?”

    • Die Frage, die sich mir als Maria stellt, ist:
      Was ist mit Gott gemeint?

      1. Ein höheres Wesen?
      Dann existiert ein Teil, Endlichkeit und alles ist bereits gewiß.

      2. Eine sich stetig verändernde Ganzheit?
      Dann existiert Unendlichkeit und alles ist ungewiß.

      Frei nach Genesis 2.16-17
      Der Baum der Erkenntnis von Gut und Böse
      Wer _urteilt_, die Welt in Gut und Böse _TEILT_, der wird sterblich.

      Nur eine Frage der Abstraktionsebene.

  36. Franz Maria Arwee,
    Definition,
    Der Vorteil von verschiedenen Definitionen ist, dass sich die Mathematik nicht selbst beengt.
    Eine gute Definition, aber darin gibt es auch wieder schiedene Sichtweisen,über Dimension liefert die Theorie über Fraktale. Erst wenn Sie verstehen, was ein Fraktal ist, dann bekommen Sie einen Überblick, wie Punkte, Flächen und Körper zu sehen sind. Alles sehr sehr interessant, und jede Minute ist gut angelegt.

  37. Die Idee, dass das woraus das Universum letzlich besteht, bestimmt, wie das Universum als Ganzes aussieht(beispielsweise in Bezug auf seine Dimensionalität), gefällt mir. Als Analogie sehe ich das Verhältnis Mensch und die vom Menschen geschaffene Welt und Zeit ( das Anthropozän). Ich behaupte: Wie Städte, Wege, Länder etc. strukturiert sind, lässt sich teilweise auf psychische, biologische und morphologische Gegenheiten des Geschöpes Mensch zurückführen. Doch das sieht man den Städten und allem anderen, was die Menschen geschaffen haben, auf den ersten Blick nicht an. Genau so könnte es für das Verhältnis Elementarteilchen Universum sein.

  38. @Arwee: Mein Beitrag sollte auf ein Problem hinweisen – z.B. eindimensionale Strings kann es nach den Regeln der Mathematik nicht geben.
    Weiter oben habe ich darauf aufmerksam gemacht, dass es die Zeit-DAUER als 4. Dimension in der Realität nicht geben kann – und dass dies seit 2500 Jahren bekannt ist.

    Z.B. ist nicht bekannt, wie unser Universum genau entstanden ist. D.h. weil wir nicht wissen wie unser Universum entstanden ist – können wir keinerlei Aussage darüber machen, ob es möglich ist, dass überhaupt noch ein weiteres andersartiges Universum entstehen kann. Und wenn String-theoretiker die Möglichkeit von 10exp500 andersartigen Universen diskutieren (siehe einleitender Blogbeitrag) – dann ist jede wissenschaftlich nachvollziehbare Denkweise dabei verloren gegangen.

    Die Physik hat heute teiweise eine ´Qualität´, die man nur noch als Esoterik bezeichnen kann.

    • 10exp500 ist doch nicht soviel, daß man nicht ein paar passende raussortieren könnte.
      Ist doch nur der Formalismus der Möglichkeiten, wieviel Punkte hat ein Kreis (?) und
      doch beherrschen wir seinen Formalismus, aber nicht seine Quadratur per einem
      Konstruktionskonstrukt, trozdem geht’s mit knotenlosem Faden und 4 Nadeln.

    • Hatte ich in der Tat mißverstanden, sorry dafür.

      Betrachten wir meine vorige Antwort einfach als Ergänzung.:-)

  39. MH,
    wie im Großen so im Kleinen, wie im Kleinen so im Großen,

    das ist ein sehr vernünftiger Denkansatz. Ohne die Planeten wäre man nie auf die Idee von Atomen gekommen.
    Die Quantenphysik steckt ja in Erklärungsnot, weil sie noch kein passendes gegenstück im Universum gefunden hat.
    Ich stelle mir die elektromagnetischen Felder als “Materienebel” vor. Der kann wie Wasser kondensieren und bildet dann die Elektronen. Nur als Beispiel.

    • Wenn es wirklich ein Teilchen gibt, dann ist dies eine absolute und daher eine einzig wahre Wahrheit!
      Willkommen in der Glaubenschaft.

  40. In einem grünen Hochglanzblättchen von G&J liest an (auf S. 120/121) einen PR- Artikel zum im April beim großen Mutterhaus erscheinenden Buch von einem Daniel Whiteson, seines Zeichesns Lehrer für Experimentalphysik an der University of California in Irvine.
    Der Herr schaut dort ins Jahr 3000, sieht wie die dortigen Wissenschaftler über unsere heutigen Theorien lächeln und vermutet, dass wir dort in einem “Physiklehrbuch” (!) kaum etwas über anerkannte Theorien unserer Zeit finden würden. Er fordert deshalb mehr (staatliche) Mittel für die Grundlagenforschung, und zwar weil er angesichts der derzeitigen Lage in der Physik “dramatische neue Einsichten” für erforderlich hält und auch prophezeit.

    Zitate aus dem Artikel:

    “………..Die dramatischen neuen Einsichten dürften sich künftig eben gerade nicht auf Feldern offenbaren, in denen unsere Ignoranz auf der Hand liegt – sondern viel eher dort, wo wir unsere konzeptionellen Denkfehler bislang noch nicht einmal bemerken…………..”
    “…..Damit wir unsere Erkenntnisfähigkeit verbessern, müssen wir uns über heutige Denkstrukturen hinauswagen und unseren Kopf für völlig neue Denkstrukturen öffnen….”
    “……Man könnte natürlich frage: Warum sollten wir uns die Mühe machen? Was spielt es für eine Rolle……….(zu erforschen) …….” ob das Universum endlich oder unendlich ist? Aber diese scheinbar abstrakten Fragen helfen uns, die fundamentalen Fragen zu beantworten: Warum sind wir hier, und wie sollten wier unser Leben führen? “
    (Ende der Zitate)
    Na Donnerwetter . Vor 30 Jahren (und auch heute noch gelegentlich) hätten die meisten “Experimentalphysiker” so etwas als weit von sich gewiesen und es ins Imperium der religiösen Metaphysik abgeschoben, weil es nicht zu ihrem Verständnis von Naturwissenschaft passe.

    Und „konzeptionelle Denkfehler“ und auch “Ignoranz” warf man sich in den hiesigen Blogs zu dieser Thematik in der Tat sehr häufig gegenseitig vor. Vor allem von den Physikern wurden manche “penetranten Disku- Tanten” öfter mal als „zu esoterische und bornierte Fundamentalkritiker“ gegeißelt, deren Engagement entweder rassische oder semitische oder antisemitische oder logische oder auch unlogische und unvernünftig spinnerte Motivationen habe.

    • Hallo Herr Senf,

      ich habe das von Ihnen zitierte Paper gelesen. Finden Sie das erhellend ?

      Ich kann doch kein Paper über die physikalische Entstehung von Dreidimensionalität schreiben, ohne zuvor den Begriff der Dimension, seine Bedeutung, das Warum und das Werden des Dreidimensionalen metaphysisch zu erörtern und die Begriffe, die ich beabsichtige zu verwenden, zu definieren.

      Ich kann doch nicht auf einer metaphysisch vollkommen unbestimmten Basis Physik machen.

      Wenn ich von einem Photon im n-dimensionalen Raum rede, dann muß ich doch zuvor sagen, was ich unter einem Punktteilchen im realen (!) n-dimensionalen Raum verstehen will. Ein Teilchen, insbesondere das spezielle Teilchen Photon, ist nun mal ein Gebilde, das ausschließlich über eine Lokalität im 3-dimensionalen Raum definiert ist, sogar dort eine Punkt-Lokalität – von der Realexistenz von Etwas im n-dimensionalen Raum zu sprechen, das das nur im 3-dimensionalen Raum definiert ist, macht doch überhaupt keinen Sinn. Nachdem die Autoren auch behaupten, die Entropie würde die Richtung der Zeit erzwingen, wo sie doch nur ein beschreibender Parameter ist, habe ich den Artikel als uninteressant beiseite gelegt.

      Auch Herr Päs hat es versäumt, auf die metaphyische Frage, die Thema seines Betrags ist, auch nur näherungsweise eine metaphyische Antwort zu geben, wenigsten am Anfang. Stattdessen beschreibt er ein Sammelsuriums an komplizierten physikalischen Ideen, die keiner versteht, und aus denen auch nicht deutlich wird, warum man überhaupt solche Überlegungen anstellt. Physik ist ja schön, aber ohne metaphyischen Unter- und Hintergrund nicht besonders sinnhaft.

      Entsprechend brauchen wir uns über die Kommentare nicht zu wundern.

      Und Sie verweisen noch auf ein Paper, das genau den gleichen Mangel hat. Vielleicht um zu zeigen, daß es viele Elaborate dieser Art gibt ?

      Grüße Fossilium

    • Hi Franz Maria Arwee,

      Sie stellen die richtige Frage: was ist eine Dimension ?

      Das hätte Herr Päs erklären müssen. Ich versuchs mal an seiner Stelle, und weiß aber nicht was dabei herauskommt:

      Damit wir uns darüber sinnvoll unterhalten wollen, müssen wir erstmal davon ausgehen, daß die Welt aus einzelnen Objekten (nicht Teilchen !) besteht, die bestimmbar sind, und es stellt sich die Frage, welche Bestimmungsmannigfaltigkeiten stehen da zur Verfügung. Ich will ja zunächst ein Objekt überhaupt als Individuum wahrnehmen, und dazu gehört ein Hier und Jetzt als fundamentalste Bestimmungsgrößen. Die Abstrakta „Hier“ und „Jetzt“ erhalten aber erst dann einen konkreten Sinn, wenn ich diese Begriffe vor einem Hintergrund benutze, auf den ich dieses Hier- und Jetzt als konkrete Lagepositionen abbilden kann. Dimensionen im obigen Sinn sind gewissermaßen Formen oder Artikulationen eines solchen Hintergrundes. Hier und Jetzt-Punkte als reine additive Elemente einer Menge ergeben aber immer noch etwas Sinnloses, weil etwas hinzutreten muß, was sie in eine umfassende Beziehung zueinander setzt. Erst dann bilden sie etwas Konkretes. Der Hintergrund muß also eine „Struktur“ haben, die dies leistet. Ich behaupte jetzt einmal, zunächst intuitiv, daß ein dreidimensionaler Raum und die Zeit die „einfachste“ Struktur sind, und zwar als logischen Gründen (logisch heißt hier: wenn wir eine logische Beschreibung haben, ist dieser näher an der Wahrheit als eine unlogische).

      Der Zustand eines jeden Objektes muß eindeutig bestimmbar sein. Orts- und Zeitpunkte per se erfüllen diese Eindeutigkeit nicht, woher auch ? Ein Zustand ist nicht gehindert, eine Raum oder Zeitstelle mehrfach zu besetzen, indem er zu einer ehemaligen Stelle zurückkehrt – und ist dann vom vormaligen Zustand nicht zu unterscheiden ist (z.B. nach der Drehung eines Zustands um 360 Grad um eine „Achse“ (AnfüZeichen beachten) hat man den identischen Zustand wie vor der Drehung). Um solche Identitätsprobleme auszuschließen, müssen entweder Orts- oder Zeitpunkte eine Einschränkung erfahren, mit der die Eindeutigkeit herstellt wird. In der einfachsten denkbaren Raumzeitstruktur mit einem Orts- und einem Zeitparameter funktioniert dies nicht, es funktioniert erstmalig im dreidimensionalen Raum mit einer eindimensionalen Zeit: hier nehmen die Zeitpunkte eine Einschränkung auf sich (eindimensional – und gerichtet (!)), so daß verhindert wird, daß niemals an einer Orts- oder Zeitstelle ein unbestimmter Zustand in der anderen Dimension entsteht.

      Das alles mag noch vage sein, es ist aber wenigstens eine in sich konsistente Definition von Dimensionalität und eine Vorab-Erklärung, warum beim Werdungsprozeß beim Dreidimensionalen Halt gemacht wird. Die Natur realisiert alle Prozesse auf ökonomischste Weise, diese Erkenntnis stammt aus der Physik und findet im Prinzip der kleinsten Wirkung seinen überzeugensten Niederschlag. Sie wird auch, wenn aus dem Nichts etwas entstanden ist, die Dreidimenionalität auf ökonomische Weise herstellen, selbst wenn sie noch die Inflation einschieben muß, um zu einem einfachen Ausgangspunkt zu kommen.

      Wie also erzeugt die Natur die grundlegende Ordnungsstruktur unserer Welt ? da kann die Physik helfen, aber nicht, indem sie mittendrin ohne metaphyisches Fundament mathematisch loslegt.

      Wenn die Physik diese Frage beantworten will, muß Sie auf diesen Überlegungen (oder auf anderen ähnlichen) aufbauen. Das meine ich mit metaphysischem Untergrund, auf dem die agiert. Wer da ankommt und sagt: ha, ich habe da ein mathematisch-topologische n-dimensionales Kneuel, das kann man mathematisch im Dreidimensionalen entwirren, sonst nicht, dann sagt doch jeder: na und ?

      Grüße Fossilium

      • Man muß sich pragmatisch wohl “ungefähr zufrieden” geben:
        https://de.wikipedia.org/wiki/Dimension_(Mathematik)
        Der Begriff der Dimension tritt in einer Vielzahl von Zusammenhängen auf. Kein einzelnes mathematisches Konzept vermag es, die Dimension für alle Situationen zufriedenstellend zu definieren, darum existieren für verschiedene Räume auch unterschiedliche Dimensionsbegriffe.
        Ist das Minimum an Freiheitsgraden, die wir brauchen, etwas zu beschreiben,
        ansonsten ist es dem Physiker aus dem Kontext klar, was gemeint ist – wie Raum.

        • Hallo Herr Senf
          Was ist denn ein Freiheitsgrad ? ich haben keine Ahnung, was das sein soll.

          Und das, was Dimension ist, erschließt sich mir nicht aus dem aus dem Kontext.
          Bin ich nicht Physiker genug.

          Grüße Fossilium

      • @fossilium
        Mir ist durchaus klar, was mir ein anderer Mensch sprachlich mitteilt, wenn er sagt, das ist 1-, 2-, 3- oder 4-dimensional. Sprache als Informationsträger ist nicht die Information selbst, wobei es keine Information ‘an sich’ geben kann.
        Entweder kann nichts nicht Raumzeit sein oder

        Was ist zwischen den Objekten?

  41. Hallo Herr Senf,

    bin hocherfreut und dankbar, daß Sie sich trotz meiner kritischen Anmerkungen weiterhin mit Schlagworten an der Diskussion beteiligen wollen.

    Grüße
    Fossilium

  42. Man sollte sich mal klar werden, was man will.
    Wild drauflos philosophieren oder die Begriffe in eine mathamtische Form bringen.

    Und dann kann man entscheiden, ist das hier Zeitverschwendung, weil wir nach Marx die Welt nicht interpretieren sollen, sondern verändern, oder verändert uns der Erkenntnis- Gewinn? Ich neige zum Zweiten, deswegen beteilige ich mich hier.

    Auf etwas radikal Neues warte ich noch immer. Wir sollten mal einen Aborigines einladen.

  43. Wir leben in einer dreidimensionalen Welt weil nur eine dreidimensionale Welt mit nach vorne gerichtetem Zeitpfeil ein Leben wie wir es kennen ermöglicht. Auch die Physik wie wir sie kennen ist nur im dreidimensionalen Raum möglich.
    Mathematisch kann man natürlich alle möglichen Raumdimensionen ersinnen aber nur mathematisch. Durch jede weitere Dimension würden die vorhandenen drei Dimensionen plus Zeitpfeil nicht mehr klar zu definieren sein.
    Sie könnten keine Koordinaten definieren. Leben braucht Sicherheit und feste Größen.

  44. Leben braucht vielleicht nicht feste Größen, aber ein komplexes Zusammenwirken verschiedenster Elemente, die es so, wie es ist, ermöglichen. Wäre das Zusammenspiel anders, würde es ein anderes Leben ermöglichen. So gesehen ist das ziemlich gleich-gültig, nur von uns aus gesehen nicht.

    Ich würde ja gern mal die Aufmerksamkeit vom Raum auf die Zeit lenken (eh dasselbe, klar). Damit zwingt ihr mich, Leute, meinen alten rotweingetränkten Körper zu dieser Zeit noch zum Bücherregal zu schleppen; old fashioned, ich weiß, geht aber in dem Fall nicht anders.

    Es gibt da diesen ziemlich abgefahrenen Zenmeiser Dôgen und sein Hauptwerk, das Shôbôgenzô. Auf Deutsch in 4 Bänden erschienen. Darin das 11. Kapitel: Uji. Uji wird im Deutschen in der Regel mit Sein-Zeit übersetzt. Persönlich würde ich SeinZeit, ohne Bindestrich, vorziehen.

    Gut, und das alles aus dem 13. Jahrhundert stammend, nachdem Dôgen von einer langen Reise nach China – damals eine lebensgefährliche Unternehmung – das Soto-Zen nach Japan brachte, bis heute eine von zwei entscheidend wichtigen Zen-Schulen.

    Zu Uji schreibt Dôgen: “Weil nur dieser eine Augenblick wirklich ist, sind alle Augenblicke der Sein-Zeit das Ganze der Zeit, und alle existierenden Dinge und Phänomene sind Augenblicke der Zeit. Das ganze Sein und das ganze Universum existieren in jedem dieser individuellen Augenblicke der Zeit.”

    Stark verkürzend kann man sagen, dass authentische, direkte Erfahrung der Wirklichkeit nur je und je im Augenblick möglich ist. Erinnerung und Zukunft mit ihren Plänen, Bestrebungen, Sorgen etc. sind demgegenüber sekundäre Konstruktionen – nicht überflüssig, aber von relativer, rein pragmatischer Bedeutung.

    Und Dôgen fährt fort: “Das ganze All bewegt sich nicht und steht auch nicht still, und es strebt weder vorwärts noch rückwärts. Es geht einfach von einem Augenblick zu nächsten.” D.h. in der Unermesslichkeit seiner Bezüge ist es nur jetzt da, und jetzt, und jetzt.

    Ich finde dieses achthundert Jahre alte Denken überaus scharf und überraschend. Und es würde mich interessieren, ob die zeitgenössische Physik etwas anfangen kann,und wenn ja: was.

    • Lesen Sie den/meinen ersten Kommentar: diese Idee ist schon 2500 Jahre alt.
      Die Physik ignoriert bekanntes Wissen.

      (Bischof Augustinus vertritt die gleiche Sichtweise. Seine Texte sind im Reclam-Buch leichter zugänglich)

      • Radio Eriwan antwortet: im Prinzip, ja. Allerdings hat erst Dôgen diese im Buddhismus angelegte Zeiterfahrung so scharf und radikal formuliert und so universal bezogen. Dazu durchaus auch widersprüchlich, denn ohne immanenten Widerspruch konnte eine solche Aussage für ihn nicht ganz sein. Er steht aber in einer Tradition, ja.

        Augustinus seinerseits ist in der Angelegenheit ja fein raus. Es sei ein eigenartig Ding, die Zeit – so seufzt er sinngemäß – : frage man ihn danach, dann stehe er völlig auf dem Schlauch. Ungefragt habe er eine gewisse Gewissheit davon. Ich geh jetzt mal, ihn nicht fragen.

    • Ideen sind Ideen sind Ideen … aber deswegen noch lange kein Wissen.
      Wissen muß belegt werden, man geht von einer These (hoppla Idee) aus.
      Angelesene Ideen ohne zeitnahen Beleg (wußten das schon vor 1000 Jahren)
      weiß man zwar auch, wenn man sie gelesen hat, waren aber nie Wissen,
      bleiben nur Ideen und Privatwissen, aber kein Wissenschaftswissen, auch
      wenn sie sich heute in “Nähe” der Wissenschaft zeigen. Pi wußte alles.

      • @ Herr Senf
        dazu
        Eine These ist eine Behauptung.

        Was ist eine Idee (Ideenleere)?

        Was kann ein Mensch über eine (platonische) Idee wissen?

        Ist eine Idee tatsächlich eine Behauptung?

        Ist die Sprechakttheorie nicht viel mehr das wirkliche Problem, das zur Lösung erklärt wird?

        Sprechen ist zweifellos eine Handlung, aber ist nicht- sprechen nicht auch eine Handlung?
        Wo bleibt die Schweigaktthorie?

        Kann denn irgendetwas _nicht_ nicht-handeln?

        • Was ist eine Idee (Ideenleere)? Der Witz ist gut.
          Leere=0, demzufolge Idee=0 mit ohne Beleg auch ohne Wissenswert qed.

  45. @HerrSenf
    Auf jedem Flohmark können Sie einen mechanischen Wecker finden, der auf der Rückseite einen +/-Hebel hat, um Fehler bei der Ganggenauigkeit zu kompensieren. Denn jeder Uhrmacher weiß, dass die Uhr eine Maschine ist, deren Ganggenauigkeit von den Umgebungsbedingungen abhängt. Das ist mit Fakten belegbares Wissen.
    Wenn sich durch die Umgebungsbedingungen die Anzeige einer Uhr verändert, dann geht die Uhr falsch.

    Physiker beachten nicht, wie die Maschine ´Uhr´ funktioniert – für sie gilt nur, was die Zeiger der Uhr für eine Uhrzeit anzeigen. Und wo für den Uhrmacher die Uhr falsch geht, ist dies für den Physiker der Beleg, dass die Zeit relativ ist – das ist dann Wissenschaftswissen, mit dem man dann sogar Zeitreisen berechnet.

    Es ist sowohl sinnvoll alte Ideen auf ihre Brauchbarkeit zu überprüfen – wie es auch sinnvoll ist, Wissenschaftswissen zu hinterfragen. Z.B. geht die Wissenschaft heute davon aus, dass es ZEIT seit dem Beginn des Universums (Urknall) gibt. Wissenschaftliche Begründung wie man zu dieser Überlegung kommt, gibt es allerdings keine – sondern man hat einfach eine Idee des Bischof Augustinus geklaut – dieser geht davon aus, dass es für Gott keine Zeit gibt und dass er erst die Zeit mit dem Universum erschaffen hat. Man mag mich gerne korrigieren – aber bisher kann ich keine wissenschaftliche Begründung finden, wo die Zeit herkommt.

    Wenn von 3+1 Dimensionen gesprochen wird, wäre es nett, wenn es nachvollziehbare Begründungen geben würde.

    • @KRichard

      Physiker beachten nicht, wie die Maschine ´Uhr´ funktioniert – für sie gilt nur, was die Zeiger der Uhr für eine Uhrzeit anzeigen. Und wo für den Uhrmacher die Uhr falsch geht, ist dies für den Physiker der Beleg, dass die Zeit relativ ist

      Sorry, aber das ist nun wirklich dummes Zeug. Die Ignoranz, die Sie den Physikern unterstellen, ist eher Ihre eigene. Wenn beispielsweise wieder irgendwo das Hafele-Keating-Experiment wiederholt wird, dann wird mit mehreren Uhren pro System gearbeitet und alle möglichen Überlegungen angestellt, um schädliche Einflüsse zu eliminieren.

      Oder glauben Sie ernsthaft, die Physiker von heute sind bescheuklappte Deppen, die gebannt auf ihre Uhren schauen und ansonsten nicht mitkriegen, was links und rechts passiert, während Ihr tief im Aberglauben verhafteter Bischof Augustinus der einzige Durchblicker war?

  46. Spritkopf,
    Hafele-Keating-Experiment. Sehr richtig. Wenn man sich vorstellt durch Wasser zu schwimmen und dann durch Sirup, dann wird es klar, dass man bei Sirup längere Zeit braucht. Wenn ein Austauschteilchen in einem stärkeren Gravitationsfeld ist, braucht es länger, ergo, die Uhr geht langsamer. Physikalische Vorgänge laufen in einem starken Gravitationsfeld langsamer ab, genau wie der Schwimmer im Sirup.

  47. @Spritkopf: Dass die Ganggenauigkeit einer Uhr von den Umgebungsbedingungen abhängt – weiß jeder Uhrmacher. Das Hafele-Keating-Experiment hat genau dies bestätigt.
    Ich verstehe Ihre Aufregung nicht.

    Dass die Sichtweise des Bischof Augustinus bis heute von der Physik übernommen wird – daran bin ich völlig unschuldig!

    Durch Rückrechnung der Ausbreitungsgeschwindigkeit des Universums, kommt die Physik zum Ergebnis, dass dieses vermutlich vor 13,8 Mrd. Jahren beim ´Urknall´ entstanden ist. D.h. man hat die Entstehung des Universums auf diesen Zeitpunkt datiert – aber eine eigenständige Definition für das Wesen von Zeit, von Seiten der Physik, gibt es nicht. Spephen Hawking schreibt in seinem Buch ´Eine kurze Geschichte der Zeit´ (ISBN: 978 3 499 62600 5 – Seite 21): “Das Modell eines expandierenden Universums schließt einen Schöpfer nicht aus, grenzt aber den Zeitpunkt ein, da er sein Werk verrichtet haben könnte!”

    • Dass die Sichtweise des Bischof Augustinus bis heute von der Physik übernommen wird
      Ich bin zwar Physiker, kannte aber den Augustinus nicht, bis Sie begonnen haben, ihn
      hier abergläubisch zu missionieren, Laienpredigen ist halt so einfach, warum überlegen.

      • Es geht mir nicht darum, zu missionieren.
        Ein Wissenschaftler sollte eigentlich Quellenstudium betreiben – und wenn Sie wirklich Physiker sind dann ist die buddhistische bzw. Bischof Augustinus´s Sichtweise zur Zeit eine Quelle, die man nicht ignorieren darf. Beide unterscheiden deutlich zwischen der Realität von Zeit und unserer Wahrnehmung von Zeit. Daraus ergibt sich z.B. dass es die Zeit-DAUER als reale 4. DImension(DAUER) nicht geben kann. Somit sind einige Ideen der Physik fragwürdig: 4D-Raumzeit, Wurmloch, Zeitreisen, Blockuniversum.
        Wenn Herr Päs wie oben geschrieben, die Idee von 3+1 Dimensionen akzeptiert, dann werden seine Vorstellungen von der Ausbreitung des Universums (Knotenmodell) von Anfang an falsch sein. Und wenn nicht bedacht wird, dass seit Epikur die Mathematik weiter entwickelt wurde – dann entstehen ebenfalls falsche Modelle (eindimensionale Strings).
        Wir können nicht wissen, ob es die Möglichkeit gibt, dass ein von unserem Universum verschiedenes zweites Universum entstehen könnte – da wir noch nicht einmal die Entstehungsbedingungen unseres Universums genau kennen. Wenn aber String-Theoretiker über 10exp500 mögliche unterschiedliche Universen diskutieren – dann fehlt hierzu jede Basis

        • Nachtrag: Man kann nicht den Buddhismus bzw. Bischof Augustinus dafür verantwortlich machen, dass heutige Physiker mögliche Quellen nicht zur Kenntnis nehmen.
          Seriöse wissenschaftliche Arbeit sollte anders verlaufen: Wissenschaftler sollten eigenständig Quellen auf Brauchbarkeit untersuchen.

    • @KRichard

      Dass die Ganggenauigkeit einer Uhr von den Umgebungsbedingungen abhängt – weiß jeder Uhrmacher. Das Hafele-Keating-Experiment hat genau dies bestätigt.

      Nochmal: Dass eine Uhr im Gravitationsfeld langsamer läuft, hat nichts mit ihrer Ganggenauigkeit zu tun. Sie läuft deswegen auch nicht falsch, wie Sie hier fälschlicherweise behauptet haben. Es ist die Zeit, die langsamer vergeht und die Uhr richtet sich danach. Solange Sie das nicht verstanden haben, brauchen wir uns nicht weiter zu unterhalten.

  48. Was ist Zeit?
    Was ist ein Takt?
    Ist beides dasselbe?
    Zeigt eine Uhr die Zeit an?
    Gibt es wirklich unterschiedliche Zeiten?

    Hat Unendlichkeit einen Maßstab?

    Kann in der Raumzeit wirklich etwas Nicht-raumzeitliches dar- oder (zumindest) vorgestellt werden, beispielsweise eine mathematische Dimension?

  49. Franz Maria Arwee,
    hat die Unendlichkeit eine Skala. Aber natürlich, wenn die Unendlichkeit einen Anfang hat.
    Die notwendigen Begriffe/Sätze dazu sind die Erhaltungssätze in der Physik und der Begriff der Energie.
    Du kannst die Energie eines Gravitationsfeldes auf das Komma genau berechnen, seine Ausdehnung aber nicht.
    Mit Intelligenz läßt sich dieses unendliche Feld auch skalieren. Man legt den O-Punkt in die Unendlichkeit, das Maximum der Gravitationsfeldstärke auf die Oberfläche des Planeten. Dann ist die Gravitationsfeldstärke nach 2r nur noch ein Viertel der Gesamtfeldstärke, die Energie dieses Feldes beträgt dan genau die Hälfte der Gesamtenergie. Dann nach 3r hast du 1/9, nach 4r ein 1/16 der Gravitationsfeldstärke usw.
    Bei Zahlenreihen:
    Mit Kettenbrüchen kannst du unendliche Dezimaldarstellung endlich machen.
    alle deine Fragen lassen sich per Definition beantworten. Über das Wesen der Zeit , da muss auch ich passen.

    • Einer der Begriffe, der in der Mathematik nicht verstanden wurde und bis heute auch nicht wird.
      Ohne Ende zu zählen ist keine Unendlichkeit. Das Paradies Georg Cantors ist eine Chimäre des Reduktionismus:

      Die Unendlichkeit hat per Definition _keine_ Grenzen, sie hat daher auch keinen Maßstab und keine Größe. Es existiert _kein_ außerhalb und folglich auch kein innerhalb der Unendlichkeit.
      Die Unendlichkeit ist nicht bestimmbar. Die Unendlichkeit ist ungewiß.
      Unendlichkeit ist mit der Sprache Mathematik nicht beschreibbar.

      In der Unendlichkeit können weder Anfang, noch Ende, in Folge auch _keine_ Teile, Objekte, Subjekte, Einzelnes ‘an sich’ existieren, ansonsten kann es _nicht_ die Unendlichkeit sein.

  50. FMA,
    du unterschätzt unsere Phantasie. Für mich ist das Unendliche viel einfacher zu denken wie eine Grenze.
    Das ist doch der große Vorteil von Mathematik, dass wir mit diesem Werkzeug die Grenzen unserer Vorstellungskraft überschreiten können. Eine mathematische Formel und deren Lösung kann Entzücken auslösen, genau wie der Anblick eines Regenbogens.

    • Etwas, das _nicht_ existiert, kann sich auch keiner vorstellen…

      Das ist es, was Wissenschaft von Religion unterscheidet, jedenfalls nach dem eigenen Anspruch.

  51. Hallo Herr Senf,

    “Ideen sind Ideen – aber deshalb noch lange kein Wissen. Wissen muß belegt werden…
    Angelesene Ideen … waren nie Wissen, sind… kein Wissenschaftswissen.”

    Das hört sich so an, als ob Wissenschaftswissen das Wissen sei, das empirisch belegbar ist.

    Herrn Bäs hat mit einem hochkarätigen Team von Physikern an der Frage gearbeitet, ob sich dreidimensionale Raum aus einer n-dimensionalen Mannigfaltigkeit – metaphorisch ausgedrückt: aus einem n-dimensionalen Kabelkneuel – durch topologische Weiterentwicklung vor oder im Zuge der Inflation hausgefaltet habe – lesen sie oben.

    Welches Wissen erwirbt Herr Päs ?

    Ist das ein empirisch belegtes Wissen ? ist es eine Verifikation der Vorhersage der Theorie, wenn die Verifikation trivial ist ? Wenn die Theorie eine Prozessanalyse ist, ist der Ausgangspunkt und sind die Prozessschritte verifizierbar ?

    Also zu welchem Wissen führt diese Wissenserwerbs-Unternehmung von Herrn Päs ?

    Sie können es sich auch einfach machen: sagen Sie uns doch einfach, warum die Ideen, die im Rahmen des Päs-schen Unternehmens aufkommen und durchdacht werden, den Ideen des Augustinus überlegen sind.

    Grüße
    Fossilium

    • Damals zu Augustinus’ Zeiten war es noch Science fiction, nicht Science.
      Heute kann man nicht sagen, daß es seit damals schon “gewußt” wurde,
      auch wenn später einiges davon in den Konsens des Mainstream überging.
      Gutes Pi gehört dagegen schon lange zum Stand der Technik.
      Grüße Dip

  52. Sorry, ich stellte Ihnen eine Frage.
    Sie antworten daran vorbei und irgendwas.
    Ich dachte man könnte mit Ihnen reden, hart und sachlich.
    War wohl ein Irrtum.
    Ist kein Physiker mehr da.,
    Herr Päs ist auch weg.
    Lassen wirs dann.
    Adieu
    Fossilium

  53. fossilium
    …..Augustinus gegen den Rest der Welt….
    Jeder wählt die Philosophie, die zu ihm passt. Besser oder schlechter, das ist schon wieder eine Wertung, geeignet für…. das ist eine neutrale Sichtweise.
    Wenn man sich dessen bewußt sit, dann sind alle Streitigkeiten über Philosphie und Wissenschaft nur Scheinstreitigekiten.

    • Die buddhistische Vorstellungen zur Zeit sind seit 2500 Jahren bekannt und überliefert. Aktuell gibt es mehrere Millionen Buddhisten. Augustinus´s Buch ´Bekenntnisse´ gehört zu den wichtigen Grundlagen europäischer Kultur – bei Reclam gibt es eine Ausgabe für 10 Euro.
      Selbst wenn man annimmt, dass ein Physiker überhaupt keine Allgemeinbildung hat/braucht – dann muss er zumindest physikalisches Fachwissen haben: Eine Uhr ist nichts anderes als eine Maschine, die Wärme abgibt und eine Uhrzeit anzeigt. Die Gesetze der Thermodynamik gehören zum Grundwissen jedes Physikers! Und wenn Uhren unterschiedliche Zeiten anzeigen, dann ist nicht die Zeit relativ – sondern die thermodynamischen Bedingungen der Maschine sind unterschiedlich.
      Aber wenn es Physiker gibt, die sogar Überlegungen/Berechungen zur Zeit-Symmetrie bzw. zur Zeit-Umkehr anstellen – dann widerspricht dies den Grundlagen der Thermodynamik.

      • Und schon stellt sich die Frage, ob es Grenzen der Gültigkeit der Grundlagen
        der Thermodynamik gibt, sind die schon ausgelotet?
        Genauso könnte man eine Verletzung des Äquivalenzprinzips gebrauchen,
        um die ART mit der Quantentheorie vereinigen zu können – unser Dilemma.
        Die aktuelle MICROSCOPE-Mission hat auf den ersten 1900 Satelliten-Umläufen
        das ÄP mit 10hoch-14 bestätigt, erreicht werden soll 10hoch-15.
        Das ist 100x besser als bisher und exzellente Bestätigung der Relativitätstheorie.

      • @KRichard

        Augustinus´s Buch ´Bekenntnisse´ gehört zu den wichtigen Grundlagen europäischer Kultur – bei Reclam gibt es eine Ausgabe für 10 Euro.
        Selbst wenn man annimmt, dass ein Physiker überhaupt keine Allgemeinbildung hat/braucht – dann muss er zumindest physikalisches Fachwissen haben

        Ich muss gestehen, dass mir bislang nur wenige Physiklehrbücher untergekommen sind, die mit diesen Worten beginnen:

        “Groß bist du, o Herr, und deines Lobes ist kein Ende; groß ist die Fülle deiner Kraft, und deine Weisheit ist unermeßlich. Und loben will dich der Mensch, ein so geringer Teil deiner Schöpfung; der Mensch, der sich unter der Last der Sterblichkeit beugt, dem Zeugnis seiner Sünde, einem Zeugnis, daß du den Hoffärtigen widerstehest; und doch will dich loben der Mensch, ein so geringer Teil deiner Schöpfung.”

        “Physikalisches Fachwissen” mit religiösem Unterwürfigkeitsgewäsch einzuleiten ist mal eine ganz neue Nummer. Bzw. in diesem Fall eine sehr alte Nummer.

        Aber vermutlich war Ihr Beitrag satirisch gemeint und ich habe das einfach nur nicht erkannt.

        • @Spritkopf: Bischof Augustinus beschäftigt sich in ´Bekenntnisse´, Buch 11, Kap. 13-29 mit dem Thema ZEIT. In seinem Text weist er klar darauf hin, dass Zeit keine Dauer (Ausdehnung) hat (und somit keine 4. Dimension sein kann) – und dass der Eindruck von Zeit-DAUER nur ein Ergebnis unserer subjektiven Wahrnehmung/Phantasie ist (Ausdehnung des Geistes bzw. der Seele).
          Dieses Buch ist eindeutig kein Physikbuch sondern eine theologische Abhandlung – welche als solche große Wirkung auf die europäische Kultur hatte und hat. Und speziell seine Ausführungen zählen zu unseren wichtigsten Quellen zum Thema ZEIT.

          Da bereits die buddhistische Philosophie seit 2500 Jahren davon ausgeht, dass es die Zeit-Dauer als 4. Dimension in der Realität nicht gibt – kann man dies gerne als Quelle nehmen; wenn man das Buch von Augustus ablehnt. Ich verweise deshalb immer zusätzlich auf das Buch ´Bekenntnisse´von Augustus, weil dieses Buch preiswert zugänglich ist (Reclam, 10 Euro) – und weil man zum Verständnis der buddhistischen Philospohie viel mehr spezielles Zusatzwissen brauchen würde.

          • @KRichard

            In seinem Text weist er klar darauf hin, dass Zeit keine Dauer (Ausdehnung) hat (und somit keine 4. Dimension sein kann)

            Als ob es irgendeine Relevanz für die Physik hätte, was der Herr Bischof da im religiösen Wahn vor sich hin brabbelt. Wenn er der Meinung ist (bzw. war), dass Zeiträume sich nicht bemessen ließen, weil vergangene Zeit ja keine Zeit mehr sei und zukünftige Zeit noch keine sei, auch wenn wir sehr wohl den Zeitraum vom 3. Januar 1999 bis zum 12. Dezember 2021 sowohl messen wie auch ausrechnen können, dann sind das die theologischen Klimmzüge eines religiösen Apologeten, aber ganz sicher nicht irgendetwas, was wir in der Physik berücksichtigen müssten.

            Dieses Buch ist eindeutig kein Physikbuch sondern eine theologische Abhandlung

            Da wenigstens haben Sie Recht. Und wer sich etwas tiefer mit theologischen Abhandlungen befasst hat, der weiß, dass da oftmals eine Menge Unsinn drinsteht.

            welche als solche große Wirkung auf die europäische Kultur hatte und hat.

            Dem könnte ich sogar zustimmen. Augustinus’ Ausführungen zur Erbsünde und zur Lehre von der Hölle haben großes Leid in der europäischen Geschichte verursacht und tun das dort heute noch, wo fundamentalistisch-religiöse Gruppierungen (etwa evangelikale Freikirchen) diese Konzepte nutzen, um ihre Kinder mit Angst und religiösen Strafdrohungen zu überziehen und sie damit möglichst duckmäuserisch zu halten.

            Und speziell seine Ausführungen zählen zu unseren wichtigsten Quellen zum Thema ZEIT.

            Nur in den Augen von Leuten, die religiösen Quatsch für wichtiger halten als beleg- und experimentbasierte Naturwissenschaft.

    • Wenn die mathematische Formelsammlung nichts anderes ist, als ein heiliges Buch, wie Bibel, Koran, die Veden usw., dann ist Wissenschaft schlicht überflüssig.

      Und heute sind wir da sehr nahe dran.

  54. Die Quellen der Naturwissenschaft sind primär die Beobachtungen der Natur. Offensichtlich sind die Beobachtungen vom Wahrnehmungsvermögen des Menschen abhängig und bestimmt. Daran können auch die Instrumente der Wissenschaft nichts ändern, die schließlich vom Menschen selber konstruiert sind.

    Beobachtungen sind nicht die alleinige Quelle. Woher wissen wir, dass die untergehende Sonne dieselbe ist wie die am nächsten Morgen aufgehende? Wir haben kognitive Fähigkeiten, um ähnliche oder sich wiederholende Beobachtungen als solche zu erkennen, dank unseres Gedächtnisses. Daraus folgt das logische Denken; eine Logik, die anscheinend der Natur zu Grunde liegt. Objekte der Wahrnehmung entstehen nicht aus dem Nichts und verschwinden nicht im Nichts, ausgenommen bei Täuschungen unserer Wahrnehmung (die Show von Magiern).

    Die Beobachtungen wie auch die Erfahrungen des Menschen sind trügerisch, weil wir die Welt aus der Gegenwart retrospektiv betrachten. Wir übersehen dabei, dass die Welt auch anders sein könnte als sie ist, unterstellen der Entwicklung daher gerne Zwecke und Ziele, besonders der Evolution des Lebens. Über den Anfang der Entwicklung fehlen uns die Informationen.

    In der erfahrbaren Logik des Naturgeschehens sind sowohl die Gesetze der Naturwissenschaft als auch die Bedeutung der Mathematik begründet, nicht nur zur Formulierung der Gesetze, sondern auch zur logischen Ableitung von Aussagen über die Natur, über die Grenzen der Beobachtung hinaus. Die mathematische Topologie könnte anhand ihrer eigenen Gesetzmäßigkeiten vielleicht erklären, ob die 3-Dimensionalität des Raumes zufällig oder zwingend ist. Die Frage ist, ob die Dimensionen eine nachträgliche Eigenschaft des Raumes sind, oder ob sie den Raum ursächlich aufspannen.

    Da der Mensch aus derselben Materie besteht wie das übrige Universum, kann er sich selbst und das Universum nicht von außen betrachten. Somit bleibt die Information immer unvollständig. Hier ist Raum für reine Hypothesen oder Theorien (Urknall-, Stringtheorie), für wissenschaftliche Philosophie (Erkenntnistheorie) und schließlich für metaphysische Spekulation.

    • Wie kann ein Lebewesen, das in der Raumzeit lebt, etwas Nicht-raumzeitliches erkennen, beobachten oder auch nur messen?

      1. Beobachten wir unter _ähnlichen_ Bedingungen, daß wir _ähnliche_ Ergebnisse erhalten?

      oder

      2. Beobachten wir unter _identischen_ Bedingungen, daß wir _identische_ Ergebnisse erhalten?

  55. AR,
    Also
    “kann er sich selbst und das Universum nicht von außen betrachten. ”

    Zum Glück haben wir Phantasie , Abstraktionsvermögen und nicht zuletzt den Glauben.
    Also, was nehmen wir war, und unter welchen Bedingungen.

    Wir nutzen bei unserer Wahrnehmung nur die elektromagnetische Strahlung, die Gravitation und unsere Trägheit. Stellen wir uns vor, wir könnten die Gravitation sehen, das magnetische Feld der Erde wahrnehmen, wie eine Brieftaube, und wir wären so klein wie eine Ameise. Das fehlende Hirnvolumen gleichen wir dadurch aus, dass wir wie Ameisen unser Denken synchronisieren könnten. Ameisen machen das über Geruchsstoffe, wir machen das dann über elektromagnetische Wellen , die unsere Fühler ausstrahlen und empfangen können.

    Dann hätten wir eine andere Art die Welt zu sehen. Unsere Philosophie ware auch anders. Und da Ameisen kaum Krankheiten haben und wir auch “gemeinschaftlich” denken, hätten wir keinerlei Mitleid, weil uns auch die Individualität fehlte.
    Würde sich so eine Ameise für 3d-Probleme begeistern können? Wohl kaum!
    Was ich damit sagen will, dieses provokante Thema ist nur ein Aufhänger ohne Bezug zu den menschlichen Problemen.
    Für Mathematiker sicher ein Thema, für das Universum sicher nicht.

    • Je länger ich mich mit Philosophen beschäftige, umso mehr komme ich zur Erkenntnis, daß sie nur Berufsschwätzer waren…ich habe nicht einmal den Eindruck, daß sie überhaupt an Weisheit interessiert waren.

      Oder mit Bertrand Russel
      Soweit Philosophen sich mit dem Problem der Erhaltung des Sozialgefüges befasst haben, strebten sie nach Lösungen, die weniger offenkundig von Dogmen abhingen als diejenigen, die ihrerseits die offiziellen Religionen anzubieten hatten. Die meisten Philosophien sind die Reaktion auf einen Skeptizismus gewesen; sie sind in Epochen entstanden, in denen die Autorität allein nicht mehr zur Erziehung des für die Gemeinschaft notwendigen Minimums an Glauben ausreichte, so dass zur Erreichung dieses Resultats neue Argumente gefunden werden mussten, die wenigstens ihrem äußeren Anschein nach vernunftgemäß waren.Dieser Beweggrund hat zu einer tiefen Unaufrichtigkeit geführt, mit der die meisten Philosophien sowohl des Altertums wie der Neuzeit behaftet sind. Eine oft nur unbewusste Furcht, dass klares Denken zur Anarchie führen könnte, hat oft die Philosophen bewogen, sich in die Nebelwolken trügerischer und dunkler Behauptungen zu hüllen.

      • Nun die Naturwissenschaften haben sich von der “schwätzenden” Philosophie
        rechtzeitig abgekoppelt, die Naturphilosopie bedient sich heute u.a. der Physik.

        • Schön wär’s (gewesen).
          Woher kommt denn der Begriff Naturgesetze?
          Die Naturwissenschaft bedient sich der Meta-Physik…nichts anderes ist es, Mathematik als Modell der Wirklichkeit auszugeben.

  56. FMA,
    die Grenzen der Philosophie oder über den Sinn von Unwichtigem zu sprechen,
    das ist auch meine Meinung, aber die Krux an der Sache ist, dass man sich erst mit Philosophie beschäftigt haben muss, um nachträglich über ihren Sinn reflektieren zu können.
    Für mich war nach Wittgenstein die Grenze des erträglichen überschritten und ich habe mich von der Philosophie abgewendet.
    Frauen sind da viel klüger, die wenden sich ihr erst gar nicht zu.

    • Ich habe mich damit beschäftigt. Als Logiker eben auch aus dieser Perspektive. Die Grundlagen, die Grundannahmen heutiger Wissenschaft, fußen auf Meta-Physik. Viele Philosophen waren nicht auf der Suche nach Weisheit, sie suchten nur die Bestätigung ihres Glaubens. Aber genau das ist für mich keine Wissenschaft und sie auch keine Wissenschaftler.

      Nach Wittgenstein begann der Konstruktivismus und in Folge die Post-Strukturalisten, Dekonstruktivisten, Postmoderne usw.
      Allen ist gemein, daß sie Sprache mit Wirklichkeit verwechseln. Ich würde gar soweit gehen, daß ich es umkehre und sage, daß es die Konstrukte sind, die nicht existieren können.
      Wir denken nicht in Sprache. Denken erlebe ich, danach kann ich erst darüber sprechen und nicht umgekehrt.

      Unsere Vorstellungen bestimmen, wie wir Sprache wahrnehmen und nicht umgekehrt.

  57. Die Evolution hat uns Menschen die Fähigkeit gegeben, über die rein physikalischen Gegebenheiten hinaus Signale aus der Umwelt zu empfangen, zu verarbeiten und als Informationen oder Wissen zu deuten und zu speichern. Die Bedingungen des Lebens selber bestimmen die Art und Weise, wie das Wissen über die Welt und über uns selbst beschaffen sein muss. Die Vielfalt des Lebens zeigt, dass es sehr verschiedenartig sein kann. Aus all dem müssen wir ein möglichst objektives Wissen destillieren, dank unserer kognitiven, “angeborenen” und darüber hinaus erworbenen Fähigkeiten.

    Eine elementare Fähigkeit des Menschen ist der Vergleich von Mustern als Daten oder Signale. Damit können Ähnlichkeiten oder Identitäten in Beobachtungsdaten erkannt werden. Das heißt selbstverständlich, dass solche Muster bereits im Gedächtnis gespeichert sind! Biologisch geschieht der Vergleich durch Transmitter und passende Rezeptoren, durch Enzyme und passendes Substrat, allgemein durch die hohe Spezifität komplexer Moleküle. Das ist das Schlüssel-Schloss-Prinzip. Duch die Integration vieler Einzelbeobachtungen lassen sich Gesetzmäßigkeiten der Natur erkennen und Wissen erwerben, z.B. Tag- und Nachtrhythmen oder die Jahreszeiten.

    Schließlich übertragen wir die Muster der Natur in abstrakte Muster der Sprache. Der Mensch hat die Fähigkeit, sehr differenzierte Laute zu erzeugen und damit komplexe Lautmuster als Sprache hervorzubringen, die scheinbar unabhängig ist von den raumzeitlichen Erfahrungen. Die Sprache ermöglicht es uns, die Welt abstrakt zu repräsentieren und zwischenmenschlich zu vermitteln, eben als Wissenschaft.

    • Noch ein mal,

      wie kann irgendein Mensch etwas identisches, das ja bleibt, wie es ist und damit nicht entstanden sein kann, also nicht existent ist, erkennen, sich vorstellen, beobachten oder messen?
      Es gibt kein Wissen darüber.

  58. Wenn ein Mensch in dieser Welt keine eindimensionalen Objekte (sich) vorstellen, erkennen, beobachten oder messen kann,
    wenn ein Mensch in dieser Welt auch keine zweidimensionalen Objekte (sich) vorstellen, erkennen, beobachten oder messen kann,
    wie ist es dann mit drei oder vier oder mehr Dimensionen?

    Idealisten sind die wirklichen Höhlenmenschen, die neben Platon sitzend, auf Wände starren, völlig gebannt die schwarz-weißen Schattenspiele ansehend, gefesselt von ihrer Gewißheit. Ignoranz ist die einzig wahre moralische Stärke eines jeden Ideologen.

  59. @FMA
    Identität im Zusammenhang mit Ähnlichkeit ist hier nicht als Identität im strengen oder logischen Sinn zu sehen, sondern als Gleichheit. Zwei “identisch” aussehende Billardkugeln sind somit nicht identisch in dem Sinne, dass sie dieselbe Billardkugel wären. Im gleichen Sinn sind Wahrnehmungen nie identisch.

    Damit ist jede Form von Realismus widerlegt. Zudem ist die Welt nicht statisch, sondern verändert sich in jedem Augenblick. Zustände haben nur lokale Gültigkeit. Unser Bild von der Welt ist unsere eigene Konstruktion. Das bedeutet keineswegs, dass dieses Bild beliebig oder rein subjektiv wäre, sondern es ist die Folge der Erfahrungen, der sprachlichen Kommunikation und der Wissenschaft.

    Wer sich ernsthaft mit Philosophie beschäftigt, erkennt die wahre Bedeutung der Philosophie. Historische Auswüchse und Irrtümer sind kein Maßstab! Die moderne Philosophie ist eng verzahnt mit Naturwissenschaften. Umgekehrt offenbart schon der Gebrauch von Begriffen in den Naturwissenschaften (sprach)philosophische Probleme.

    • Es gibt _keinen_ Zusammenhang zwischen Identität und Ähnlichkeit. Das eine schließt das andere in Gänze aus. Falsifikation.
      Es ist die Dogmatik des Idealismus, die notwendige Ignoranz, die alles umbringt. Da ist keine Verbindung zwischen Diesseits und Jenseits.

      Die wahre Bedeutung der Philosophie ist die Tatsache, daß die meisten Philosophen lediglich ihren sinnfreien Glauben bestätigen woll(t)en:

      Wenn dies eine Welt der Regeln, der Naturgesetze ist, dann ist bereits alles geregelt, denn gegen wirklich existente Naturgesetze kann kein Mensch jemals verstoßen.
      Menschen, die Regeln in einer solchen Welt erschaffen wollen, üben eine sinnfreie Tätigkeit aus.

      Ist die Welt jedoch wirklich unendlich, ist in dieser Welt nichts tatsächlich absolut bestimmbar. Auch hier ist die Schaffung von Naturgesetzen sinnfrei, denn sie existieren _nicht_.

      Die Welt ist _keine_ Sprach Konstruktion:
      Die Vorstellungen die ein Mensch von der Welt hat, bestimmen, wie er Sprache wahrnimmt und nicht umgekehrt. Die Welt, das Universum, die Wirklichkleit, besteht nicht aus Sprache. Sprache ändert nichts an tatsächlichen Vorgängen, sie erschafft lediglich Illusionen, in Glaubenssätzen.

      Genau darüber läuft alle Manipulation, mit der Behauptung, es gäbe Identität, Unveränderliches. So machen sich die Naturwissenschaften zum Helfershelfer aller Ideologien.

      Sie, die (Natur)Wissenschaft, ist dann keine Aufklärung, sie ist dann nur eine weitere sinnfreie Religion, eine Glaubenschaft und Wissenschaftler sind nur weitere Pfaffen.

  60. FMA,
    die Sprache erschafft die Umwelt , zumindest erkärt sie die Zusammenhänge, differenziert Ähnliches und schafft Assoziationen, wenn der Mensch assoziativ denken kann. Frauen sind da besser als Männer.
    Aber auch bei analytischen Denken, dringen wir in die Wirklichkeit erst ein, wenn wir die Begriffe dazu haben.
    Der berühmte Aha – Effekt, kommt daher, dass wir noch unbekanntes auf bekanntes zurückführen. Und das geschieht meist über die Sprache. Taubstumme, die auch noch blind sind, können über Sprache die Welt begreifen.
    Der Volksmund nennt das 1 + 1 zusammenzählen können.

    • Solange Du daran glaubst, solange ist es für _Dich_ geschaffene Realität. Es ist allerdings keine Wirklicheit.

      Die Wissenschaft ist längst zum Ritualismus verkommen:

      Die Vorfahren oder indigene Völker haben nichts mit dem ganzen Sprachschwachsinn am Hut. Wie konnten sie nur überleben?

      Indem sie nur in sprachlichen Konstrukten leb(t)en?

      Du bist zutiefst gläubig…Universalienstreit…
      Abstraktes Denken, das Erleben, ist die Voraussetzung für Sprache, nicht umgekehrt.
      Teile existieren auch dann nicht, weil Du den Begriff Teilchen verwendest, Du kannst nur so tun als ob. Es ist nur ein Spiel, aber es wird für wirklich gehalten.

      Ersetze den Begriff ‘Teilchen’ mit dem Begriff ‘Gott’, vielleicht verstehst Du es dann.

  61. FMA,
    also die Kernwaffen sind der Beweis dafür, dass es die Teilchen gibt. Wir können sogar Elemente erschaffen, die es vorher noch nicht gab. Das ist fast göttlich. Wir sind mit der Sprache und ihren Begriffen, der Realität schon sehr nah gekommen, zu nah.
    Gott ist eher das Gegenteil eines Teilchens. Er lässt sich begrifflich gar nicht fassen, weil er die Sprache selbst erfunden hat. Deswegen dürfen im Judentum oder bei den Moslems Gott nicht dargestellt werden. Für die Christen besteht dieses Verbot auch, nur halten die sich nicht daran, wie sovieles andere auch.

    • In Deinem Weltbild vielleicht…
      Die Behauptung, daß Teilchen wirklich existieren, ist die Behauptung, es existiert eine einzig wahre Wahrheit.
      Die Behauptung, daß Gott wirklich existiert, ist die Behauptung, es existiert eine einzig wahre Wahrheit.
      Die Behauptung es existiert eine einzig wahre Wahrheit bedingt ein höheres Wesen, das diese _unveränderliche_ Wahrheit geschaffen hat. Ob dies Natur oder Gott genannt wird, ist gleichgültig.

      Alles ist bereits vorherbestimmt. Nichts ist veränderbar. LaPlace hat Recht. Jegliche Veränderung ist nur eine Illusion. Mathematik ist eine Illusion, Logik ist eine Illusion, der Verstand ist eine Illusion.

      Ein Beweis?
      Falsifikation ist etwas anderes als Verifikation, letzteres, der Positivismus, gilt schon lange als widerlegt und daher überholt.

      Über das Unveränderliche kann es kein Wissen geben, Du kannst daran nur glauben.
      Wenn der Mensch die Welt verstehen und verändern kann, sind Kernwaffen ein klarer Hinweis, daß es keine einzig wahre Wahrheit und folglich auch _keine_Teilchen gibt.

      Wenn das Ganze aus Teilchen zusammengesetzt ist, stellt sich die Frage, was zwischen den Teilchen ist.
      Wenn das Ganze aus Teilchen zusammengesetzt ist, kann das Ganze _nicht_ das Gegenteil eines Teiles sein. Das Gegenteil eine Teiles ist das _NICHTS_.
      In einer Teilchenbetrachtung ist das _NICHTS_ der unsichtbare Freund, der niemals erwähnt wird, jedoch für die Erklärung erforderlich ist.

      Aus was ist der (Welt)Raum denn zusammengesetzt, wenn doch im Vakuum per Definition gar keine Teilchen sind?

      Nach meinem Kenntnisstand ist die Raumzeit nicht gequantelt.

  62. FMA,
    Die Welt besteht aus Atomkernen und leerem Raum. Der Raum ist wiedrum nicht leer, er ist gefüllt mit elektromagnetischen Felder, Gravitationsfelder , Neutrinos usw. Das ist wie in einem Ozean. Der ist gefüllt mit Wasser, im Wasser schwimmen Fische, Plankton, Einzeller.
    Und die Einzeller bestehen wieder aus Atomen und leerem Raum.
    Ich finde diese Vorstellung spannend und beruhigend zugleich.

    • Etwas, das leer ist, ist aber nicht leer, aha…

      Was ist leerer Raum?

      Der Weltraum ist kein Gefäß indem sich etwas abspielt. Dein Bild vom Ozean ist irreführend und stimmt sowenig wie das Atommodell mit dem Vorbild des Sonnensystems. Diese Bilder taugen nicht. Du vermischt Wirklichkeit mit ‘Modellen der Wirklichkeit’…Jeder Pfaffe hat hier bessere Erklärungen.

      Du hast ein typisches wirklichkeitsfremdes mechanistisches Weltbild. Außer leeren platonischen Ideen ist da auch nicht viel. Ich finde Deine Vorstellungen erschreckend, erklärt jedoch die Entrücktheit (oder wars Verrücktheit?) der Welt. Genau deshalb zerstören Gläubige mit ihrer totalitären Ignoranz ihren Lebensraum, nur dann sind sie endgültig beruhigt, da alles _endlich_ & tot.

      Der leere (Welt)Raum besteht aus Feldern oder vielmehr aus Esoterik? Wie ist dieser Flickenteppich Raum denn zusammengesetzt? Mit Zeit?

  63. Könnte es sein, dass Leere und “Dinge” leicht zu sehr als Gegensätze gedacht werden? Vielleicht verhalten sie sich ja wie eine Seite der Münze zur anderen? Zwei Aspekte, beides dasselbe?

    • Dao, Yin & Yang?
      Buddhismus, Karma?
      Welchen Wolf der Mensch füttert?
      Alles fließt, Panta Rhei, zwischen zwei unveränderlichen Polen?
      Komplementarität der Pole?

      Das Problem ist für mich die Polarisierung, die sich aus einer gegensätzlichen Betrachtung ergeben.
      Ich gehe davon aus, daß es diese beiden _unveränderlichen_ Polaritäten nicht geben kann. Polarität ist allerdings die Voraussetzung für jegliche Moral, sprich um Gut und Böse bestimmen zu können. Die alten griechischen Philosophen hatten ja genau diese Obsession zur Tugendhaftigkeit, sie glaubten, Tugend wäre Weisheit. Gibt es eine Moral, die nicht verlogen ist?

      Das Problem ist grundsätzlicher Art, denn es gibt zwei mögliche Voraussetzungen für die Sichtweisen in Stichworten:

      1. Das Einzelne existiert und das Ganze ist aus diesen Teilen zusammengesetzt. Menschen tun so, als ob das Ganze mehr wäre als das zusammengesetzte Einzelne, Emergenz. Das ist eine Welt der Gegensätze, der Polaritäten. Ordnung und Chaos beherrschen die Welt. Es ist eine sich offenbarende Welt. Eine Welt des ‘Etwas’ und des ‘Nichts’. Alles ist bereits vorherbestimmt, auch wenn dies Einzelne nicht können. Willkommen in der Welt der Moral. Das Zuhause der Ismen wie Determinismus, Fatalismus und Nihilismus. Es kann keine zwei einzig wahre Wahrheiten geben. Hier liegt die Büchse der Pandorra, Hoffnung auf einen Ausweg aus der Ausweglosigkeit, die Münze mit den zwei Seiten ebenso begraben, wie die gefütterten Wölfe im Fluß.
      Induktion, reduzieren, weglassen ist die Kunst des Folgerns, Deduktion ist Illusion.
      (Die ganzen sich ergebenden Widersprüche lasse ich ganz in reduktionistischer Tradition einfach mal weg, wie bspw. in einer geregelten Welt überhaupt Chaos herrscht)
      Erzählung: Masse krümmt den Raum.

      2. Das unteilbare Ganze existiert und das Einzelne ist ein Phänomen des sich stetig ändernden untrennbaren Ganzen. Menschen tun so, all ob sie etwas Einzelnes bestimmen könnten. Das existente Ganze hat kein Gegenteil, denn es gibt kein außerhalb/innerhalb der Unendlichkeit-Ewigkeit. Es ist eine unbestimmte Welt. Es kann nur bestimmt werden, was nicht sein kann (alles unter 1.).
      Deduktion, abstrahieren, verallgemeinern ist die Kunst des Folgerns, Induktion ist Illusion.
      Einsteins Betrachtung wird einfach umgekehrt.
      Existenz, die Raumzeit selbst erzeugt alle Energien und Massen.

      Zum sprachlichen Unterschied Abstraktion und Reduktion: Abstraktion löst ins Ganze auf, durch Verallgemeinerung, Reduktion löst ins Nichts (Einzelne) auf, durch Weglassen. Zwei unterschiedliche Vorgänge.

      Abhängig von einer dieser Grundannahmen nimmt der Mensch Sprache wahr.

  64. Hin zusammen,

    „Nun die Naturwissenschaften haben sich von der “schwätzenden” Philosophie
    rechtzeitig abgekoppelt, die Naturphilosopie bedient sich heute u.a. der Physik“

    Ihre hier zum Ausdruck kommende Geringschätzung der Philosophie ist vielen Physikern zu eigen, in unterschiedlichem Grade. Diese Geringschätzung speist sich aus dem Glauben, der Weg zur Erkenntnis der äußeren Welt würde nur über die Physik führen, was ein Irrglaube ist. Sie speist sich auch aus dem Irrglauben, die Physik wäre im Wissens-Wettbewerb mit anderen Wissenschaften, und hätte diese ja schon übertrumpft. Und sie speist sich aus Ignoranz über das, was die Philosophie macht und was ihre Aufgabe ist.

    Physiker suchen in der Regel das Gespräch mit Philosophen nicht. Das ist mir schon klar.

    Aus der Sicht der Philosophie ist die Physik ein pragmatisches Ausprobieren nach dem Motto: mach das Beste draus und rede viel darüber. Die Physik besteht z.B. auf empirischer Verifikation von Behauptungen über die objektive Welt, obwohl sie sich ausschließlich nicht verifizierbarer Grundlagen bedient, und häufig unbeweisbare Voraussetzungen in ihre Theorien aufnimmt. Ihre Ontologie ist unbestimmt (Quantenfeldtheorie), viele ihrer Begriffe inkonsistent (Felder als physikalische Entitäten, Punktteilchen), sie weiß nicht, auf welche Wirklichkeit denn ihre synthetisch entstandenen Theorien verweisen, die keine konsistente Deutung zulassen, dabei bleiben viele Grundbegriffe wie Energie, Masse, Wirkung, dynamischer Raum, Quantisierung, virtuelle Teilchen und vieles mehr in ihrer metaphysischen Bedeutung unklar. Die Physik muß zahlreiche Widersprüche in ihrem Begriffsapparat und ihren Denkansätzen ignorieren.

    Das gelingt ihr gut. Denn letztlich kommt es ihr mehr darauf an, Ingenieurwissen zur Verfügung zu stellen – womit sie sehr erfolgreich ist -, als all die Fragen, die sich stellen, zu beantworten, und all die Probleme zu lösen, die sie hat.

    Das ist die Sicht der Philosophie, natürlich eine spezielle Sicht, aber eine, die gut begründet werden kann. Dabei ist es weder Aufgabe noch Absicht der Philosophie, mit der Physik in irgendwie zu konkurrieren oder sie schlecht zu machen. Es ist Aufgabe der Philosophie, der Physik zuzuarbeiten. Die Metaphysik z.B. liefert einen Ordnungsrahmen, eben die Leitlinien, nach denen kohärente Weltmodelle entwickelt werden können, aber das sind Leitlinien ohne Wahrheitsanspruch oder Endgültigkeit. Die Metaphysik will sich nicht mit dem Titel der Wahrheit adeln, sondern Orientierung geben, auf der Basis von Bewährung, mehr nicht.

    Wer das alles nicht versteht, kann – aus meiner Sicht – keine guter Wissenschaftler sein, auch wenn er erfolgreich ist. Der Erfolg allein ist eben – wie bei Allem im Leben – kein Garant für das Gute.

    Grüße Fossilium

    • Nuja, ganz so pessimistisch wäre ich nicht. Zumindest die ganz Großen ihrer Zunft, die der neuen Physik die Bahn gebrochen haben – Einstein, Heisenberg, Schrödinger, Bohr u.a. – , hatten durchaus intensive Schnittstellen zu Philosophie und Religion, via Schopenhauer auch zum Buddhismus.

      Für Schrödingers Katze müsste das noch geklärt werden. Schwer zu fassen, das Tier.

      • Meine Nachbarin hat eine kleine Klappe in die Tür eingebaut. Weil das Tier freiwillig kommen möchte. Es ist also real. Die Physik sollte einfach genau so verfahren und abwarten, ob wirklich wer reinkommt.
        Na ja , sie experimentieren ja schon länger. Aber sie streiten sich halt jetzt, ob wirklich wer gekommen ist, oder ob jeweils der andere sich das nur eingebildet hat. Aber die haben es ja auch schwerer als Ah, nonymus. Denn wenn sie (offiziell) gewisse Rahmen überschreiten ,fallen sie eventuell ins existentiell noch weit Ungewissere als es das Tierchen schon immer war. (-;

        • Ha! Die Nachbarin hat ganz offensichtlich ein völlig normal existierendes Hauskazz. Und die Kazznklapp: woher, HERR L.Schaber, wissen Sie davon?

          (“Frau Nachbarin, Euer Fläschchen!”)

          • Zur Erfoeschung meiner näheren Nachbarschaft bediene ich mich grundsätzlich der empirischen Methode und bevorzuge die unmittelbare Beobachtung der zu messenden Objekte. Aber nicht dass jemand jetzt glaubt, dass mir das auch noch Spaß macht. Es ist, weil harte Wissenschaft, eigentlich ein Knochenjob. Besonders da das Objekt noch relativ jung und wendig ist.

          • Die Kazz oder die Nachbarin?

            À propos. Zurzeit geistern ja Nachrichten herum, der Urknall habe ein für allemmal ausgeknallt. Frage an die Wissenden: ist das so? Und was passiert dann mit unseren Knoten?

        • Aber, aber. Ich bin doch kein Äh- Nonymus. Und deswegen sind wir zwar vielleicht in Gomorrha, aber keinesfalls in Sodom!

          • Große Verwirrung allenthalben, weil ein Teil der Theoretiker den anderen Teil (der “Standard-System-sprengend hinter den Urknall “schauen” möchte) für halbesoterische Metaphysiker hält und glaubt, solches habe nichts mehr mit Physik zu tun und gehöre deshalb ins Aufgabengebiet der Theologen.

          • Dann halten wir’s doch lieber mit dem Cherubinischen Wandersmann und seinem – geringfügig modifizierten – Diktum: “Der Knall ist ohn warumb / er knallet weil er knallet / Er achtt nicht seiner selbst / fragt nicht ob man ihn …” Und da haben wir den Schlamassel. Reim kaputt.

          • Die Behauptung des Urknalls _ist_ Metaphysik…metaphysischer wirds dahinter nicht mehr…

            Das _Nichts_ ist das (behauptete) Gegenteil des Seins. Im _Nichts_ gibt es kein Sein.

  65. Es geht hier nicht um Polaritäten, so kommen wir nicht weiter. In dem Moment, wo sie benannt sind, lösen sie sich auf. Paff.

    Es geht erst recht nicht um Moral. Moral ist nichts Gesetztes, Moral ist emergent.

    Der Passus, auf den ich mich stillschweigend bezogen habe, stammt aus dem Herz-Sutra. Dessen Essenz wiederum: “Form ist Leere, und Leere ist nichts als Form.” Eines ist zugleich jeweils das andere, jetzt, in diesem Augenblick. Das ist zugegebenermaßen eine Zumutung für den Geist und wohl kaum als intellektuelles Konzept aufzulösen.

    Für einen der bedeutendsten Kognitionswissenschaftler unserer Zeit, den leider früh verstorbenen Francisco Varela, war dies jedenfalls ein entscheidender Bezugspunkt. Ich hatte das große Glück, noch kurz vor seinem Tod mich darüber mit ihm austauschen zu dürfen.

    • Was ist hier Leere? Was ist hier Form?
      Der Behälter ist gleich dem Inhalt?
      Der Behälter ist der Inhalt?
      Reinkarnation?

      Ich kann tatsächlich nicht viel anfangen mit verschleiernder Sprache, die ein Guru, ein Lehrer, benötigt.
      Das erinnert mich mehr an Nostradamus:
      Lies raus was Du willst, der Meister, der Guru, der Lehrer weiß mehr, sonst wäre er kein Lehrer und du nicht ein dummer Schüler.

      Das Leib-Seele-Problem?
      Die Welt besteht weder aus Materie, noch aus Geist.

      Aus was sie wirklich besteht?
      Keine Ahnung, ich kann nur sagen, aus was sie nicht bestehen kann:
      Aus Unveränderlichem.

      Die Welt ist nicht Komplex, sie verändert sich nur stetig, was wie ein Konstrukt aussieht ist keines, das Konstrukt ist gar das, was nicht existiert. Die Welt kann nicht ‘an sich’ erkannt werden. An dieser Stelle versagt Humberto Maturanas/Francisco Varelas Autopoiesis an der Sicht auf sich selbst, denn die Welt kann kein Regelsystem sein. Die Systemtheorie ist ein mechanistisches Prinzip.

      Mir wurde auch beigebracht, daß Norbert Wiener den Begriff Kybernetik geprägt hätte. Eines Tages bin ich auf die ältere Version des Steuermannes gestoßen: Theologische Kybernetik.

      Das war ein Aha-Effekt.

      So stammt eben auch der Begriff Naturgesetze aus genau dieser Ecke.

      Bedaure, ich bin nicht gläubig, sondern denke, daß kein höheres Wesen zur Erklärung der Welt erforderlich ist.

      Die Welt ist einfach stetig selbstähnlich veränderlich, das ist für Regelianer wie Büro- und Technokraten erfahrungsgemäß nicht leicht zu verstehen.

      In einer systemischen Welt gibt es weder Offenheit, noch Freiheit. Ein völlig sinnloses Leben in Ohmacht:

      Jedes Ideal _ist_ eine Polarität. Damit die Polarität sich auflösen könnte, müßte sie zunächst entstanden sein, was nicht der Fall sein kann, da das Ideal bleibt wie es ist.

      • Für mich(!) ist es zunächst mal nicht so erheblich, ob “die Welt an sich” erkannt werden kann. Denn gäbe es etwas, das so beschaffen wäre, dass es auf mich niemals eine Wirkung auszuüben in der Lage wäre, dann wäre es (eventuell nicht nur für mich) im wahrsten Sinn des Wortes Un- wirklich
        Und inwieweit es sich lohnt, sich mit Unwirklichem abzugeben, muss jeder selbst (für sich) entscheiden. Ich jedenfalls mag metaphysische Science- fiction. (-:

        • @L.Schaber
          Oh, in der von mir vertretenen Sichtweise ist Metaphysik sehr wichtig, denn sie dient zur Falsifikation, was nicht sein kann.
          Ich lehne nicht den Glauben ab, das könnte ich auch gar nicht, nur die Glaubensinhalte.

          Ich mag auch die Sprache Mathematik, um in Kurzform etwas ausdrücken zu können. Wenn sprachliche Ideale jedoch verwirklicht werden sollen, dann reicht ein Geschichtsbuch, um zu begreifen, wo dies bisher endete.

          Das Dumme dabei ist, daß Menschen sich auf ihren Glauben an Ideale, auf Regelsysteme verlassen und ihrem Denken, ihrem Wissen selbst mißtrauen.

      • Es ist alles viel einfacher.

        Behälter/Inhalt, Reinkarnation, Guru, Nostradamus, Leib/Seele: brauchen wir alles nicht, vergessen wir das.

        Form ist Leere, und Leere ist nichts als Form. Zugegebenermaßen ist es schwierig, die mit einem breiten Spektrum von Bedeutung geladenen Sanskritbegriffe zu übersetzen. Form: die Chinesen bezeichnen das als “Welt der zehntausend Dinge”. Okay, es sind ein paar mehr: es meint die Welt in ihrer Mannigfaltigkeit, alles, was existiert, alles, was Gestalt hat, alles, was es gibt. Leere (Sanskr.: shûnyatâ): Nichts. Nada. Einfaltigkeit. So, und dies beides soll nun ein und dasselbe sein? Was bedeutet es aber, wenn etwas da ist, besteht, und zugleich nichtig ist und leer? Die Chinesen mit ihrem wunderbar plastischen Denken und Sprechen fügen noch hinzu: zwischen beides passt kein Blatt Papier (und wer mal Maulbeerbaumpapier in Händen gehalten hat, der weiß ganz sinnlich: da passt wirklich nix, aber auch garnix dazwischen).

        Es bedeutet: die mannigfachen Dinge der Welt sind inhärent leer. Was meint: sie sind da, zweifellos, aber sie bestehen nicht aus sich selbst heraus. Haben keine (aristotelische) Substanz. Und was machen wir jetzt? Nuja, ganz klar: wenn sie nicht Bestand aus sich selbst heraus haben, dann eben einzig aus dem Bezug zu- und untereinander. Aus ihrer vielstrahligen wechselweisen Abhängigkeit. Man stelle sich vor: alle bestehenden Dinge als kleine Perlen, die in einem gigantischen drei(!)dimensionalen Gitternetz untereinander verbunden sind und einander wechselweise spiegeln. (Das Bild ist zu schön um von mir zu sein, dieses Konzept ist in der buddhistischen Philosophie als ‘Indras Netz’ bekannt.)

        Zu meiner Erwähnung von Francisco Varela: habe mich nicht umsonst auf ihn allein und nicht auf seine Kooperation mit Maturana bezogen (Lit. z.B.: F. Varela, E. Thompson, E. Rosch: Der mittlere Weg der Erkenntnis. Die Beziehung von Ich und Welt in der Kognitionswissenschaft – Brückenschlag zwischen wissenschaftlicher Theorie und menschlicher Erfahrung.)

        • Danke zunächst für den Tip auf Varela, werde ich mir besorgen. Wann ich zum Lesen komme, weiß ich noch nicht.
          Hier einer der Mitautoren
          https://ethik-heute.org/achtsamkeit-ist-kein-inneres-teleskop/

          Vieles dabei, dem ich zustimme, doch auch einige Widersprüche.

          Aber zur Leere
          Form ist gleich Leere und Leere ist gleich Form. Ist das die Nicht-Zweiheit?

          Mir ist klar, daß Übersetzungen immer schwierig sind, zumal wir uns in einer anderen Lebenswirklichkeit bewegen.
          Leere verbinde ich hier auf dem Planeten mit einem Behältnis. Mir ist klar, wenn einer sagt, ein Glas, eine Flasche, ein Zimmer, ein Lagerraum sei leer, ebenso wie eine leere Hülle.
          Was mich schon lange stört, ist die Verwendung für das Weltall, der leere Weltraum. Das ist zutiefst unlogisch, denn den Weltraum kann ich wahrnehmen, da ist nichts leer und schon gar nicht _Nichts_. Ich meine hier nicht das Licht, sondern die Schwärze. Einfach gesagt: Da ist Schwärze angereichert mit Lichtpunkten. Wie ist diese Schwärze entstanden, die ja da ist? Welche Wirkung hat der Weltraum, wenn er da ist, ändert er sich auch und muß zwingend eine Wirkung auf ‘Teile’ haben, nicht nur umgekehrt.

          Das ist einer der Gründe, weshalb ich die Gleichsetzung von Form und Leere ablehne. Eine weitere ist, daß es keine Form geben kann, die sich nicht stetig ändert. Der Begriff Zeitpunkt ist hier irreführend, ein Widerspruch in sich.

          Da ist kein gangbarer Mittelweg, indem ich einfach alles zusammenwerfe, Leib-Seele, Körper-Geist in einen Topf und verrühre.
          Wenn eine existente Grenze postuliert wird, hier zwischen Teilen, gleich welcher Form, dann erwarte ich eine Begründung, wie Grenzen entstanden sein können, nichts anderes als das Absolute, das Losgelöste ist eine einzig wahre Wahrheit. Eine existente Grenze kann keine Folge haben. Eine einzig wahre Wahrheit muß vollständig und endgültig, daher unveränderlich sein. Käme nur eine Information hinzu, wäre dies eine andere einzig wahre Wahrheit und es kann keine zwei einzig wahre Wahrheiten geben.

          Das was ich sehe, das Bild ist nur ein Symbol der Wirklichkeit, kein Abbild, da es kein Bild ‘an sich’ geben kann. Das ist in Sprache, insbesondere einer, die von der Teile-Erzählung geprägt ist, nicht leicht auszudrücken, zu verstehen ist es nur, wenn ein Perspektivwechsel zu einer Sicht des existenten Ganzen vorgenommen werden kann, Heute lebende Menschen sind so stark im ständigen Fokussieren gefangen, daß ihnen eine Distanz zum eigenen Ego kaum noch möglich ist und in Folge abstraktes Denken verkümmert. Mathematische Objekte sind nicht abstrakt, sie sind gar so konkret, unveränderlich, daß sie auch nicht vorstellbar sind. ‘Formale Logik’ ist tatsächlich dialektisch, _nicht_ logisch.

          Vor einem Jahr habe ich hierzu eine Mail an die Redaktion Scobel geschrieben, so ungefähr würde ich sie wieder schreiben
          https://pastebin.com/a3bBm9zA

          • Form ist Leere, und Leere ist nichts als Form.

            Nein, Nicht-Zweiheit ist damit wohl nicht gemeint. Weder Zweiheit noch Nicht-Zweiheit. Das ist dieser weired stuff, an sich einfach, aber von einer Einfachheit, die einem erstmal (obacht, zurück zum Thema:) Knoten ins Hirn macht. Unser im Westen favorisiertes Denken, bei dem man irgendwann ganz gern irgendwas in der Hand hat, scheint im besagten Fall nicht zu funktionieren. Man muss sich da wohl einrichten im beständigen Entzug. Glaubt man was festgekloppt zu haben – schon isses wech.

            Vor allem sind Form und Leere nicht auf Raum gemünzt. Sie zeigen eine Daseinsweise auf, im Kontext. Wie ich schrub: Form (die zahllosen konkreten seienden Dinge): da, aber nicht aus sich selber heraus (nochmal Aristoteles, zum Kontrast: sie verkörpern keine Substand, und auch der Satz vom zureichenden Grund – giltet net). Dieses “Nicht aus sich selbst heraus” bringt pünktlich die Leere ins Spiel, die es gewährt: die Dinge sind inhärent leer.

            Ach, es ist ein Kreuz mit dem Zeuchs …

          • @Ah, nonymous
            Weltraum war nur ein Beispiel meinerseits.
            In Deinem letzten Satz steckt genau das, was ich meine, daß bereits ein Ding, ein Behältnis da sein müßte, das überhaupt leer sein könnte. Die Zeit steht im Wege, nicht nur Raum.
            Ich hatte einige Diskussionen mit Buddhisten, die verfingen sich auch in Widersprüche.

            Ich weiß natürlich nicht, inwieweit Siddharta Gautama überhaupt korrekt übermittelt wird, das ist ja in vielen Religionen so, daß der Ursprungsgedanke nach und nach durch Ritualisierung und Dogmen verfälscht w(i)urde.

          • @Ah, nonymous
            Tja,
            das ist dann wiederum nichts anderes als das Spiel mit These-Antithese-Synthese, das dialektische Verfahren, das immerzu hierarchische Gesellschaftstrukturen rechtfertigt. Bereits beim Begriff Dharma wird dies deutlich.
            Es ist das, was Emotionen, Schuld bei auserkorenen Untertanen, gegen das höhere Wesen, gleich welchen Namens, und seiner Stellvertreter(!) auf Erden, erzeugen soll, nichts ändern zu können. Ohnmacht. Rituale bieten einen Ausweg?
            Nein. Subversive psychologische Techniken.

            Das ist genau das, was auch im asiatischen Raum tatsächlich _herrscht_, nicht erst seit den Kolonisationszeiten.

            Nur wenige Buddhisten versuchten auf aufgezeigte Widersprüche zu lachen, es ist ihnen spätestens nach diesen Aussagen vergangen, denn sie negieren auch nur ihre Verantwortung für das Ganze…und so _spielen_ Menschen weiter mit Regeln Unterwerfung unter das Spiel des Lebens, ohne wirklich je sich selbst sein zu können. Tatsächlich lachen sie nur über die eigene Unfähigkeit, Illusion von Wirklichkeit trennen zu können, ja nicht einmal zu wollen. Und wenn sie nicht gestorben sind, leben sie umso leichter weiter in ihrer erschaffenen unwirklichen Realität. Deshalb ist auch asiatische (Schein)Philosophie keine Lösung. Das _Nichts_ kann nicht das Dasein verneinen, um selbst da zu sein. Das _Nichts_ kann nicht da sein.
            Was geschieht mit dem Menschen, der versucht, das nicht Begreifbare, nicht vorstellbare, nicht denkbare _Nichts_ zu begreifen?

            In einer Regelwelt gibt es keine Offenheit und keine Freiheit, beides ist die Voraussetzung um überhaupt lernen zu können. Indoktrination ist etwas anderes als Lernen, es ist hier unter dem Begriff Nürnberger Trichter bekannt.

            Induktion-Abduktion-Deduktion
            These-Antithese-Synthese
            Täter-Opfer-Retter
            Dramadreieck.

            Das ewige Spiel mit ‘Gut und Böse’, es verschwindet nicht durch ‘Form ist Leere, Leere ist Form’, es ist umgekehrt.
            Es existiert weder unveränderliche Form, noch eine unveränderliche Leere. Damit ist auch geklärt, daß es _keine_ Dimensionen geben kann…

          • Geschätzter FMA, ich bin weit davon entfernt, Lösungen anzubieten, Rituale zu offerieren, Widersprüche aufzulösen oder gar beurteilen zu wollen, warum Buddhisten lachen. Nichts davon. Sie haben völlig recht: Unveränderlichkeit gibt es nicht, kein Ort, nirgends. Selbst auf den Big Bang scheint kein Verlass mehr zu sein. Von Big Ben ganz zu schweigen: er schweigt https://www.zdf.de/nachrichten/heute-sendungen/videos/big-ben-dieletzten-glockenschlaege-100.html

            “Was geschieht mit dem Menschen, der versucht, das nicht Begreifbare, nicht vorstellbare, nicht denkbare _Nichts_ zu begreifen?” Der implodiert irgendwann, der Arme, oder überschreibt seine eventuell nicht vorhandene Seele dem inhärenten Beelzebub, um es doch noch zu was zu bringen, Weltherrscher oder so. Nein, im Ernst: das Begreifbare begreifen, ohne sich davon schalu machen zu lassen, wäre doch eine Alternative.

            Nur eines noch: “These-Antithese-Synthese”. Ist es nicht erstaunlich, dass Hegels Inspiration zu seinem nobel klappernden Dreischritt durch ein Missverständnis erfolgte? Er hatte nämlich bei Leibniz nachgeschaut, der, Überraschung, angeregt durch einen in China tätigen jesuitischen Freund schon im 17. Jahrhundert sich mit dem I Ging auseinandergesetzt und daraus sein binäres Zahlensystem entwickelt hat, Grundlage für unsere digitale Welt. Die muss man zwar genau so wenig mögen wie Hegels Weltgoischd. Aber es hat was.

        • Nicht unbedingt, FMA. Es müsste nicht etwas dasein, denn es ist ebenso gut nicht(s) da – der erste Halbsatz: Form ist Leere. Zeit steht nicht im Wege, da gibts doch diese fabelhafte Dôgen’sche Uji, SeinZeit, in deren Takt mein Wasserkocher , die Kazz nebenan und das ganze Universum Augenblick für Augenblick “rückt”. Nichts geht verloren, aber alles ist nur Jetzt da.

          Ich bin ja kein Buddhist, aber ich denke, wenn’s feine Buddhisten waren, mit denen Du diskutiert hast, haben Widersprüche sie nicht besonders beunruhigt. Ein japanischer Freund in solcher Situation würde in die Hände klatschen und in ein infernalisches Lachen ausbrechen. Naja, sein Dichtername lautet halt auch, übersetzt: “krummes Tal”.

          Buddhisten würden vermutlich auch das Konzept eines “Ursprungsgedankens” ziemlich fremd finden – vor Shakjamuni gab es ja schon massenhaft Buddhas, und nach ihm drohen noch ganze Kalpas davon. Maitreya ist der nächste – aber das hat gute Weile – , immerhin sitzt der anständig auf einem Stuhl und nicht so schrecklich verknotet durch die Gegend. Verfälscht wird da so schnell nix, der Buddhismus hat eigentlich eine ganz kreative Karriere hingelegt, indem die indischen Konzepte, vor allem das Dhyana (Versenkung), sich in China mit dem Daoismus verbandelt hat (-> Chan), und das Chan wiederum in Japan mit dem dort geschätzten Shintoismus -> Zen. Alles gut : – ) Und guuds Nächtle.

  66. Philosophie ist eine wesentliche Grundlage von Wissenschaft. Wenn Herr Päs Überlegungen zur 3-dimensionalität des Universums oder zur Knotentheorie veröffentlicht – dann ist dies zunächst nur eine philosophische Fragestellung. Ein Akt von Neugier: ein Problem wurde erkannt und es wird ein Versuch zur Lösung unternommen.

    Zur philosophischen Lösung eines Problems gibt es mehrere Strategien: A) GOOGLEn -bereits vorhandenes Wissen reicht aus, um die Fragestellung zu lösen oder zu verwerfen. B) vorhandenes Wissen reicht nicht aus. Nun muss überlegt werden, ob es Möglichkeiten gibt, eine neue Idee zu prüfen – z.B. mittels physikalischer Experimente, Messungen. C) eine Fragestellung ist momentan unlösbar – dann wird das Problem zurückgestellt – bis andere, neue Erkenntnisse vorhanden sind, mit denen die Fragestellung bestätigt oder verworfen werden kann

    Neugier und die Lust nach neuen Erkenntnissen (= Philosophie) ist ein wichtiger Grund warum wir Menschen uns weiter entwickelt haben als manche Tiere.
    Ob eine neue Idee bestätigt wurde oder verworfen wurde, ist zweitrangig – wichtig ist die Bereitschaft, sich überhaupt mit neuen Ideen zu beschäftigen.

    • Philosophie oder die Liebe zum Denken ist Das Fundament der Wissenschaft, nach und nach sind Fachdisziplinen ausgelagert worden, bspw. die Mathematik, die tautologisch existiert, auf Axiomatiken beruht, und natürlich auch die Naturwissenschaft, letztlich spezifisch und sinnhaft, so darf zumindest gehofft werden.

      Die eigentliche Idee bestand darin, dass erfasste Datenlagen mit sich anschließender Theoretisierung als für den hier gemeinten Primaten, der gerne dem “Schamanentum” abgeschworen hat, nützlich erweisen, naturwissenschaftlich.
      Oder dass die Tautologie, die Mathematik ist als Fähigkeitslehre bekannt, ähnlich leistet, als Grundlage der Naturwissenschaften.
      ‘Mathematics is the gate and key of the sciences.’ war von Roger Bacon, die wissenschaftlichen Grundlagen sind bereits im alten Griechenland geschaffen worden.
      Neben der Naturwissenschaftlichkeit und der Logik, Mathematik ist logisch (“sprachlich”) , gibt es noch die Ethik, die “nicht ganz” unwichtig ist, und der sogenannten Moralphilosophie zu obliegen hat, wenn es nicht religiös werden soll, natürlich nur und nicht sonderlich politisch.

      Fragestellungen werden in etwa wie skizziert bearbeitet, in gewissem Sinne utilitaristisch, wobei der Schreiber dieser Zeilen nicht dem Utilitarismus das Wort reden will, es interessiert schon, was ist oder da zu sein scheint.

      Der eigentliche Gag beim Wissenschaftlichen ist das Sapere Aude.
      Das gar nicht so intuitiv und plausibel erscheint, auch heute vielen nicht, wie es ist.
      Es kann auch alternativ unvernünftig und sozusagen in der Horde gelebt werden, kollektivistisch, der Kollektivismus, auch der theozentrische, bleibt stark verbreitet.

      MFG
      Dr. Webbaer

      • Skepsis gegenüber dem “Unwirklichen” muss nicht unbedingt zum ethischen Utilitarismus führen. Vielleicht aber zu einem wissenschaftstheoretischen
        “(nearly) everything goes”.

        • @ L.Scha :
          Negativ, die naturwissenschaftliche Methode, ist bspw. in der bekannten Online-Enzyklopädie brauchbar beschrieben und womöglich liegt regionale Besonderheit vor, in Deutschland wird ja ernsthaft in Wissen und Wissenschaft gemacht, statt in Erkenntnis (“Scientia”), so dass es i.p. Erkenntnis nicht immer so klappt. wie sich dies manche vorstellen.
          Merkwürdigerweise scheint die Nicht-Annahme der szientifischen Methode, die durch einen Glaubensentscheid (!) zu erfolgen hat, in der BRD auch generell mit dem Relativismus Hand in Hand zu geben.
          Die szientifische Methode trägt hier keine Schuld.

          • Bekanntlich läuft in der “Humanistenszene” und in der “Skeptikerszene” (um es mal so unscharf zu nennen) schon seit längerer Zeit ein ziemlich heftiger Streit zu diesen Themen (“Naturalismus” gegen versch. Arten von “Realismus” usw.)
            Bei den Skeptikern gabs ja schon Abspaltungen deswegen.

  67. KRichard,
    was willst du lösen? Unsere eigene Existenz?
    Philosophieren ist ein Luxus, den sich nur der leisten kann, der eine Frau hat, die seine Hosen wäscht.

    FMA,
    Das Gegenteil von allem ist die Idee. Gott hat es immer gegeben und wird es immer geben. er braucht weder Raum noch Zeit. (Allerdings hat er die Menschen geschaffen, dass sie ihn denken)

    • Was ist ‘alles’?
      Das Gegenteil von Diesseits ist das Jenseits?
      Was weißt Du über die Existenz des Jenseits?

      Was ist Gott?
      Wenn Gott unveränderlich ist, kann er keine Wirkung haben…

      Philosophieren ist dem gläubigen Untertan ein Greuel, er müßte sich ja mit der Wirklichkeit beschäftigen, kann dann nicht mehr bequem auf Anweisungen der Bürokratie warten und sein Duckmäusertum ausleben…

    • Philosophieren ist ein Luxus, den sich nur der leisten kann, der eine Frau hat, die seine Hosen wäscht. (Zitatende)
      Der Schreiber dieser Zeilen ist wahrlich der tiefste Denker unsrer Zeit. Ich zum Beispiel habe auch nur aus diesem Grund(fast) alle Zeit der Welt Und das sogar in (fast) allen Dimensionen aller denkbaren Welten. Und vielleicht sogar in all denen, die man zwar nicht denken, dafür aber um so besser rechnen kann!
      Und das alles beweist eben, dass deDicke recht hat.
      Vielleicht fast genau so wie deAlte
      Ich verneige mich in Ehrfurcht.
      Amen.

  68. Was sind Dimensionen?
    A: Ausmessungen.

    Hier steckt auch der ‘Mens’ drinnen, womöglich der Mensch ebenfalls.

    Es obliegt dem erkennenden Subjekt, nicht zuletzt seinem Fortkommen geschuldet, diese Welt in Dimensionen aufzuteilen.
    Das Oben und Unten, das Rechts und Links, wie auch die Tiefe eines Gegenstands liegen insofern auf der Hand festzustellen, wie auch die Veränderung, die sogenannte Zeit.
    Die Zeit ist eine “weiche” Größe, weil sie nur die Veränderung meinen kann, die wiederum nur von erkennenden Subjekten festgestellt werden kann, die über ein Gedächtnis verfügen.
    Weitere Dimensionen sind denkbar, sie dürfen gerne gefunden und naturwissenschaftlich nachgewiesen werden.
    Der Theoretisierung (“Sichtenbildung”) sind hier keine Grenzen gesetzt; ob Steve Hawking hier mit seinen zehn oder elf oder zwölf Dimensionen recht bekommt, nun ja…

    MFG + schöne Woche noch,
    Dr. Webbaer

    • Eine Messung setzt Veränderung voraus, das steht im Widerspruch zu einer unveränderlichen Dimension.

      Es existiert kein nur vorwärts-rückwärts im Raum, genauso wenig wie ein vorwärts-rückwärts-links-rechts.

      Wo ist im (Welt)Raum vorwärts? rückwärts? links? rechts? oben? unten?
      In der Unendlichkeit?

      Veränderung=Raumzeit ist nicht nur der Wandel=Zeit, sondern auch die Bewegung=Raum.

  69. Zu folgendem Textabschnitt von weiter oben :

    “……….Bischof Augustinus beschäftigt sich in ´Bekenntnisse´, Buch 11, Kap. 13-29 mit dem Thema ZEIT. In seinem Text weist er klar darauf hin, dass Zeit keine Dauer (Ausdehnung) hat (und somit keine 4. Dimension sein kann) – und dass der Eindruck von Zeit-DAUER nur ein Ergebnis unserer subjektiven Wahrnehmung/Phantasie ist (Ausdehnung des Geistes bzw. der Seele).
    Dieses Buch ist eindeutig kein Physikbuch sondern eine theologische Abhandlung – welche als solche große Wirkung auf die europäische Kultur hatte und hat. Und speziell seine Ausführungen zählen zu unseren wichtigsten Quellen zum Thema ZEIT
    .”
    (Ende des Zitats)

    Der heilige A. hat all das aber vermutlich auch nicht von oben geoffenbart bekommen, sonderen solcherlei Überlegungen (bzw. metaphysische Spekulationen) gehen (ebenso wie weite Teile des NT ) zurück bis zu den Vorsokratikern.Welche selbst auch nicht traditionslos aus dem luftleren Raum heraus dachten. Manche denken da an Traditionslinien bis zurück zu den Sumerern und Ägyptern (der ersten Dynastien) .
    Bei den frühchristlichen Theologen kommen (über das NT oder zusätzlich) vermutlich wohl noch Einflüsse aus dem (ost-)asiatischen Bereich ( Buddhismus usw.) hinzu.
    An die Profis: Stimmts so in etwa?

    • Wo Augustinus seine Ideen her hat ist zweitrangig.
      Wenn er z.B. davon ausgeht, dass es Zeit gibt, weil die Welt/Universum+Zeit von Gott geschaffen wurde – ist dies zunächst nur eine theologisch/metaphysische Sichtweise. Allerdings wurde diese stillschweigend von der Physik übernommen – wo man auch davon ausgeht, dass es die Zeit seit dem Urknall gibt (vorher ist sie nicht definiert).

      Sie werden keine einzige wissenschaftliche Abhandlung finden, die erklärt wo denn die Zeit nach dem Urknall herkommt bzw. was man sich überhaupt darunter vorstellen kann (Was vergeht, wenn Zeit vergeht?). (Dass man die Ausbreitungsgeschwindigkeit des Universums zurückrechnet und so auf die Entstehung des Universums vor 13,8 Mrd. Jahren bei einem Urknall/Singularität kommt – beantwortet diese Frage nicht.)

      Und dies ist ein Grund, warum die Ideen von Augustinus zum Thema ´Zeit´ bis heute so bedeutend sind – die Physik hat keine besseren.

      • Raum und Zeit sind Erfindungen bzw. nichts als Ideen des Menschen. Es gibt sie nicht im Kosmos. Deshalb ist physikalisch eine Herkunft nie zu finden, außer mit der Herkunft des Menschen selber. Der Mensch definiert mehr oder weniger willkürlich oder vernünftig, was er als Raum und Zeit verstehen will. Die Natur liefert lediglich die Grundlagen dafür in Form von Bewegung oder Veränderung der wahrnehmbaren energetischen und materiellen Strukturen.
        http://www.spektrum.de/news/der-ursprung-von-raum-und-zeit/1207031

        • @Reutlinger: Grundlagen zor Thermodynamik wurden z.B. von Nicolas L.S. Carnot, Rudolf Clausius, Ludwig Boltzmann erforscht. Und eine Uhr ist nicht anderes als eine Maschine deren Funktion/Ergebnis (Zeitanzeige) nur auf ganz bestimmte Bedingungen geeicht ist. Verändert man diese Bedingungen, dann geht die Uhr (nach den Gesetzen der Thermodynamik) falsch.
          Aus dem Ablesen einer falschen Uhrzeit die Annahme abzuleiten ´Zeit ist relativ´- ist eine gewagte Schlussfolgerung

          Wissenschaftlich korrekt wäre es, eine geeichte Uhr jeweils neu zu eichen, wenn sich die Bedingungen ändern.

          • Womit und wie eicht man eine Ur-Uhr ?? oder eine “gute” Uhr?
            Eine Uhr zeigt die “Ortszeit” an, bringt man die Eichuhr dort hin – was zeigt die an?
            Und in der ART, also im beobachtbaren Universum, kriegt man keine abgestimmte
            Gruppe von Beobachtern hin. Es hängt vom “Weg des Datenaustausches” ab, ob
            zwei Beobachter sich einigen können (wollen) relativ zueinander in Ruhe zu sein
            oder daß ihre Uhren synchron gingen.
            Man kann kein Netz synchronisierter Uhren aufspannen, die “ortsunterschiedliche”
            Gravitation macht einen Strich durch die Rechnung – absolute Gleichzeitigkeit ade.
            Bleibt defacto nur der Rückgriff auf gute Modelle und Simulationen, die dann an
            Beobachtungen abgeglichen werden, um sie ggf zu verbessern – Standardmodell.

          • @Herr Senf
            Wird eine Uhr als Messgerät zum Messen von Uhrzeit verwendet – dann müssen für sie bestimmte Qualitätsanforderungen erfüllt sein; wie sie für Messgeräte üblich sind.
            Kalibrieren, Eichen, Feststellen von Messungenauigkeit, Definieren von Fehlerquellen – müssen nachvollziehbar beschrieben und geprüft werden. Dafür gibt es Qualitätsstandards und -regeln.

          • Wird ja auch gemacht, aber eben lokal, zB in der Schweiz 😉
            die bauen dort Atomuhren zB für die Satellitennavigation, mindestens 10hoch-13.

      • Zunächst mal wenig Widerspruch. Das Thema wurde und wird ja immer noch in den Einstein- Blogs hier von vorne bis hinten durchgekaut.

      • An einigen Stellen in diesem Blog ist von metaphysisch die Rede. D.h.:
        Die Physik liefert keine Antwort auf: Was ist Zeit, woher kommt sie, wohin geht sie
        und wann ist finish?
        Das kann sie auch nicht, weil sie von Beginn an einen Schöpfer ausklammert.
        Es gibt aber eine sehr zu beachtende Aussage im letzten Buch der Bibel (Offbg. 10,6),
        wo es heißt:
        “Es soll hinfort keine Zeit mehr sein“.
        Meine Gedanken gehen dahin:
        Vom Schöpfer heißt es: “Ich bin der Anfang und das Ende“.
        Diese Beschreibung deutet auf die geometrische Darstellung eines Kreises.
        ohne Anfang und Ende – sinngemäß auch gültig für die Zeit, (von Ewigkeit zu
        Ewigkeit).
        Daraus die Schlussfolgerung: Zeit gab es schon vor der Erschaffung des Universums und
        auch danach.
        Wenn es heißt: Es wird keine Zeit mehr sein, betrifft es ausschließlich die Gestalt
        des uns bekannten Universums.
        Von Beginn an ist der Begriff “Zeit“ unmittelbar mit der Expansion des Universums
        (verursacht durch kontinuierliche Zunahme der “Dunklen Energie“) verbunden,
        wie bereits im obigen Kommentar erwähnt. Die Expansion muss aber nicht an jedem
        Punkt gleich sein, wegen der ungleichen Dichte (Galaxien).

        Für die Berechnung eines Kreises benötigen Mathematik und Physik die Zahl > Phi.
        Diese Zahl war ausschlaggebend bei der Ermittlung des Grund-Bausteines des
        Universums, verbunden mit der Erzeugung der “Dunklen Energie“,
        nachzulesen unter:
        http://www.4-e-inigkeit.info/EsN-Recherche.htm
        Wie dort schon erwähnt, handelt es sich zunächst um eine hypothetische Theorie,

        • Wenn der Schöpfer ewig ist, ist er auch unendlich. Es kann nichts außerhalb des Unendlichen geben und so ist alles der Schöpfer.

          Nichts kann nicht der Schöpfer sein.

          Das _Nichts_ schwingt nicht, das das Nichts nicht da sein kann.

          Das Dasein kann nicht nicht da sein.

          🙂

          • Zu: “Das Dasein kann nicht nicht da sein” (Ende Zitat)

            Das sind (literarische) Sprachspielewie ähnlich wie : Sein oder nicht sein, das ist….
            Oder auch : Ist etwas oder ist vielmehr nichts?
            Der Satz allein ist (in sich) paradox. Denn die Tatsache dass er da steht/gedacht wurde/ aufgeschrieben wurde /gesagt wurde, beweist, dass etwas existiert.
            Klingt (!) gewaltig philosophisch. Aber der Zweck ist wohl das (literarische) Klingeln.

            Ich glaube wirklich gemeint sein kanns nur so :
            Entweder es ist etwas Bestimmtes da, oder dasjenige ist nicht da.
            DAS “Nichts” ist wohl nichts
            – als eine (zur Vereinfachung erfundene) mathematische Abstaktion (die “Null”)
            Und soll halt bedeuten, dass keine Münze mehr im Geldsäckchen ist.

          • Nein, das ist nicht der Sinngehalt der Aussage.
            Die Aussage des Daseins bezieht sich auf ein existentes untrennbares, sich stetig änderndes Ganzes.
            Das bedeutet, das eben nicht etwas Bestimmtes da sein kann.

            Genau darum geht es , Mathematik ist Sprache, mathematische Objekte sind _nicht_ existent.

            Erklärt vielleicht auch die Behauptung im anderen Thread einer ‘Existenz ohne Bewegung’, ein Objekt, das keinen Raum und keine Zeit einnimmt, kann nicht da sein, kann nicht existieren.

            “Und soll halt bedeuten, dass keine Münze mehr im Geldsäckchen ist.”
            Nein, die Münze erklärt das _Nichts_ eben gerade nicht.

            Etwas, das nicht da ist, ist nicht das _Nichts_. Ersteres ist eine Möglichkeit, letzeres, das _Nichts_ verneint jegliche Möglichkeit des Daseins überhaupt.
            Der Begriff _Nichts_ ist das behauptete Gegenteil des Seins. Oft kommt noch das Seiende ins Spiel. Dialektischer Schwachsinn. Das Sein, Existenz als Ganzes kann kein Gegenteil haben. Genau um diese Klärung des _Nichts_ geht es.

            Die Frage ist für was steht ein Begriff, beschreibt ein Begriff die Wirklichkeit?

            Nein und den meisten Wissenschaftlern ist dies nicht einmal klar. Das zeigt den allgemeinen Zustand der Wissenschaften.

          • @ F.M.A
            Stimme weitgehend zu. Deswegen sagte ich ja oben, dass das “postulieren” eines oder das Reden über ein “absolutes Nichts” (als reale Möglichkeit) etwas paradox sei.
            Denn von “Nichts” kommt nichts. Und darüber ob etwas Reales vorhanden gewesenes ab einem bestimmten Grad der “Verdünnung” irgenwann mal zu einem “Nichts” wird, streiten sich ja die Gegner der Homöopathen mit ihren Gegnern.
            Eventuell ist es so, dass das vorherige “Etwas” zwar nicht generell aus der Realität verschwindet, sondern nur keine (System-) Wirkung mehr bewerkstelligen kann.
            Oder aber die Wirkung war immer schon nur eine Art “Information” für das System die vom “Wirkstoff” auf den “Lösungsstoff übertragen werden kann. Wie die Homöopaten ja spekulieren
            Also viel Getue und viele Worte um Nichts. Aber ich essen jetzt halt doch mal Etwas.
            Weils halt Zeit ist.

    • @L. Schaber

      Der heilige A. hat all das aber vermutlich auch nicht von oben geoffenbart bekommen,

      Da muss ich Ihnen ausnahmsweise einmal zustimmen. Und wir wollen auch den gerngeschmähten Platon nicht vergessen, Augustinus war nämlich fast so sehr Platoniker wie er Christ war…

  70. AR
    die Zeit ist nicht nur eine Idee. Du kannst sie sehen, wenn du zum Sternenhimmel schaust. Der eine Stern ist 6 Lichtjahre von dir entfernt, du siehst also in die Vergangenheit, wie sie vor 6 Jahren war. Wenn das nicht real ist?

    • Am Sternenhimmel sehe ich Lichtpunkte, aber keine Zeit. Man kann beobachtbare, periodisch sich wiederholende Ereignisse abzählen und als vergleichenden Maßstab für andere Ereignisse verwenden. Das ist die Zeit.

    • Das hiese dann aber auch, dass es keine “echte Gegenwart” geben kann, weil alle “Übrmittlung” von Allem mit (begrenzter ) LG stattfinden muss. Und die neuronale Verarbeitungsgeschwindigkeit ist (entuell !?) ja noch viel geringer. Wenn Sie also einen Gedanken denken, betrifft dieser schon die Vergangenheit.
      Zumal kann niemand “die Zeit” sehen, sondern nur die “Bewegung” eines Messgerätes zur “Messung” der Zeit. Also ist das, was man am Sternenhimmel beobachtet, nicht “die Zeit” sondern es ist (extrem vereinfacht gesagt )die neuronale Reaktion der Netzhaut auf ein dort eingetroffenes Licht-Wellen/ Korpuskel .
      Zu radikale Absraktionen (als Grundannahmen) erleichtern zwar die Bildung sehr abstrahierender Hypothesen oder Theorien. Dies müssen dann aber nicht zwangsläufig etwas mit der (wirklichen (-:) Realität zu tun haben. Auch dann nicht, wenn sie konsistent sind.
      Denn falsche Grundannahmen (über die Realität) führen auch bei logischer Widerspruchsfreiheit der Theorie zu falschen (irrealen) Vorhersagen der Theorie.

      Die Sache ist also womöglich weit vertrackter,als Sie sie hier so pauschl darstellen.

  71. Zeit für die Grundfrage allen Seins

    Woher kommt alles und wohin geht alles, und wo ist das, wo alles herkommt und alles hingeht, während der Mensch diese Frage stellt?

  72. https://www.youtube.com/watch?v=kV33t8U6w28&feature=em-subs_digest-vrecs

    Die Musik ist etwas “emotional” (“dramatisch”) – aber im Universum passieren ja manchmal ebenfalls “gewaltige” (“bombastische”) Dinge. Auf Erden – od. z.B. in Form der Kollision von zwei Neutronensternen. Auch ist das Video “bereits” über 10 Jahre alt – nicht mehr der neueste (“Zu”-) Stand des Denkens. Und it’s been around for a finite amount of time (bei 1:28) ist ja nicht gesichert.

    https://www.youtube.com/watch?v=-XQODQnEr9o

  73. LSchaber, AR,
    Zeit ist nicht nur mathematisch/physikalisch zu denken, sondern in erster Linie real.
    Wenn mich ein Meteorit trifft, der seit 6 Lichtjahren unterwegs ist, dann trifft die Zeit meinen Kopf.

    Schauen Sie sich einmal vor dem Spiegel an. Zuerst ihren rechten Zeh, dann das rechte Knie, dann die rechte Gesichtshälfte. alle sind gleich alt ! Das bringt uns auf die Idee, dass wir von der Zeit her gesehen eine Einheit bilden.
    Jetzt schauen wir genauer hin. Der rechte Fußnagel, der ist nicht gleich alt. Das was übersteht, ist älter als die Stelle, wo der Fußnagel aus dem Zeh herauswächst. Das Knie ist wahrscheinlich so alt wie sie selbst, aber beim Gesicht kommen schon wieder Zweifel auf. Wenn Sie einen 2 m langen Bart haben , dann ist das Bartende älter als der Bartanfang. Und die Haut der Handinnenfläche ist vielleicht erst 2 Tage alt.
    Also zeitlich gesehen sind wir Fleckvieh.
    Nur das Gehirn macht eine Ausnahme. Das ist so alt wie wir. Das stirbt erst mit uns. Deswegen kam vielleicht Augustinus auf die Idee, dass die Zeit stille steht.

    • Du verwechselt immer noch Modellbegriff, einen Takt (Lichtjahre, gleich alt), mit Zeit=Wandel.
      Nicht einmal das Gehirn könnte ohne stetige Veränderung=Raumzeit (Bewegung=Raum & Wandel=Zeit) existieren. Hier liegt die Begründung, weshalb Konstanten nicht existieren können, Konstanten beruhen auf Sprache, nicht auf Wirklichkeit.

      Es kann kein unveränderlicher Takt da sein, er hätte keine Zeit und würde keinen Raum einnehmen.

      Es existiert kein ‘wirklicher’ Zeitpunkt.

    • Nur kurz angemerkt: einen Meteoriten, der Lichtjahre unterwegs ist, wirst Du nicht finden, glücklicherweise aber auch er Dich nicht. Das ist beruhigend. Die jibbet nämlich nur innerhalb unseres Sonnensystems. Außerdem heißen die Dinger, solange sie noch da draußen rumkurven, Meteoroide. Ab dem Eintritt in unsere Atmosphäre sind’s Meteore, und glücklich gelandet dann diplomierte Meteoriten. [/Klugscheisserei]
      Aaaber, obacht: sie brauchen solch lichtjahrelangen Anlauf garnicht, die sind so mit Eintrittgeschwindigkeiten in die Atmosphäre von 70.000, 80.000 km/h immer noch schnell genug, um beim Impakt für allerhand Ungemütlichkeit zu sorgen. Also trauen sollte man denen besser nicht.

  74. @Helmut Bäs

    wird die doch deutlich offensichtlichere Frage des Ursprungs der Dimensionalität des Universum kaum diskutiert.

    In die Liste derer, die sich mit der Dreidimensionalität befassen, sollten auch Gonzales-Ayala et al. aufgenommen werden, deren Fragestellung allerdings eine andere ist, nämlich die nach einer Notwendigkeit der Dreidimensionalität

    Was mich eigentlich noch viel mehr überrascht, ist, dass bisher nicht gefragt wurde, ob es nicht so etwas wie ein Universalgesetz gibt, mit dem u.a. auch die Dimensionalität “vorgeschrieben” ist (wobei es sich kaum um ein einzelnes Gesetz handeln kann, sondern eher um so etwas wie ein Regelwerk). Denn wie Sie schreiben:

    Da sich der Kabelsalat überhaupt nur in drei Dimensionen bilden kann, können nur dreidimensionale Universen (oder Baby-Universen) auf diese Weise “inflatieren”.

    , setzen Sie damit bereits voraus, was Sie erklären wollen. Gonzales-Ayala et al. versuchen mit ihrer Theorie zu erklären, warum es nicht mehr als nur drei räumliche Dimensionen gibt, was potenziell aber denkbar wäre.
    Was meinen Vorschlag für ein “Universalgesetz” betrifft, läuft er darauf hinaus, dass
    letztlich auch das Universum ein System ist – also eine Unterscheidung zwischen dem, was innen ist und zu ihm gehört, und dem, was außerhalb ist und nicht zu ihm gehört -, und autonome selbstorganisierende Systeme nach ihren eigenen Gesetzen funktionieren, sich diese Gesetze also selbst geben. Von diesen Gesetzen kennen wir aber nur einige, die bestimmte Phänomene betreffen, aber keines, das die ‘Organisation’ des Gesamten betrifft und daher in verschiedene (drei?) Klassen unterteilt und abstrakt genug sein muss, um die gesamte Organisation zu umfassen. Dazu müsste ein Gesetz gehören, nach dem es nur drei räumliche Dimensionen mit der Eigenschaft Zeit geben kann, Zeit also eine Eigenschaft der Raumzeit ist.

    Zu Ihrer Theorie (? oder ist es noch eine Hypothese?): Sie schreiben:

    In unserem Modell verursacht also eine neue, der QCD ähnliche Kraft (in der QCD selbst zerfallen die Flux-Tubes zu schnell) einen riesigen Kabelsalat, der das ganze Universum ausfüllte, und dessen Energie die Inflation antrieb.

    , postulieren Sie, dass es diese neue Kraft geben muss und muss nun danach gesucht werden, oder gibt es sie?

    Danke im Voraus für Ihre Antwort

    • @trice und
      “…letztlich auch das Universum ein System ist – also eine Unterscheidung zwischen dem, was innen ist und zu ihm gehört, und dem, was außerhalb ist und nicht zu ihm gehört …” (Ende des Zitats)
      Wenn Sie ihr “System” begrenzen wollen, sollten Sie es vielleicht aber nicht (mehr) UNIversum nennen. (-:

      • @L.Schaber: Uni-versum / univorsus / unus – verto

        Wenn Sie ihr “System” begrenzen wollen, sollten Sie es vielleicht aber nicht (mehr) UNIversum nennen

        Es ist nicht mein System, sondern betrifft den Begriff, der aus dem Lateinischen kommt:
        Unus – eins, verto (von vertere) – wenden (althochdeutsch: werden) und universus = ganz, sämtlich oder auch allgemein.

        Der funktional-genetische Ansatz der Systemtheorie meint, nach Luhmann, diesen Begriff: “Prozess und System sind verschiedene Aspekte von Selektivität. Der Prozessbegriff bezeichnet die Faktizität des selektiven Geschehens und damit die Notwendigkeiteiner Grenzziehung; der Systembegriff bezeichnet die notwendige Grenzziehung.”

        • Ich bezog mich auf “universal” im Sinne von “allumfassend” bzw. ALLES umfassend. Und wenn man da dan über Begrenzungen spekuliert, wirds (zumindest) schwierig.

          • @L.Schaber

            Ich bezog mich auf “universal” im Sinne von “allumfassend” bzw. ALLES umfassend.

            Fein, ich auch, 🙂

            Und wenn man da dan über Begrenzungen spekuliert, wirds (zumindest) schwierig.

            Ganz im Gegenteil, es wird einfach.

    • Hallo Trice,

      In die Liste derer, die sich mit der Dreidimensionalität befassen, sollten auch Gonzales-Ayala et al. aufgenommen werden

      Danke für den Hinweis, das Paper könnte interessant sein, aber um das zu beurteilen, muss ich das erst in Ruhe lesen.

      Was mich eigentlich noch viel mehr überrascht, ist, dass bisher nicht gefragt wurde, ob es nicht so etwas wie ein Universalgesetz gibt, mit dem u.a. auch die Dimensionalität “vorgeschrieben” ist

      Nun wie gesagt gibt es Vorschriften für Dimensionalitäten in Quantengravitationstheorien, aber das Ergebnis ist typischerweise nicht 3+1…

      , setzen Sie damit bereits voraus, was Sie erklären wollen.

      Das tun wir nicht. Es könnten entweder in einem höherdimensionalen Universum nur 3+1-Branes durch Inflation makroskopisch werden, oder alternativ im Multiversum nur 3+1-dimensionale Babyuniversen eine Inflation und damit Leben zulassen (womit wir im zweiten Fall zugegebenerweise wieder bei einer anthropischen Argumentation angelangt sind.

      letztlich auch das Universum ein System ist – also eine Unterscheidung zwischen dem, was innen ist und zu ihm gehört, und dem, was außerhalb ist und nicht zu ihm gehört -, und autonome selbstorganisierende Systeme nach ihren eigenen Gesetzen funktionieren, sich diese Gesetze also selbst geben.

      Ich glaube nicht, dass wir beim Universum von Selbstorganisation sprechen können – dazu müssten Strukturen “im Innern” auf Kosten einer Entropiezunahme “außen” entstehen und das erscheint mir zumindest auf den ersten Blick unmöglich, ist das Universum doch größer als der Horizont, also der maximal beeinflussbare Bereich.

      Zu Ihrer Theorie (? oder ist es noch eine Hypothese?): Sie schreiben:

      Ich würde es momentan “Szenario” nennen….

      , postulieren Sie, dass es diese neue Kraft geben muss und muss nun danach gesucht werden, oder gibt es sie?

      Richtig, diese Kraft müsste es geben, sowie die Felder, die ihre Symmetriebrechung bewirken und diejenigen, die in der QCD den Quarks entsprechen. Möglicherweise haben die aus den neuen, den Quarks entsprechenden Teilchen aufgebauten “Hadronen” etwas mit der Dunklen Materie zu tun. Das genau auszuarbeiten und konkrete Vorhersagen für Experimente und/oder Beobachtungen zu formulieren, ist für die Zukunft geplant und würde unser Szenario zu einem Modell machen.

  75. Unfassbar – nachdem ich nun auch noch den Nachnamen verbockt habe, brauche ich jetzt entweder einen Kaffee oder frische Luft – Herr Päs, ich entschuldige mich ein weiteres Mal und gelobe Besserung

  76. Wie es scheint, @Trice, kommen sie aus einer anderen Dimension und müssen sich erst noch ein wenig aklimatisieren ; – )

  77. @ Ah, nonymous

    Den Vorschlag hat man mir auch schon früher gemacht, 🙂 aber er kommt nicht so ganz hin, weshalb es mit der Akklimatisierung auch so schwierig ist. 🙁

    Dennoch hoffe ich auf Vergebung und eine Antwort ….

  78. spezielle Fragen für @Trice

    Angenommen, ein System existierte tatsächlich, was ist die Folge?

    Wie kann ein System ohne Teile gedacht werden?

    Ist der Beobachter ein Teil des Ganzen oder ein Teil eines Systems oder beides?

    Ist der Beobachter ein System, das ein anderes System beobachtet?

    Was ist zwischen den Teilen/Systemen?

    Wie kann eine _existente_ Grenze, die ja bleiben muß, wie sie ist, beobachtet werden, die ja logisch durch ihre Endbedingung keine weitere Folge haben kann?

    Ist hinter der Grenze das _Nichts_?

    Wie kann das _Nichts_ da sein, wo doch das _Nichts_ behauptet, es kann nichts, also auch das _Nichts_ nicht da sein?

    Ist ohne Ende zu zählen, Unendlichkeit?

    Haben Zahlen Grenzen?

    Kann Unendlichkeit wirklich eine Grenze, wie etwa eine Dimension haben?

    Wenn ein Teil absolut existiert, wie kann diese einzig wahre Wahrheit nicht vollständig und endgültig sein?

    Käme zu einer einzig wahren Wahrheit etwas dazu, ist es dann noch eine einzig wahre Wahrheit?

    Woher kommt die Energie in einer endlichen Welt, die ja bleibt wie sie ist?

    Woher kommt die Energie für ein Teil, das ja Grenzen haben muß?

    Wann beginnt ein Teil ein Teil zu sein?

    Ist die Systembetrachtung ein geozentrisches, heliozentrisches oder ein EGOzentrisches Weltbild?

  79. @Franz Maria Arwee : Fragen

    Sie stellen eine Menge Fragen, von denen nicht klar ist, worauf Sie hinauswollen und was die Fragen mit dem Blog-Beitrag zu tun haben. Einige lassen sich schon deshalb nicht beantworten, weil sie keinen Sinn ergeben. Frage von mir: Sagt Ihnen der Begriff “Systemtheorie” etwas? Wenn nicht, werden Ihnen die Antworten nicht viel weiterhelfen.

    Angenommen, ein System existierte tatsächlich, was ist die Folge?

    Diese gehört zu den sinnlosen oder sinnfreien Fragen, denn wir sind Systeme und leben in Systemen, also nach welchen Folgen fragen Sie?

    Wie kann ein System ohne Teile gedacht werden?

    Gar nicht.

    Ist der Beobachter ein Teil des Ganzen oder ein Teil eines Systems oder beides?

    Aha, also ganz so ahnungslos sind Sie offenbar nicht. Welchen Beobachter meinen Sie, den Beobachter erster Ordnug oder den Beobachter zweiter Ordnung? Der Beobachter erster Ordnung ist Teil des Systems …

    Ist der Beobachter ein System, das ein anderes System beobachtet?

    Welcher?

    Was ist zwischen den Teilen/Systemen?

    Wovon reden Sie jetzt? Was meinen Sie mit “Teilen” ?Eine Definition von System lautet, es sei eine Menge von wechselseitig voneinander abhängigen Elementen und deren Beziehungen. Meinen Sie mit Teilen die Elemente, die Beziehungen oder was?
    Im Übrigen ist diese Definition für hochkomplexe Systeme unbrauchbar, u.a., weil mit wachsender Komplexität sich das Verhältnis von Element und System auch qualitativ verändert.

    Und dann: von welchen Systemen reden wir? Biologischen, soziologischen, physikalischen – selbstorganisierenden, autonomen, ?

    Wie kann eine _existente_ Grenze, die ja bleiben muß, wie sie ist, beobachtet werden, die ja logisch durch ihre Endbedingung keine weitere Folge haben kann?

    Ich verstehe die Frage nicht. Wieso muss die Grenze beobachtet werden? Die Beobachtung betrifft die internen Prozesse

    Ist hinter der Grenze das _Nichts_?

    Andere Umwelten. Ein System existiert nur in Verbindung mit seinen Umwelten – z. B. anderen Universen, Branen …

    Wie kann das _Nichts_ da sein, wo doch das _Nichts_ behauptet, es kann nichts, also auch das _Nichts_ nicht da sein?

    Parmenides: Es gibt kein Nichts? Heidegger: Das Nichts nichtet?

    Kann Unendlichkeit wirklich eine Grenze, wie etwa eine Dimension haben?

    Inwiefern ist eine Dimension, wie die von denen Herr Päs spricht, eine Grenze?

    Die nächsten Fragen ergeben ebenfalls keinen Sinn, es sei denn, Sie können sie präzisieren. Vielleicht mögen Sie sich ja mal mit Systemtheorie befassen? Es muss ja nicht unbedingt nur Luhmann sein.

    Ist die Systembetrachtung ein geozentrisches, heliozentrisches oder ein EGOzentrisches Weltbild?

    Sie ist kein Weltbild, sie ist eine Betrachtungsweise oder auch eine Art und Weise, etwas Komplexes zu untersuchen, ohne es in seiner Einzelteile zu zerlegen.

    • Wie kann ein System ohne Teile gedacht werden?
      Gar nicht.

      Wie wäre es mit einem verschränkten Quantensystem?
      Bei den anderen Fragen teile ich allerdings Ihre Ratlosigkeit 🙂

      • @ Heinrich Päs
        Ratlosigkeit klingt gut.
        Einige befinden sich in einer Endlosschleife von Zirkelschlüssen.

        Da ich eine ‘offene Welt’ für existent halte, kann ich es nicht sein, selbst wenn ich möglicherweise derjenige bin, der in der Illusion lebt, etwas ändern zu können.

      • @Herr Päs

        Wie wäre es mit einem verschränkten Quantensystem?

        Ok, daran habe ich nicht gedacht. Vielleicht müsste auch geklärt werden, was genau unter dem Begriff System von uns verstanden wird (in der Physik ist ja auch ein Pendel ein System, 😉 )

    • Die Fragen stelle ich als Logiker, der aus der formalen Logik kommt. Ich habe viel mehr den Eindruck, daß sehr vielen nicht klar ist, wie sinnfrei ein System wirklich ist.
      Mir ist auch ‘theologische Kybernetik’ ein Begriff, die sehr viel älter ist, als Norbert Wieners Begriff der Kybernetik.

      Wie unterscheidet ein Mensch denn ohne Grenze zwischen Dimensionen?

      Was all dies hier zu suchen hat?
      Die Grundlage der Systemtheorie, ist die Kybernetik, nur mal so.

      Ein System ist quantitativ, nicht qualitativ, ansonsten bräuchte es nicht ständig den Begriff Emergenz.

      Ich bin kein System, in mir lebt auch kein System, lebe in keinem System und beabsichtige auch nicht dies zu tun zu können, das ist etwas für Systemgläubige…
      Es existiert kein System, nirgends. Jedenfalls nicht, von dem ich etwas wissen könnte.

      Über eine andere Umwelt ‘jenseits’ einer existenten Grenze, kann keiner etwas wissen, daran kann der Mensch nur Glauben.

      Euch ist nicht einmal klar, daß ihr einzig wahre Wahrheiten behauptet und damit nichts anderes als eine Religion betreibt.

      Aber gut, Gedankenexperiment:
      Angenommen, ein System existiert.
      Dieses System wird von Gesetzen bestimmt. Auch wenn ein Element das Ganze nicht bestimmen (berechnen) kann, so ist trotzdem die Grundannahme, daß die Welt bestimmbar sei.
      Das bedeutet, daß Gewißheit existiert. Wenn Gewißheit existiert, kann nichts nicht gewiß sein. Existiert Gewißheit, ist Offenheit und Freiheit nur Illusion. Eine Welt der Offenbarung, der übernatürlichen Vernunft. Die Sprache Mathematik ist hier ebenso überflüssig wie Logik, denn diese ändern nichts.
      Warum will der Mensch, in einer Welt, die ‘a priori’ bereits vollständig gewiß sein muß, Gewißheit schaffen?
      Das _IST_ sinnfrei.

      Angenommen es existiert kein System
      In diesem Fall gibt es keine Systeme, keine Naturgesetze, keine Regeln. Alles ist ungewiß. Offenheit und Freiheit ist ein existenter Zustand, Gewißheit ist eine Illusion.
      Warum will der Mensch, in einer Welt, die die Voraussetzung von Ungewißheit hat, Gewißheit schaffen?
      Das _IST_ sinnfrei.

      In _beiden_ Betrachtungen ist ein System schaffen zu wollen, wirklich sinnfrei.
      Die zunehmende Komplexität ist eine geschaffene Illusion der Systemtheoretiker, die Herde der Regelianer.

      • @FAW
        Auch die Frage nach Teil und Ganzem, nach Systemen inklusive Varela usw. ist zweifellos ganz spannend und auch für die Physik (z.B. das Verständnis der Quantenphysik höchst relevant). Mir scheint allerdings, dass Sie ignorieren, dass wir über die Welt auf verschiedenen Ebenen sprechen können. Wir bestehen aus Atomen, die sich (modulo Quantenphysik) deterministisch verhalten. Heisst das, das wir keinen freien Willen haben? Würde ich nicht sagen, denn der freie Wille bezieht sich auf eine andere Organisationsebene, in der z.B. bestimmte Informationsinhalte viel wichtiger sind als das materielle Substrat. Sie schütten mit Ihrer Fundamentalfragerei das Kind mit dem Bade aus.

        • Existiert wirklich ein Atom (Teilchen welcher Form auch immer), ist es völlig wurscht, was immer irgendein Mensch denkt, weiß oder glaubt, denn die Aussage behauptet, die Welt, das Universum, sei bestimmbar und folglich unveränderlich.
          Ist das der Fall, bleibt alles wie es ist und wir leben in der Leibniz’schen besten aller Welten. Es ist aufgrund der gesetzten Prämissen gleichgültig, wieviel anschliessend für den menschlichen Betrachter relativiert wird.

          Nichts kann verhindert, nichts kann verändert werden.

      • @Franz Maria Arwee

        Die Grundlage der Systemtheorie, ist die Kybernetik, nur mal so.

        Stimmt, damit fing es an: Am Anfang war der Regelkreis.
        Und wie Norbert Bischof es so schön formulierte: “Das problem ist nur, daß eine Mode noch keinen Paradigmenwechsel macht. Was unbedingt hinzukommen muß, ist eine erkennbare Steigerung des wissenschaftlichen Ertrages. Diese aber setzt Professionalität voraus, und es war das Verhängnis der Kybernetik, daß es just an dieser Professionalität mangelte. ”
        Ist es das, was Sie kritisieren möchten?

        Ein System ist quantitativ, nicht qualitativ, ansonsten bräuchte es nicht ständig den Begriff Emergenz.

        Hm, klingt irgendwie sehr verkürzt …

        Ich bin kein System, in mir lebt auch kein System, lebe in keinem System und beabsichtige auch nicht dies zu tun zu können.

        Mir anderen Worten: Sie diskutieren etwas, dass es Ihrer Meinung nach nicht gibt. Warum tun Sie es dann?

        Es existiert kein System, nirgends. Jedenfalls nicht, von dem ich etwas wissen könnte.

        Das heißt, für Sie ergibt z. B. die Unterscheidung zwischen selbstorganisierenden und offenen (Input-Output-) Systemen keien Sinn?

        Über eine andere Umwelt ‘jenseits’ einer existenten Grenze, kann keiner etwas wissen, daran kann der Mensch nur Glauben.

        Sie sind nicht zufällig Solipsist?

        • @trice und:
          “..Über eine andere Umwelt ‘jenseits’ einer existenten Grenze, kann keiner etwas wissen, daran kann der Mensch nur Glauben….” (Ende Zitat)

          Woher wissen Sie das und weshalb sind Sie sich dessen so sicher?

  80. @ Trice und:
    ” … weil mit wachsender Komplexität sich das Verhältnis von Element und System auch qualitativ verändert. (Ende des Zitats)
    Klingt ziemlich mystisch. Sie meinen also, bei Erhöhung der Systemkomplexität entstünde etwas völlig neues? Eventuell etwas, das sich aus den Systemkomponenten nicht mehr ableiten lasse oder dessen neue Qualität sich nicht mehr auf die ursprünglichen Komponenten des Systems oder auf deren vorheriges (einfachere) Zusammenwirken zurückführen lasse?
    Das wäre aber dann zunächst mal nur reine (metaphysische) (KI-) Spekulation. Wie sie ja sehr häufig (auch) von SF – Autoren Thematisiert wird.

    Wie schrieben Sie doch oben:
    …..ergeben ebenfalls keinen Sinn, es sei denn, Sie können sie präzisieren.”

    Und nur nebenbei: Das Fokussieren auf “Systeme” und deren Funktionalitäten gibt es in der Philosophie schon recht lange. Man nannte es “Strukturalismus.”

    • @L.Schaber

      Sie meinen also, bei Erhöhung der Systemkomplexität entstünde etwas völlig neues?

      Eine Erhöhung der Komplexität ergibt sich schon dadurch, dass etwas Neues entsteht.

      Eventuell etwas, das sich aus den Systemkomponenten nicht mehr ableiten lasse oder dessen neue Qualität sich nicht mehr auf die ursprünglichen Komponenten des Systems oder auf deren vorheriges (einfachere) Zusammenwirken zurückführen lasse?

      Warum schreiben Sie nicht einfach: Emergenz? 😉
      Kennen Sie die Formel, die diesen Vorgang beschreibt?

      • Nein.
        Erklären Sie allgemeinverständlich. Und bedenken Sie: Was man überhaupt sagen (und denken) kann, kann man vermutlich auch klar sagen.

        • @L.Schaber : Formeln

          Auf meine Frage nach der Formel, die den Vorgang der Emergenz beschreibt, antworten Sie mit:

          Nein.

          Diese Antwort habe ich erwartet, das Folgende jedoch icht:

          Erklären Sie allgemeinverständlich. Und bedenken Sie: Was man überhaupt sagen (und denken) kann, kann man vermutlich auch klar sagen.

          Es trifft zu, dass man das, was man denken und sagen kann, auch klar sagen kann. In diesem Fall gibt es aber zwei Hindernisse, die dazu überwunden werden müssten. Hindernis 1 ist, dass diese Formel zu einem Satz (oder einer Klasse) von Formeln gehört, die ein großes Theorienetz bilden. Man muss also auch diesen Zusammenhang kennen, und das würde hier den Rahmen sprengen. Und Hindernis 2, das vermutlich noch schwieriger zu überwinden sein dürfte, haben Sie genannt:

          Wissenschaft ist eine (Denk-) Methode, möglichst ohne allzu wirre Umwege etwas über die Realität zu erfahren. Und Folgerungen aus der Erkenntnis der Realität oder Wirklichkeit (=was generell die Potenz besitzt auf irgebndetwas für mich Releventes einzuwirken), also Technologie, sind Werkzeuge dazu, Ziele zu erreichen, die sich Menschen setzen.

          Dies ist eine knappe Beschreibung der Art und Weise, wie Wissenschaft betrieben wird. Mit meiner obigen Frage an Herrn Päs, dass es mich überrascht, dass bisher noch nicht nach einem Universalgesetz (das kein einzelnes Gesetz sein kann, sondern so etwas wie ein Theorienetz, dessen Elemente /Theorien die einzelnen Formeln sind) gesucht wurde. Stattdessen macht man, was Sie beschreiben: man untersucht mit den Mitteln, die die Technologie zur Verfügung stellt, die einzelnen Teile / Teilstücke eines Ganzen separat, meist indem man immer mehr reduziert, und versucht, über die Ergebnisse zu einem Modell des Ganzen zu kommen. Wobei jede Disziplin ihre eigenen Modelle hat, die nicht unbedingt kompatibel sind.
          Das heißt, die Hürde, die genommen werden muss, ist ein Umdenken.

          • Die alte Diskusssion halt. Meist unter der Überschrift: “Das Problem des Reduktionismus”.

  81. Tja, die Physik und die Philosophie… Tatsächlich findet man bei alten Philosophen und in religiösen Texten oft interessante Ideen, die manchmal auch modernen wissenschaftlichen Konzepten verblüffend ähneln können. Das kann verschiedene Gründe haben: Vielleicht wird Wissenschaft durch unser kulturelles Erbe geprägt? Vielleicht hat die Menschheit überhaupt nur eine Handvoll von Konzepten, die in verschiedenen Abstraktions- und Elaborationsgraden immer wieder auftauchen? Vielleicht lesen wir in der Rückschau in alte Quellen jetziges Wissen herein? Oder vielleicht gibt es ein intuitives Verständnis wahrer Zusammenhänge, das den antiken Philosophen zugänglich war und jetzt bestätigt wird? Das alles ist möglich…

    Aber natürlich sind wissenschaftliche Szenarien wie der kosmische Kabelsalat im Gegensatz zu den Behauptungen antiker Denker in ein funktionierende Paradigmen (Quantenfeldtheorie, Symmetriebrechung, Inflation usw…) eingebunden und können durch Konkretisierung zu Modellen befördert werden, die dann Vorhersagen für zukünftige Experimente und/oder Beobachtungen machen, z.B. die Existenz neuartiger Teilchen, die aus den Feldern aufgebaut sind, die hier die Rolle der Quarks einnehmen, und nach denen man suchen kann.

    Schließlich ist es zweifellos richtig, dass Physik oft auf einem wackligen philosophischen Fundament aufgebaut ist. Nichtdestotrotz funktioniert sie, wie die Anwendungen der Grundlagenwissenschaft in modernen Technologien zeigen.

    Philosophie kann für die Physik also zweierlei leisten: Sie kann neue Ideen und Konzepte liefern, und sie kann der Physik helfen zu reflektieren, was die Physik eigentlich macht und was das bedeutet. Oft verschwimmen dabei auch die Grenzen der Disziplinen. Die Besonderheit, die die Physik auszeichnet, nämlich experimentell testbare Modelle der Realität zu entwerfen, die quantitative, mathematische Vorhersagen erlauben, ist aber zweifellos durch nichts anderes zu ersetzen. Wer das ignoriert gehört mit seinem Denken ins Mittelalter und nicht an eine Computertastatur… (und ich glaube die allermeisten wirklichen Philosophen würden mir da zustimmen)

    • Ludwik Fleck
      Entstehung und Entwicklung einer wissenschaftlichen Tatsache

      Texas Sharpshooter. Bestätigungsfehler.

      Was wird denn entworfen, Modelle der Wirklichkeit, oder Modelle eines Meßaufbaus?

      Genau hier beginnt die sprachliche Unschärfe, die Verschleierung.

      Aus einer Tatsache, des Sein, läßt sich keine Regel, kein Gesetz, kein sollen, ableiten.
      Das nennt sich Sein-Sollen-Fehlschluß.

      Die Wirklichkeit ist nicht meßbar, nicht quantisierbar, infolge sie sich stetig verändert.
      Die Wirklichkeit ist nur erlebbar, nicht beschreibbar. Aus diesen wenigen Aussagen ergibt sich, daß wir nur bestimmen könne, was _nicht_ Wirklichkeit sein kann. Das ist mein Punkt.

      Die mathematisch-naturwissenschaftliche Methode, Physik im besonderen, hat ein absolutes und totalitäres Weltbild in die Menschen gepflanzt, die Folgen waren und sind in meinen Augen entsprechend, insbesondere psychologisch, verheerend.

      Ich zweifle nicht an der Wirklichkeit. Ich zweifle nicht an den Erfolgen des Forschens, wobei noch zu klären wäre, was als Erfolg zu gelten hat.
      Ich zweifle an dem vermittelten Weltbild, an den Anmaßungen der Gottgleichheit, der Erkenntnisfähigkeit des Menschen. Ich zweifle an der Existenz höherer Wesen. Ich denke, daß wir tatsächlich sehr viel weniger wissen, als die Wissenschaft ‘glaubt zu wissen’. Dialektik, das Formale, wird nachweislich mit Logik, den wirklichen Folgen verwechselt. Letzteres wird wissentlich ignoriert.

      Damit keine Mißverständnisse auftreten, ich finde, es wird viel zu wenig generell, speziell experimentell, geforscht. In der Informatik ist das sicher noch eine Stufe religiöser.

      Ich denke, daß mit all dem Glauben an eine heilige allwissende Wissenschaft, dies jeder Menschlichkeit zuwider läuft. Letztlich auch die Lebensgrundlage für Menschen durch Menschen zerstört.

      Wissenschaft ist mit den gegebenen Prämissen, nur eine sich selbsterfüllende Prophezeiung.

      Ich habe nicht vor irgendeinen aufzuhalten, könnte ich auch nicht. Was ich mache ist etwas anderes, ich lasse nicht zu, daß sich Wissenschaftler nach der nächsten erwartbaren Katastrophe hinstellen und wieder mal sagen, sie hätten nichts gewußt, das Opfer im Dramadreieck, sofern noch einer da ist, dem überhaupt noch etwas gesagt werden könnte.

      • Sie verstehen nicht, dass jede Aussage über die Welt Physik ist – oder inhaltsleer. Bei den ihrigen habe ich den Verdacht, dass eher das zweite zutrifft. Wer die Wissenschaft ablehnt, lehnt damit die Welt selbst ab in ihrer Vielfältigkeit und Schönheit. Das ist m.E. die wahre Quelle des Fundamentalismus. Niemand behauptet die Physik liefere zeitlose Wahrheiten, diesen Anspruch erheben nur Sie.

      • Sie übertreiben gewaltig. Wissenschaft ist eine (Denk-) Methode, möglichst ohne allzu wirre Umwege etwas über die Realität zu erfahren. Und Folgerungen aus der Erkenntnis der Realität oder Wirklichkeit (=was generell die Potenz besitzt auf irgebndetwas für mich Releventes einzuwirken), also Technologie, sind Werkzeuge dazu, Ziele zu erreichen, die sich Menschen setzen.

        Man muss halt dafür Sorgen, dasss “Die Bösen” nichts Schlimmes damit anfangen. Und gleichzeitig das Werkzeug einsetzen für das , was man für “das Gute” hält.
        Das nennt man Politik.
        Darüber zu jammern, dass Gottes Geschöpfe ihrem Schöpfer (mit Wissenschaft und Technologie ins Handwerk pfuschen, ist (inhumanes) Mittelaterdenken.

  82. > Heinrich Päs, 11. Dezember 2017 @ 19:38
    > Die Besonderheit, die die Physik auszeichnet, nämlich experimentell testbare Modelle der Realität zu entwerfen, die quantitative, mathematische Vorhersagen erlauben, ist aber zweifellos durch nichts anderes zu ersetzen.

    Nicht nur das. Die Vorhersagen sind auch auf Phänomene anwendbar, die zum Zeitpunkt der Entstehung einer Theorie noch gar nicht bekannt oder denkbar waren. Vorhersagen konnten auch die frühen Astronomen machen. Doch mit ihren Erkenntnissen kann man keine Sonde zu Jupiter oder Saturn schicken. Man muss es deutlich sagen: Sie wussten nicht, warum der Mond an diesem Zeitpunkt die Sonne verdeckt ungeachtet der Tatsache, das sie diesen ziemlich genau vorhersagen konnten. Sie waren ahnungslos.

    Ingenieure stützen sich auf Erkenntnisse und Methoden der Physik. So bestens gerüstet können sie Vorhersagen machen. Besser als Versuch und Irrtum schneiden sie immer ab. Doch all zu weit sollten sie lieber nicht extrapolieren.

    Anders geht es in der Physik. Nach mehr als drei Jahrhunderten sind deren Erkenntnisse immer noch aktuell. Die Satelliten des GPS bewegen sich genauso wie man es nach der Newtonschen Theorie erwarten würde. Die Voyager Sonden tun es seit vier Jahrzehnten ebenso.

    Das Verschmelzen zweier ziemlich schwerer Schwarzer Löcher wird durch die Allgemeinen Relativitätstheorie befriedigend beschrieben. Diese basiert auf allgemeinen Überlegungen und korrigiert die Newtonsche Theorie dort, wo auf Grund besonderer Umstände Diskrepanzen zu Beobachtungen bestehen. Die Bedingungen beim Verschmelzen der Schwarzen Löcher unterscheiden sich enorm von denen in der Umlaufbahn des Merkurs um die Sonne und trotzdem funktioniert die Allgemeine Relativitätstheorie befriedigend.

    Die Realität ist so kompliziert, dass man die Gleichungen der Physik gar nicht exakt lösen kann. Trotzdem ist die Physik brauchbar und äußerst effektiv. Sie erlaubt es, einfache Modelle zu erstellen, die die Realität sehr befriedigend beschreiben. Der Physiker weiß, was er berücksichtigen muss und was er vernachlässigen darf.

    • Karl Mistelberger schrieb (12. Dezember 2017 @ 07:31):
      > Die Satelliten des GPS bewegen sich genauso wie man es nach der Newtonschen Theorie erwarten würde. […]

      Macht “die Newtonsche Theorie” denn irgendwelche konkreten Vorhersagen hinsichtlich der “Kräfte“, die auf Satelliten des GPS “einwirkten”, bzw. insbesondere hinsichtlich der Verteilung von “Masse(n) “um” diese Satelliten des GPS ??

      Wohl kaum.
      Stattdessen handelt es sich um Vorhersagen von (bestimmten astro- bzw. geo-physikalischen) Modellen,
      die unter Benutzung (von bereits ermittelten oder noch hypothetischen Werten) dieser Größen “der Newtonschen Theorie” aufgestellt werden mögen.

      • > Macht “die Newtonsche Theorie” denn irgendwelche konkreten Vorhersagen hinsichtlich der “Kräfte“, die auf Satelliten des GPS “einwirkten”, bzw. insbesondere hinsichtlich der Verteilung von “Masse(n) “um” diese Satelliten des GPS ?

        Das Gravitationsfeld der Erde ist kompliziert. Es wird durch Gravimeter an der Erdoberfläche und aus der Bewegung niedrig fliegender Satelliten bestimmt. Mit den gemessenen Daten wird ein Modell des Gravitationsfeldes der Erde erstellt. Dieses Gravitationsfeld ist die dominante Komponente der Kräfte, die auf die GPS Satelliten einwirken.

        Die Widerspruchsfreiheit und Konsistenz der Messungen und Berechnungen wird von keiner anderen Methode als der Physik erreicht. Die Philosophie schweigt dazu. Der schlecht aufgelegte Zeitgenosse redet oft nur von “Zahlenwichserei”.

    • @ Karl Mistelberger und zu :
      “…..Die Realität ist so kompliziert, dass man die Gleichungen der Physik gar nicht exakt lösen kann. Trotzdem ist die Physik brauchbar und äußerst effektiv. Sie erlaubt es, einfache Modelle zu erstellen, die die Realität sehr befriedigend beschreiben. …”
      (Ende Zitat)
      1.
      Woher wissen Sie eigentlich (so genau Bescheid über das) was sie im erste Satz des obigen Zitats behaupten?
      2.
      Sie sprechen (wohlweislich) immer nur von “befriedigender” Physik. Wie müsste denn nach ihrer Meinung eine Physik beschaffen sein, der Sie die Note ” gut” oder gar “sehr gut” zu erteilen bereit wären?

      • > Woher wissen Sie eigentlich (so genau Bescheid über das) was sie im erste Satz des obigen Zitats behaupten?

        Siehe die Antwort auf Frank Wappler oben.

        > Sie sprechen (wohlweislich) immer nur von “befriedigender” Physik. Wie müsste denn nach ihrer Meinung eine Physik beschaffen sein, der Sie die Note ” gut” oder gar “sehr gut” zu erteilen bereit wären?

        Physiker sind gierig. Wenn sie etwas erreicht haben dann liegt die Latte höher. Die Geschichte geht weiter. Sehr gut ist auch nicht das Ende der Fahnenstange.

  83. Herr Päs – 11.12. – 19:38

    Tja, die Physik und die Philosophie …

    Ich habe (wenn auch auf die Gefahr hin “immer” bloß “Links” zu setzen) dazu letztens **Der Ursprung des Bewusstseins durch den Zusammenbruch der bikameralen Psyche** von J. Jaynes gelesen [Psychologie/Philosophie und Life Science (Biowissenschaft)]. Und jetzt bin ich gerade bei **… und Gott würfelt doch: Irrtümer und Halbwahrheiten über die Quanten … und wie es wirklich ist** von T. Görnitz — durchaus empfehlenswert.

  84. @ Herr Päs,

    vielen Dank für die Annahme meiner Entschuldigung – und natürlich auch für Ihre Antworten, 🙂
    Also:

    Nun wie gesagt gibt es Vorschriften für Dimensionalitäten in Quantengravitationstheorien, aber das Ergebnis ist typischerweise nicht 3+1…
    Das [voraussetzen, was Sie erklären wollen] tun wir nicht. Es könnten entweder in einem höherdimensionalen Universum nur 3+1-Branes durch Inflation makroskopisch werden, oder alternativ im Multiversum nur 3+1-dimensionale Babyuniversen eine Inflation und damit Leben zulassen

    Gut, das leuchtet mir ein, es beantwortet aber noch nicht meine Frage, die ich wohl auch nicht präzise genug gestellt habe. Ich werde noch mal darüber nachdenken müssen.

    Ich glaube nicht, dass wir beim Universum von Selbstorganisation sprechen können – dazu müssten Strukturen “im Innern” auf Kosten einer Entropiezunahme “außen” entstehen und das erscheint mir zumindest auf den ersten Blick unmöglich, ist das Universum doch größer als der Horizont, also der maximal beeinflussbare Bereich.

    Ich gebe zu, ich habe den Begriff Selbstorganisation recht großzügig verwende – gemeinhin ist ja damit gemeint, dass die gestaltenden Einflüsse vom System selbst ausgehen, von ihm organisiert werden, während ich darunter verstehe, dass es Regeln, Gesetze, Vorschriften geben muss, die der Organisation dienen.

    diese Kraft müsste es geben, sowie die Felder, die ihre Symmetriebrechung bewirken und diejenigen, die in der QCD den Quarks entsprechen. Möglicherweise haben die aus den neuen, den Quarks entsprechenden Teilchen aufgebauten “Hadronen” etwas mit der Dunklen Materie zu tun.

    Ja, das: “die den Quarks entsprechenden Teilchen” habe ich wissen wollen, danke, 🙂

  85. HP,
    Die Grenze der Physik ist die Sprache. Wenn es den Begriff Atom nicht gegeben hätte, gäbe es heute keine Chemie und keine Kernphysik, die Thermodynamik wahrscheinlich auch nicht.
    Wir warten auf ein neues Genie mit den Qualitäten eines Newton, Maxwell oder Einstein.
    Daran führt kein Weg vorbei.

  86. @ heinrich Päs und zu dem Folgenden von ihm:
    “….Die Besonderheit, die die Physik auszeichnet, nämlich experimentell testbare Modelle der Realität zu entwerfen, die quantitative, mathematische Vorhersagen erlauben, ist aber zweifellos durch nichts anderes zu ersetzen. Wer das ignoriert gehört mit seinem Denken ins Mittelalter und nicht an eine Computertastatur… (und ich glaube die allermeisten wirklichen Philosophen würden mir da zustimmen)……”
    (Ende des Zitats)

    1. Wer aber, Herr Päs, ignoriert das denn (zum Beispiel hier)?
    2. Was sie in ihrem Idealbild gnädig verschweigen, ist zum Beispiel, dass sehr viele der “großen” physikalischen Theorien ( Über “Alles” ) überhaupt keine Wert mehr auf reale Testbarkeit oder Falsifikation in absehbaren Zeiträumen mehr legen, sondern allein der internen Stringenz und Kohärenz iherer mathematischen Modellbildungen vertrauen wollen.
    Und wenn phsikalische Vorhersagen gemacht werden, die zumindest im Prinzip experimentell zu “prüfen” sind, liegen diese merkwürdigerweise immer im Graubereich des gerade noch oder gerade nicht mehr so genau Messbaren. Sodass nur unter Aufbietung allen möglichen statistisch- mathematische Tricks eine vernünftige Ausssage herausgekitzeltt werden kann.

  87. Karl Mistelberger schrieb (12. Dezember 2017 @ 20:09):
    > Das Gravitationsfeld der Erde ist kompliziert. Es wird durch Gravimeter an der Erdoberfläche und aus der Bewegung niedrig fliegender Satelliten bestimmt. Mit den gemessenen Daten wird ein Modell des Gravitationsfeldes der Erde erstellt

    Dann sind wir uns wohl darüber einig, dass ein/jedes solche(s) Modell von der/jeder Theorie zu unterscheiden ist, die die zugrundeliegenden Begriffe bzw. Messgrößen (“Gravimeter” bzw. “Schwerebeschleunigung“, “Oberfläche“, “Bewegung“, “Satellit“, “Kraft” usw.) überhaupt erst definieren würde.

    > Dieses Gravitationsfeld ist die dominante Komponente der Kräfte, die auf die GPS Satelliten einwirken.

    Ist diese Dominanz eine Konsequenz der Definition des Begriffs “Satellit” bzw. insbesondere “GPS Satellit
    (im Zusammenhang mit den übrigen definierten Begriffen),
    also ein logisch-zwingendes Theorem der entsprechenden Theorie ?

    Oder (“nur”) eine Modell-Annahme, die Fall zu Fall experimentell prübar wäre ?

    > Die Widerspruchsfreiheit und Konsistenz der Messungen und Berechnungen […]

    Widerspruchsfreiheit bzw. Konsistenz wird offenbar bestimmten Systemen aus selbstverständlichen Begriffen (Axiomen), daraus formulierten Definitionen (z.B. von Messgrößen bzw. -operatoren) und deren logischen Konsequenzen (Theoremen) zugesprochen; also bestimmten Theorien.

    Die eventuelle Übereinstimmung von Messwerten mit den Annahmen bzw. Vorhersagen bestimmter Modelle (einschließlich der zwangsläufigen Nicht-Übereinstimmung mit bestimmten anderen Modellen) … ist wohl ein anderes Thema.

  88. @L.Schaber;
    Natürlich haben Sie recht mit Ihrer Kritik an der Physik. Das ist den Physikern aber längst bekannt. Insofern rennen Sie nur offene Türen ein. Viele Kritiker meinen, mit dieser banalen Erkenntnis selber den Schlüssel zur Wahrheit entdeckt oder in Händen zu haben.

    An der Zirkularität naturwissenschaftlicher Erkenntnis führt kein Weg vorbei. Es gibt nur einen Ausweg. Das ist die Vernetzung aller Erkenntnisse zu einem komplexen, zusammenhängenden, widerspruchsfreien Gebilde oder System. Je mehr Erkenntnisse eingefügt werden, desto zuverlässiger ist das Gesamtgebilde.

    Jede Information, nicht nur naturwissenschaftlicher Art, ruht auf einem System von Präformation, das ist das Zahlensystem, das ist die Sprache, das sind die kulturellen Vorbedingungen und Vorleistungen. Diese Präformation ist nicht begründbar oder hinterfragbar, denn sie beruht auf den spezifisch menschlichen, biologischen und evolutionär entstandenen Vermögen der Wahrnehmung, des Denkens, des Gedächtnisses und des Vorstellungsvermögens.

    Man kann darüber diskutieren, man kann es bemängeln, aber niemand hat eine bessere Alternative. Die Wissenschaft ist ein Prozess der Annäherung an die “Wahrheit”, wobei Wahrheit eben als “empirische Adäquatheit”, als Kompatibilität mit wiederholten empirischen Bestätigungen, z.B. mit dem Eintreffen von Vorhersagen, zu verstehen ist. Aus Theorien logisch abgeleitete Aussagen, die durch Beobachtung widerlegt werden, widerlegen ebenfalls die Prämissen der Aussagen, also die ganze Theorie oder Teile davon (Duhem-Quine-These).

  89. AR,
    Die Idee der Präformation sollte sich jeder Physiker zu eigen machen. Das Haupthindernis für Erkenntnis ist die Sprache selbst. Wenn der Begriff fehlt, stockt das Denken.
    Für alle , die gern überinterpretieren, sei die Kritik der reinen Vernunft von Emmanuel Kant empfohlen.

  90. Hallo Herr Päs,

    „Die Besonderheit, die die Physik auszeichnet, nämlich experimentell testbare Modelle der Realität zu entwerfen, die quantitative, mathematische Vorhersagen erlauben, ist aber zweifellos durch nichts anderes zu ersetzen.“

    Jui, Jui. Wenn eine physikalische Theorie oder ein Modell Vorhersagen macht, die nicht eintreffen, wird niemand die Theorie oder das Modell als gültig ansehen. Die Besonderheit, die die Physik auszeichnet, scheint mir daher gar nicht besonders zu sein, sondern eine Banalität.

    Oder was soll daran besonders sein ?

    Die Frage, welche Rolle die empirische Verifikation in der Physik spielt, kann man im Rahmen der analytischen Philosophie, speziell im Rahmen der Erkenntnistheorie diskutieren. Führt man diese Diskussion, würden wir wahrscheinlich schnell darüber einig werden, daß die Strategie der empirischen Verifikation sehr mächtig ist, aber auf weiten Strecken des Weges zur physikalischen Erkenntnis auch ohnmächtig. Denn es gibt wichtige und grundlegende physikalische Theorien, die zutreffende Vorhersagen machen, und aus denen man sogar einen Mechanismus (z.B. ein Modell) herauslesen kann, der uns erklärt, wie das vorhergesagte Ereignis zustandekommt, oder zustande kommen muß. Aber dann kommt ein anderer Physiker, der liest aus der Theorie für das Zustandekommen des gleichen Ereignisses einen ganz anderen Mechanismus heraus – und dann kann man eben mit der Empirie nicht entscheiden, welcher Mechanismus der richtige ist. In allen Aussagen von Theorien, in denen man die Ontologie nicht genau kennt, die da am Werke ist, ist das eben ein Dilemma dieser besonderen empirischen Methode. Man benötigt zusätzliche Annahmen, die auch von außerhalb der Physik kommen können.

    Auch in dem hiesigen Beitrag über die Dreidimensionalität dürfte es wohl auf dem Wege empirischer Verifikation nicht zu entscheiden sein, ob das von Ihnen favorisierte Modell der Multikneuels oder andere konkurrierende Modelle der Wahrheit am nächsten kommen. Oder genauer: Alle Modelle sagen das Richtige vorher, was in diesem Fall trivial ist, aber wenn Sie an die Natur die Frage stellen, welches ist der richtige Mechanismus, werden Sie die Antwort bekommen: alle Mechanismen sind richtig, oder keiner. Die empirischen Methoden setzen eben voraus, daß die richtigen Fragen auf richtige Weise an die Natur gestellt werden, sonst liefert die Super-Strategie der empirischen Verifikation eben auch Mumpitz. Und was richtige Fragen sind, kann die Physik allein gar nicht entscheiden.

    Also lassen Sie doch die Frage zu, ob diese besondere Methode die Physik wirklich auszeichnet.

    Die Physik ist doch eine großartige Wissenschaft ! Sie ist dies aber nur dadurch, daß diese Großartigkeit durch alltägliche Praxis der Physiker immer und überall aufs Neue begründet wird. Einen Beitrag zu dieser Großartigkeit haben Sie getan, indem Sie einen Blog in diesem Forum aufgemacht haben, in dem über die Physik geredet werden kann. Das ist ein Super-Schritt – ohne Wenn und Aber !

    Aber die Verkündung von Unbedingtheiten – sozusagen Ex-Kathedra – gehören da nicht rein. Sie tun der Physik damit keinen Gefallen. Statt Abgrenzung zu anderen Methoden wünschte ich mir bei Ihren Beiträgen mehr Distanz, Nachdenklichkeit, kritische Reflexion über die Physik und Ihre Methoden. Das ist eigentlich kein Problem, das zu machen. Sie werden auch präzisere Kommentare bekommen. Ich hoffe Sie fassen das nicht falsch auf – Klappern für sein Metier ist gut, aber Inhalte kontrovers besprechen – das ist doch viel besser. Ich wünsche Ihnen jedenfalls viel Erfolg mit Ihren Beiträgen und will gerne meinen Beitrag dazu leisten.

    Frohe Weihnachtsgrüße
    Fossilium

    • (Eine) Meine(r) Oma (s) hat nach dem Festgottesdienst immer gesagt: “Heut hat er wieder schön gepredigt.” Aber trotzdem : Ich kann Fossiliums Apell ziemlich zustimmen.
      Auch ohne festtägliche Euphorie.

  91. Hallo Fossilium,

    Wenn eine physikalische Theorie oder ein Modell Vorhersagen macht, die nicht eintreffen, wird niemand die Theorie oder das Modell als gültig ansehen. Die Besonderheit, die die Physik auszeichnet, scheint mir daher gar nicht besonders zu sein, sondern eine Banalität.

    Die Besonderheit ist gerade, dass überprüfbare Vorhersagen gemacht werden. Das mag Ihnen vielleicht banal erscheinen, aber ein kurzer Blick in die Kommentarspalte meines Blogs (siehe Augustinus, Bibel oder Buddhismus) zeigt, dass das banale manchmal das Besondere ist. Übrigens zeigt das auch ein kurzer Blick in die Geschichte der Menschheit, die erst im 17. Jahrhundert und nur in Europa geschafft hat, die moderne Naturwissenschaft zu entwickeln.

    Die Frage, welche Rolle die empirische Verifikation in der Physik spielt, kann man im Rahmen der analytischen Philosophie, speziell im Rahmen der Erkenntnistheorie diskutieren.

    Tatsächlich ist spätestens seit Popper bekannt, dass wissenschaftliche Theorien nicht verifizierbar sind. Das ist keine Neuigkeit.

    Aber dann kommt ein anderer Physiker, der liest aus der Theorie für das Zustandekommen des gleichen Ereignisses einen ganz anderen Mechanismus heraus – und dann kann man eben mit der Empirie nicht entscheiden, welcher Mechanismus der richtige ist.

    Deswegen sprechen Popper (und Physiker) auch von der Falsifizierbarkeit von Theorien. Und die zeichnet physikalische Theorien wirklich aus.

    Auch in dem hiesigen Beitrag über die Dreidimensionalität…: Alle Modelle sagen das Richtige vorher, was in diesem Fall trivial ist, aber wenn Sie an die Natur die Frage stellen, welches ist der richtige Mechanismus, werden Sie die Antwort bekommen: alle Mechanismen sind richtig, oder keiner.

    Da haben Sie nicht richtig gelesen: Das Modell erlaubt Vorherhersagen für zukünftige Messungen, die es von anderen Modellen unterscheidbar machen. Natürlich ist nicht gesagt, dass alle denkbaren physikalischen Theorien mit verfügbaren Mitteln unterscheidbar sind, und da kann man in der Tat eine Diskussion führen, welche Theorie (oder ob überhaupt eine) vorzuziehen ist, aber das leugnet niemand.

    Aber die Verkündung von Unbedingtheiten – sozusagen Ex-Kathedra – gehören da nicht rein. Sie tun der Physik damit keinen Gefallen.

    Auf welche “Unbedingtheiten”, die ich angeblich verkündige, beziehen Sie sich denn? Ich will Ihnen nicht zu nahe treten, aber wissen Sie, dass es eine beliebte Strategie von Populisten ist, dem politischen Gegner Aussagen zu unterstellen, gegen die man dann lautstark protestieren kann? Natürlich unterliegen wissenschaftliche Theorien dem Wandel der Zeit, natürlich kann man hinterfragen, ob und wie getreu sie eine hinter den Erscheinungen vermutete Realität wiederspiegeln und in wie weit sie bloß Konstrukte sind. Natürlich kann man auch darüber spekulieren, ob eine wissenschaftliche Theorie durch eine bessere ersetzt werden kann. Aber dazu muss man die Theorie erst einmal verstanden haben. Und so lange man im Kontext einer Theorie diskutiert, gibt es in der Physik tatsächlich auch ganz einfach “richtig” oder “falsch”, ob es Ihnen nun passt oder nicht.

    Statt Abgrenzung zu anderen Methoden wünschte ich mir bei Ihren Beiträgen mehr Distanz, Nachdenklichkeit, kritische Reflexion über die Physik und Ihre Methoden.

    Was sind denn die “anderen Methoden”? Ganz ehrlich, ich kann in den Vorstellungen des Augustinus keine ernstzunehmende Alternative zur Relativitätstheorie sehen, wenn es darum geht, ein Modell von Raum und Zeit zu konstruieren. Ernest Rutherford sagte einmal “Wissenschaft ist entweder Physik oder Briefmarken sammeln”. Die meisten Kommentare hier sind allerdings nicht einmal das zweite.

  92. @ H.Päs und:
    “..deswegen sprechen Popper (und Physiker) auch von der Falsifizierbarkeit von Theorien. Und die zeichnet physikalische Theorien wirklich aus…” (Ende des Zitats)

    Sie stellen die Sache eventuell etwas zu schlicht bzw. einseitig dar.
    Selbst ich kann mich an Blogdiskussionen mit/unter Physikern/Naturwissenschaftlern erinnern, die bei Hinweisen auf Popper bzw. das Falsifikationsprinzip genervt abgewunken haben. Entweder weils sie als Experimentatoren und (eventuell) etwas “naive Empiristen” das für lästige Metaphysik hielten, oder weil sie einer “anything goes”- Methodik mehr Erkenntnispotenz zutrauten.
    Es gibt ja auch einen immer noch aktuellen Streit zwischen “Naturalisten” und sowas Ähnlichem wie ” kritischen Realisten” usw.. Meiner Ansicht nach gibt es da zur Zeit (noch?) keine eindeutigen Antworten diesbezüglich. Vielleicht widersprechen sich die Positionen auch überhaupt nicht so dramatisch, wie oft dargestellt und eine sehr weitgehende Methodenanarchie (aber immer im Verbund mit eingebauter theoretischer Falsifikationsoption) ist der beste Weg.
    Wenn ich mich richtig erinnere, hat das Popper in den späteren Jahren und nach den Diskussionen mit Feyerabend , Kuhn usw. auch nicht mehr viel anders gesehen.

    • Ich stimme insoweit zu, dass die Poppersche Vorstellung einer Abfolge von Theorien, die die Wissenschaftler versuchen zu falsifizieren, und die sich so immer mehr der Realität annähern, mit Sicherheit zu stark vereinfacht ist und nicht den wirklichen Ablauf der Wissenschaftsgeschichte abbildet. So hat z.B. Kuhn sicherlich auf wichtige Aspekte hingewiesen, die er durch die Begriffe “Paradigma”, “Normalwissenschaft” und “wissenschaftliche Revolution” beschreibt. Ich persönlich würde auch auch keinesfalls auf einem naiven wissenschaftlichen Realismus bestehen, ich finde Kuhns Modell in weiten Teilen sehr überzeugend, denke allerdings auch, dass wissenschaftliche Theorien zwar nicht unbedingt ein getreues Abbild einer zugrundeliegenden Realität geben, aber nichtsdestotrotz doch mehr sind als nur nützliche Instrumente. Aber das ist meine persönliche Einschätzung und darüber kann man sicherlich streiten.

      Mir ging es hier allerdings darum, dass man seit Popper nicht mehr davon spricht, naturwissenschaftliche Theorien zu verifizieren. Man könnte, sagen die Verifizierbarkeit wurde falsifiziert. Wie genau Wissenschaft nun tatsächlich funktioniert ist tatsächlich eine Frage, die nach wie vor umstritten ist. Dass sie funktioniert zeigt sich allerdings an den technischen Anwendungen.

      • @ H. Päs
        O.K. Danke für die Replik. Bezüglich des letzten Satze könnte man natürlich weiter diskutieren. Aber seis drum. Bin ich halt auch mal optimistisch. (Nicht nur) Weils weihnachtet. (-:

  93. Hallo Herr Päs,

    ich bin doch kein Anhänger von Augustinus oder Populist, oder so was, mit denen dürfen Sie mich nicht in einen Topf schmeißen. Ich bin auch in keiner Weise wissenschaftskritisch, sondern habe eine tiefe Liebe zur Wissenschaft, insbesondere zur Naturwissenschaft. Aber wer ausschließlich Naturwissenschaft intensiv betreibt, wird schnell zu festgefahrenen und schablonenhaften Weltbildern verleitet. Das passiert – auch in der Physik, und ich spreche hier aus Erfahrung.

    Ich bin z.B. dagegen, die Physik mit leuchtenden Augen über den Klee zu loben, sozusagen ihr besondere Fähigkeiten zuzusprechen, die sie vor anderen Wissenschaften auszeichnet, so als ob man vermitteln wollte, die Physiker hätten im Rahmen rationaler Erkenntnisgewinnung über die äußere Welt eine herausragende Stellung, die es rechtfertigt, auf andere herabzusehen, die auch an diesem kollektiven Erkenntnisprozeß teilhaben. Lesen Sie mal die dahingeworfenen Kommentare von Herrn Senf und Herrn Mistelberger. Welche Vorurteile und Mißachtung anderer kommt da zum Vorschein.

    In der Sache meine ich, daß auch im Alltag ständig Ereignisse vorhergesagt werden, und diese Vorhersagen sind doch nichts wert, wenn sie nicht zutreffen. In diesem Sinne ist es nichts Besonderes, Vorhersagen zu testen, die bloße Behauptung, dies würde die Physik vor allem machen, begründet keine Besonderheit. Worauf es ankommt ist doch, daß die Physik Regelhaftigkeiten formalisiert, die eine systematische (!) Vorhersage unter allen möglichen Bedingungen erlauben – das Systematische an den Vorhersagen ist das Besondere, genau darin ist die Physik einzigartig und besonders,

    Aber die eigentliche Besonderheit der Physik ist auch das nicht – sondern wenn man sie von allen anderen Wissenschaften abgrenzen will, dann muß man auf die Tatsache hinweisen, daß die Physik am konsequentesten von allen Naturwissenschaften reduktionistisch arbeitet, also eine reduktionistische Strategie zum Erkenntnisgewinn verfolgt. Was das bedeutet, ist höchst wichtig, hat große praktische Folgen, insbesondere für die Begriffsbildung, wird in den Wissenschaftsgemeinden aber viel weniger diskutiert (wenn überhaupt) als Poppers Falsifikationsargument, das mich so gut wie gar nicht interessiert.

    Die Besonderheiten der Physik, von der Sie sprechen, sind vorhanden, aber anders und differenziert zu reklamieren. Wenn Sie so lapidar dahinsagen, nur die Physik als Einzige oder Erste würde prüfbare Vorhersagen machen, dann kommt nicht zum Vorschein, das Ihre Vorhersagen eine besondere Qualität haben, und sie genau das auszeichnet (und einschränkt), und nicht etwa der Anspruch, für Falsifikationen offen zu sein, oder testbare Ereignisse vorherzusagen.

    Wie Sie sehen, ich möchte die Physik stark machen, das können Sie mir glauben, auf die richtige Weise. Sie ist ja nicht von sich aus stark, sondern wird stark gemacht durch die, die Physik betreiben. Wie schrieben Sie in irgend einem Kommentar ? Der Physiker müsse offen sein für Kritik, und müsse seine Erkenntnisse diskutieren, aber Sie meinen wohl, das könnten Sie nur mit Kollegen, die Ihre Physik verstanden haben ? Dann erklären Sie doch, um was es physikalisch geht. Das ist doch ganz einfach. Oder funktioniert das nicht ? Und liegt an Ihrem Gesprächspartner ? Das müßten Sie mal genauer erklären – ich komm bestimmt noch mal drauf zurück.

    Bis dahin beste Grüße

    Fossilium

    • Hallo Fossilium,

      ich bin doch kein Anhänger von Augustinus…

      Nun denn, vielleicht tue ich Ihnen unrecht….

      Da sind Sie aber meines Erachtens selbst nicht ganz unschuldig dran, wenn ich mal Ihre Kommentare so durchforste: Gleich im ersten Kommentar wird mir Unehrlichkeit vorgeworfen, es wird kritisiert, dieser Blogeintrag beruhe “auf formalen, statt auf rein physikalischen Überlegungen” (das habe ich zugegebenermaßen nicht verstanden, aber es klingt mir nach einem Angriff auf die Mathematisierung der Physik), sie fragen “was ist eine Dimension”, nun – das hatte ich versucht, am Anfang des Eintrags anschaulich zu machen, es ist auch relativ allgemeinverständlich, und den Hinweis von Herrn Senf auf Freiheitsgrade haben Sie auch nicht recherchiert, insofern klingt das für mich nach philosophischer Fundamentalkritik ala “wir können sowieso nichts wissen, also ist auch jede Behauptung gleich richtig”, was Sie in meinen Augen im Folgenden mit der Forderung unterstrichen haben, doch einmal zu erklären, warum die hier vorgestellten Modelle den Ideen des Augustinus überlegen sind. Schließlich wird pauschal behauptet, Physiker würden nicht mit Philosophen sprechen und ich würde Unbedingtheiten ex Kathedra verbreiten, was ich meines Erachtens nirgends getan habe.

      Verstehen Sie mich nicht falsch: Man kann durchaus berechtigterweise das erkenntnistheoretische Fundament der Physik hinterfragen. Man kann auch Verständnisfragen stellen und Kritik üben.

      Aber es nervt, wenn einem Dinge unterstellt werden wie Gewissheiten zu predigen, obwohl man ständig auf offene Fragen innerhalb der Physik hinweist, die Philosophie ignorieren würde, obwohl man schon mehrmals auf Erkenntnis- und Wissenschaftstheorie eingegangen ist. Es nervt, wenn schulmeisterlich Kritik an physikalischen Theorien geübt wird, die auf dem eigenen Unverständnis beruht. Es nervt auch, wenn Verschwörungstheorien ala “es gibt Wissenschaftler, die Daten manipulieren -> alle Wissenschaftler lügen -> der Wissenschaft ist nicht zu trauen” oder “ich verstehe Mathematik/Relativitätstheorie usw… nicht -> Mathematik/Relativitätstheorie usw… muss falsch sein -> Wissenschaft ist ein einziger großer Betrug” produziert werden. Ich bin tatsächlich überzeugt davon, das die Wissenschaft und Wissenschaftler (wie eigentlich alle Menschen) einen Vertrauensvorschuss verdient haben.

      Wie gesagt, ich will Ihnen hier nichts unterstellen, aber ich glaube ich tue Ihnen kein Unrecht, wenn ich sage, dass einige Ihrer Kommentare in dieser Richtung missverstanden werden können.

      Übrigens bin ich tatsächlich der Meinung, Physik habe “im Rahmen rationaler Erkenntnisgewinnung über die äußere Welt eine herausragende Stellung”, das sieht man m.E. allein schon daran, dass viele Physiker ganz entscheidend zur Entwicklung anderer Wissenschaften wie z.B. Biologie, Chemie, Volkswirtschaft, Informatik usw… beigetragen haben, während das umgekehrt eher selten vorkommt. Dabei geht es mir keineswegs darum, irgendwen oder andere Wissenschaften herabzuwürdigen, sondern Physik als einen besonderen Zugang zur Welt zu feiern, von dem auch andere Wissenschaften profitieren können.

      Und auf jeden Fall muss ich hier auch nochmal eine Lanze für die Herren Senf und Mistelberger brechen, denen tun Sie nun wirklich unrecht: Gerade Herr Senf klärt hier mit einer Engelsgeduld Fehler und Missverständnisse auf, die von einigen Kommentatoren als Fakten verbreitet werden. Damit hilft er jedem, der an einem ernsthaften Austausch interessiert und lernbegierig ist. Wenn einem dann auch ab und zu mal der Geduldsfaden reisst finde ich das nur menschlich.

      Und genau so wünsche ich mir auch meine Kommentatoren: Bezogen auf das Thema des Blogposts, lernwillig, freundlich, bei Unverständnis oder Meinungsverschiedenheit auch die eigene Vorstellungen kritisch hinterfragend und vor Allem die oben erwähnten Lästigkeiten unterlassend.

      So, genug ex Kathedra gepredigt. Ich wünsche Ihnen auf jeden Fall frohe Feiertage und freue mich dass Ihnen der Blog anscheinend Spass macht, das ist eine wichtige Information, die tatsächlich motiviert, weiter zu machen.

      Beste Grüße, Heinrich Päs

      • @ Herr Päs und:
        “….klingt das für mich nach philosophischer Fundamentalkritik ala “wir können sowieso nichts wissen, also ist auch jede Behauptung gleich richtig”,

        “….es gibt Wissenschaftler, die Daten manipulieren -> alle Wissenschaftler lügen -> der Wissenschaft ist nicht zu trauen” oder “ich verstehe Mathematik/Relativitätstheorie usw… nicht -> Mathematik/Relativitätstheorie usw… muss falsch sein -> Wissenschaft ist ein einziger großer Betrug” produziert werden. Ich bin tatsächlich überzeugt davon, das die Wissenschaft und Wissenschaftler (wie eigentlich alle Menschen) einen Vertrauensvorschuss verdient haben.

        Sorry, Herr Päs, aber kann nicht soganz glauben, dass Sie das von Ihnen oben zitierte wirklich so fest glauben, wie sie behaupten.Denn:
        1. Wer von den Diskutanten in Ihren blogs hier behauptet denn wirklich ernsthaft, dass die Wisennschaft generell reiner Betrug sei. Und zwar deswegen, weil er die Mathematik nicht versteht?
        2. Wer hier behauptet denn wirklich, dass ausnahmlslos alle Wissenschaftler lügen würden?
        3. Und können Sie wirklich allen Ernstes behaupten, alle Fundamentalkritiker wären eine Art erkenntnistheoretisch- nihilistischer Wissenschafts-Relativisten und würden behaupten,dass alles, was man sage könne , gleich richtig sei, weil man grundsätzlich nichts wissen könne?

        Das alles klingt eher nach der uralten Rhetorikstrategie, seinen Kritikern Absurdes zu unterstellen , um dann “haarscharf beweisen” zu können dass diese wissenschaftlich (oder generell ) nicht ernst zunehmen seien.
        Schämen Sie sich jetzt (wenigstens ein ganz kleines bisschen. Ich hab mich ja auch wegen des Vorschlags zu einem “Einstein -Erotik- Blog” bei jemandem entschuldigt.
        Und bin in mich gegangen.
        Ach ja fast hätte ich es vergessen: Schon vor recht langer Zeit habe ich (zum ersten mal) bei einem anerkannten Erkenntnistheoretiker und Wissenschaftsphilosophen gelesen, man solle NIEMALS Autoritäten einen vertrauensvorschuss geben. Schon gar nicht, wenn sie sich einer ausgesucht blumig- abtrakten “Insidersprache” bedienen, um ihre Communityzugehörigkeit sicherzustellen. (Ich meine damit ausdrücklich nicht Sie)

  94. Was sind denn die “anderen Methoden”? Ganz ehrlich, ich kann in den Vorstellungen des Augustinus keine ernstzunehmende Alternative zur Relativitätstheorie sehen.

    Sehen Sie, Herr Päs, Sie lesen auch nicht genau,
    Ich spreche von Methoden, nicht von Theorien.
    Wer will denn eine andere Theorie – das wäre doch Schwachsinn.
    Über die Methoden habe ich oben schon mal was gesagt, aber am Thema nur gekratzt, Aber Sie können sehen, man kann darüber reden.
    Sie müssen es nur wollen.

    Grüße
    Fossilium

  95. Der Furor der Physik hat mich bei der Lektüre von Carlo Rovellos Büchlein wieder gepackt. Und ich denke zugleich an jene Parabel: eine Gruppe Blinder wird an einen Elefanten herangeführt. Sie umgeben das große Tier und erhalten die Aufgabe, es durch Tasten zu beschreiben. Dieses Tier ist nicht allzu groß und sehr dünn, wie eine Schlange, sagt Einer. Dieses Tier muss fliegen können, sagt ein Weiterer, es hat zwei große, dünne Flügel aus Haut. Dieses Tier ist kein Tier, vielmehr ein Baum, meint ein Anderer, säulenförmig und dick. Nein nein, meint schließlich ein Kollege, ein Schlange, mag sein, aber eine große, dicke, mit ungewöhnlichen Säbeln links und rechts.

    Ganz unterschiedliche Geschichten von ein und derselben Sache – und alle treffen zu, wie in Kurosawas ‘Rashomon’.

    Versuchen wir die einzelnen Ansätze zu integrieren, erhalten wir vielleicht das wirkliche Bild (ein Bild auf jeden Fall). Vielleicht aber auch nicht, denn was ist das: ein wirkliches Bild?

  96. @Karl Mistelberger;
    Der Physiker im Labor braucht bei seiner täglichen Arbeit sicher keine Philosophie. Aber die Physik als Wissenschaft braucht die Philosophie dringend. Schon die Logik ist eine Sache sowohl der Mathematik als auch der Philosophie. Die kognitiven Vermögen des Menschen sind keine Sache der Physik, sondern der Biologie und wiederum auch der Philosophie. Der Physiker baut auf seinen kulturellen Grundlagen und auf den Ideen unserer Vorfahren auf, ohne dass es ihm bewusst wird und ohne dass er es thematisiert. Auch das ist Sache der Philosophie. Physik ohne Philosophie ist Scheuklappendenken!

    Es gab viele berühmte Physiker, die sich der Bedeutung der Philosophie bewusst waren, auch schon vor dem 20.Jhdt.

    Immer mehr erkennen wir, dass alle Naturgesetze, die wir aufstellen, alle Theorien, die wir bauen, nichts anderes sind als Bilder, die wir herstellen, um das Naturgeschehen verständlich zu machen und zu begreifen. [..] Nur Gleichnisse können wir uns von dem machen, dessen wir um uns gewahr werden. Bilder, die als Menschenwerk notwendig unvollkommen sind und niemals abgeschlossen werden können.

    Wilhelm Wien, 1905

  97. @Heinrich Päs;
    Eine Falsifizierung kann selber falsch sein. Das sieht man gerade hier in den Foren, wo immer wieder versucht wird, mit Falschbehauptungen die Relativitätstheorie zu widerlegen bzw. zu “falsifizieren”.

    Eine Beobachtung kann einzelne Aussagen widerlegen, muss deswegen aber nicht eine ganze Theorie unbrauchbar machen. Solche Aussagen können dann korrigiert werden, was alltäglich in der Physik geschieht, z.B. mit verbesserten Instrumenten.

    Das gilt für jede Naturwissenschaft oder empirische Wissenschaft gleichermaßen, nicht nur für die Physik.

    Immer gibt uns unser Erkennen, das ja im Urteilen besteht, nichts als Zeichen, niemals das Bezeichnete. Dieses bleibt ewig jenseits. Und wer vom Erkennen fordert, daß es uns das Wirkliche realiter näher bringen sollte, der stellt damit nicht etwa eine zu hohe, sondern eine unsinnige Forderung. Wir sahen ja seit langem ein, [..]: im erkennen können und wollen wir das Erkannte gar nicht gegenwärtig haben, nicht eins mit ihm werden, nicht es unmittelbar schauen, sondern nur Zeichen zuordnen und ordnen. Daß die Erkenntnis dies leistet und nichts anderes, ist nicht ihre Schwäche, sondern ihr Wesen.

    Moritz Schlick (1882-1936), Allgemeine Erkenntnislehre

    • Lieber Herr Reutlinger,
      ich verneige mich vor Ihnen.
      Da haben Sie ja jetzt mal was Anständiges geschrieben, dem ich zustimmen kann.
      Werde in Zukunft Ihre Kommentare wieder besser beachten.
      Frohe Weihnachten!
      Fossilium

  98. @anton reutlinger und:
    “…Eine Falsifizierung kann selber falsch sein”….Eine Beobachtung kann einzelne Aussagen widerlegen, muss deswegen aber nicht eine ganze Theorie unbrauchbar machen….” (Ende der Zitate)
    Ich glaube (!), dass Sie Popper hier nicht ganz korrekt “interpretieren”. Die Falsifikation in Bezug auf (große bzw. umfassende) Theorien war wohl umfassender gedacht. Und zwar insofern, als jede Theorie immer auch auf einen generelleren “Lackmustest” hin konstruiert werden solle. Sodass bei einem negativen Ausgang dieses Tests das ganze Gebäude zusammenstürzen können solle.
    Ich denke, es ging Popper gerade nicht darum, durch immer neue ad hoc- Zusatzannahmen Schwachstellen notdürfig abzudichten, um die Theorie auf “Teufel komm raus” am Leben zu erhalten.
    Es ist natürlich klar, das Zusammenbrüche eventuell existenzberohende Folgen für die
    “Hausbewohner” haben können. Und man deswegen versucht ist, solches zunächst mal so gut es eben geht irgendwie zu vermeiden. (-:

    • Lieber L.Schaber,
      Sie haben einen scharfen analytischen Verstand.
      Sie erkennen alle möglichen Schwachstellen in einer Argumentation.
      Sie schreiben viele Kommentare.
      Aber es ist furchtbar kompliziert, diese zu lesen.
      Man weiß nie, auf was Sie eigentlich raus wollen.
      Ich möchte es aber wissen. Weil ich glaube, daß Sie etwas zu sagen haben.
      Können Sie ihre Sachen nicht so schreiben, daß nur das Wesentliche zum Vorschein kommt ? Das worauf es Ihnen ankommt ? Einen Satz, der alles sagt ?
      Wenn Sie das nicht schaffen, dann stimmt mit Ihnen was nicht.
      Also probieren Sie das bitte, mein Kopf brummt.
      Grüße
      Fossilium
      Fossilium

      • @ Fossilium
        Danke für die Blumen. Das mit dem Scharfen analytischen Verstand sehe ich selbst etwas relativer. (-: Hier gehts ja eigentlich nur um ein paar Grundkenntnisse in Wissenschafts- bzw. Erkenntnistheorie.
        Bei meiner Bemerkung zu Anton Reutlinger z. B. geht es um die Wissenschaftstheorie von K.R.Popper. (Kritischer Hationalismus: Logik der Forschung und Folgewerke)
        Das hat früher sogar ein Dorfschullehrer wie ich im Studium gelernt. Natürlich nur bei diesbezüglichem Interesse.
        Fall jemand wirklich noch nichts davon gehört haben sollte:
        Unterhaltsam geschriebene Zusammenfassungen gibt/gab Popper selbst z. B. in:
        K.R.Popper:”Alles Lebenist Problemlösen” Piper, 1994

        Eine Sammlung ausgewählter Orginaltexte z. B. auch in:
        “Karl Popper Lesebuch” MohrSiebeck UTB 1997

        oder eine kurze Fremdbeschreibung in:
        “Köpfe des 20. jahrhunderts: Karl R. Popper” Morgenbuch Verlag 1994

        Man sollte Popper aber immer auch zusammen mit anderen Erkenntnistheoretikern
        wie z. B. Thomas Kuhn und Paul Feyerabend rezipieren.

  99. @L.Schabers Beiträge glaube ich hier so ziemlich am besten zu verstehen. Sie sind sehr klar, basieren auch auf dem analytischen Verstand und einem beachtlichen Wissen, d’accord. Darüber hinaus aber zeichnen sie sich an ihren Rändern durch eine gewisse Lockerkeit und Vielstrahligkeit in den Bezügen was, was ich als angenehm relaxt empfinde. Mir scheint, das gibt ihm der vorweihnachtliche Geist so ein, oder der Daimon der Wintersonnwende, keine Ahnung.

  100. @ anonymus und fossilium
    Weder Sonnenwende noch Strahlkraft. Auch keine Dämonen. Noch nicht mal die geistigen (des Glühweins). Wegen des angebrochenen Pensionsalters bin ich da jetzt vorsichtig ängstlicher. Bei dem, was zur Zeit sonst noch so in der Welt los ist ( vor allem in der politischen) , frag ich mich bei jedem Tastenschlag, weshalb ich eigentlich meine Zeit mit (für mich manchmal) “alten Kamellen” verbringe. Aber eigentlich war das in der Politik ja immer schon so.
    Dann sehe ich aber auch wieder,dass viele gegenwärtige “Probleme” direkt wieder direkt zu den “Grundlagen” zurückführen und ohne Kenntnis derselben nur schwer zu verstehen bzw. zu lösen sind. Man muss sich doch nur die Blogthemen hier anschauen.Und nicht umsonst taucht regelmäßig immer wieder K.R. Poppers “Offene Gesellschaft” auf. Leider auch oft zum Zwecke propagandistischer Verdrehungen. Und keiner merkts, weil heut fast keiner mehr Orginaltexte liest. Denn einem sehr mächtigen Online- lexikon ist auch oft nicht zu trauen, denn die (angeblich nicht existierenden) Herrscher dort, haben leider auch eine (wenn auch eine etwas versteckte) weltanschauliche Agenda. Meine Dämonen raten da immer zur Vorsicht und sagen, ich solle wenigstens immer die Diskussionsseite mitlesen. Ach, die Welt ist ja sowas von kompliziert. Keinem kann man mehr blind vertrauen! Auch mir nicht. Das ganze nennt sich “Skeptizismus” (auch gegen sich selbst) (-:
    Hoffentlich sagen meine Omas da oben im Himmel hoch jetzt: Heut hat er wieder schön gepredigt. Aber ich glaub, manche Opas würden mir zustimmen. Von nix kommt nix. Oder doch irgendwie?

    • Hi Schaber,

      gehen Sie mit mir in eine Bar auf St. Pauli.
      Wir setzen uns ein eine Ecke wo die Musik nicht so laut ist und wir nicht ständig angequatscht werden.
      Dann unterhalten wir uns, wie man zu einer nicht mehr hinterfragbaren Meinung über die wichtigen Sachen im Leben kommt. Das muß man nämlich, sonst versinken wir im Relativismus und Skeptizismus. Wenn wir das geschafft haben, mischen wir und unter Leute und ich sage Ihnen: Sie sehen alles mit anderen Augen.

      Ausgemacht ?

      Grüße Fossilium
      Sorry, persönliches gehört nicht hierher, aber eigentlich outen wir uns in den Posts ja immer irgenwie persönlich – also hier mal Augen zugedrückt.

  101. Wenn man viele weiße Schwäne gesehen hat, dann kann man daraus die Hypothese (Vermutung) schlussfolgern, dass alle Schwäne weiß sind. Das ist die Methode der Induktion zur Bildung von Hypothesen und Theorien. Wenn man viele weitere weiße Schwäne sieht, wird die Theorie bestätigt, aber die Beobachtung hat kaum einen Informationswert. Sieht man dagegen einen einzigen schwarzen Schwan, dann wird die Theorie sofort widerlegt und die Beobachtung hat somit einen hohen Informationswert.

    Popper fordert daher die Aufstellung “kühner Hypothesen”, die falsifizierbar sind und einen hohen Informationswert aufweisen. Das heißt, die Falsifizierbarkeit ergibt sich aus dem hohen Informationswert. Das Gegenstück sind Allgemeinplätze ohne Informationswert, wie sie bei Politikern und besonders bei Ideologen üblich sind. Allgemeinplätze sind kaum falsifizierbar, man kann sie fast immer subjektiv so interpretieren, dass sie eine wahre Behauptung enthalten. Die Falsifizierbarkeit von Theorien schließt die Möglichkeit der Bestätigung durch Beobachtung mit ein. Sie ist jedoch kein Allheilmittel für die Wissenschaft und ist von anderen Wissenschaftsphilosophen modifiziert worden.

    Leider gibt es heute viele selbsternannte “Skeptiker”, die den Skeptizismus missbrauchen und vorschieben für ihre unbegründeten Zweifel und zur Ablehnung wissenschaftlicher Erkenntnisse. Die wahren Gründe dabei sind nichtwissenschaftlichen Ursprungs, wie religiöse, ideologische, politische oder schlicht egoistische Gründe. Das deutlichste Beispiel ist der Klimawandel. Andere Beispiele gibt es in den Foren hier zuhauf.

    • Hallo Herr Reutlinger,

      alles was Sie schreiben ist o.k. , auch was Popper gesagt hat.

      Aber hier geht es nicht um die Aufstellung “kühner Hypothesen” – das können Leute, die im Wissenschaftsbetrieb sind, machen. Wer will auch Theorien aufstellen, wenn die vorhandenen doch perfekte Vorhersagen machen.

      Das Problem liegt ganz woanders. Die Physik hat wirklich ein Problem, und das besteht darin, daß sie so langsam nicht mehr weiß, was ihre grundlegenden Theorien metaphysisch eigentlich aussagen. Das Problem hat sie von Anfang an gehabt, seit Newton, jedoch ist es mit der Quantenmechanik erst zum echten Problem geworden, weil die experimentellen Ergebnisse nicht mehr konsistent erklärt werden können. Poppers erkenntnistheoretische Analysen helfen da keinen Deut – sie helfen null komma garnichts.

      Ich meine die Frage: was sagt eine physikalische Theorie über die Welt aus ? – mit Theorie ist die mathematische Zusammenfassung, die mathematischen Grundgleichungen (!) gemeint.

      Was beschreibt dieser Formalismus ?

      W o v o n r e d e n d i e G l e i c h u n g e n ?
      Das ist hier Mutter aller Fragen.

      Diese Frage ist zu klären.

      Diese Frage wird aber im wissenschaftlichen Diskurs aber nicht gestellt. Die Frage ist einfach kein Thema. Keiner geht sie an. Gottvater Bohr hat von 80 Jahren eine Antwort gegeben. Na gut, was besseres werden wir nicht finden, erst Recht nicht im Gespräch mit Philosophen.

      Lesen Sie mal was da so im naturphilospohischen Hier und Da geschrieben wird. Kein Versuch einer Lösung. Nur Resignation. Shut up and calculate.

      Mit dieser Frage habe ich mich beschäftigt. Man kommt da zu einigen Einsichten. Aber kein Physiker, und ich kenne einige, begibt sich herab, um über diese Frage auf Augenhöhe mit einem Philosophen zu diskutieren – obwohl es eine philosophische Frage ist. Das ist das, was ich kritisiere. Es gibt keine Diskussion. Es ist eine Überheblichkeit der Physiker, die da zum Ausdruck kommt, Resignation, Ignoranz und Abschottung, die da stattfindet. Die Physik ist in dieser Frage nicht offen. Sie traut keinem, und kommt allein nicht zurecht.

      Sie können das Problem ja nicht durch ein Mehr am physikalischer Erkenntnis lösen ! Im Gegenteil – mit mehr Erkenntnis tritt das Problem nur schärfer zu Tage. Die Physik kriegt mehr und mehr das, was sie im Fundamentalen macht, nicht mehr an Interessierte kommuniziert, sie hat keine Ontologie mehr, sie kriegt ihre Vorstellungen nicht mehr verständlich an die Leute rübergebracht. Daher tummeln sich jede Menge Halbwissende mit seltsamen Idden auch hier im Forum. Man versteht die Physik nicht mehr – weil die Physiker selbst die Metaphysik ihrer Theorien nicht mehr verstehen. Das sind harte Worte, aber ich denke so ist es.

      Na gut, jetzt ist Schluß damit.

      Grüße Fossilum

      • @ fossilium:
        Mit Ihrer General-Kritik im letzten Absatz gebe ich ihnen Recht. Aber damit :
        “….Gottvater Bohr hat von 80 Jahren eine Antwort gegeben….” Na gut, was besseres werden wir nicht finden, erst Recht nicht im Gespräch mit Philosophen. (Ende Ihrer Zitate)
        – übertreiben Sie wirklich gewaltig. Menschen , die (besonders an Weihnachten) das “Heil” (von oben) suchen, gefällt Bohrs Metaphysik natürlich. Denn sie eröffnet Freiräume zu fast allem. Bzw. zu allem Möglichen und auch Unmöglichen.
        Aber tatsächlich ist es so (gewesen) ,dass über Bohr bzw. dessen “Antwort” schon etwa 100 Jahre lang gestritten wird. Und eine Menge Alternativen (zu seiner etwas geisterhaften Interpretation) vorgeschlagen wurden. Und zwar sowohl von Philosophen als auch von Naturwissenschaftlern. Sogar von Experimentalphxsikern.
        Und gerade auch zur Zeit wieder. Bohrs Gottstatus wurde von Anfang an erheblich angekratzt. Ich kann mir nicht vorstellen, dass Sie keine Kenntnis von all dem haben.

  102. @ anton reutlinger am 20.12. 17 um 11:39
    1. Absatz eins korrekt. Allerdings spricht Popper Meines Wisssens nicht von “Informationswert” sondern von der “Qualität” einer Theorie etwas über die Realität auszusagen bzw. aussagen zu können.
    2. In Absatz zwei ist die Bemerkung über “Allgemeinplätze” wegen ihrer “Hoffärtigkeit” selbst zu Allgemein- geplatzt. Popper war da wohl nicht so (dogmatisch) streng und hat “vorhypothetische” Inspirationen “zugelassen” bzw sogar für notwendig gehalten.
    3. Zu Absatz 3:
    Ich formuliere mal um:
    Leider gibt es heute viele selbsternannte “Skeptiker” oder sonstige Wissenschaftler, die den Skeptizismus missbrauchen und vorschieben für ihre Versuche ,den gegenwärtigen Mainstream der professionellen Wissenschaftler gegen Kritik zu immunisieren.Und zwar indem sie sich weigern, den Skeptizismus gegen sich selbst anzuwenden oder indem sie nur vorgeben, dies zu tun. Und dadurch eventuell wissenschaftliche Erkenntnisse bewusst verhindern. Die wahren Gründe dabei sind nichtwissenschaftlichen Ursprungs, wie religiöse, ideologische, politische oder schlicht egoistische Gründe. Das deutlichste Beispiel ist der Klimawandel. Andere Beispiele gibt es in den Foren hier zuhauf.

    Nur so könnte eventuell ein richtiger Schuh draus werden.

  103. @L.Schaber;
    Wissenschaftliche Erkenntnisse sind auch Informationen. Zu der Zeit als Popper seine Hauptwerke schrieb, gab es die Informationstheorie noch nicht. Als Informatiker sehe ich eben die Verbindungen zum Begriff der Information. Je prägnanter eine Aussage, desto höher ist generell der Informationswert (oder -gehalt).

    Allgemeinplätze sind Behauptungen, die praktisch keine Information enthalten und so gut wie immer einsichtig und wahr sind, z.B. “viele Asylanten sind kriminell”. Deshalb sind sie nicht falsifizierbar. Eine Aussage nach Poppers Muster wäre “5,6% der schwarzafrikanischen Asylanten sind straffällig geworden”. Diese prägnante Aussage ließe sich durch eine Studie bestätigen, korrigieren oder widerlegen.

    Popper war ein Feind von Dogmen. Was Sie zur Skepsis von Wissenschaftlern hier schreiben, ist mir rätselhaft. Jeder Wissenschaftler darf an seine Theorien glauben und sie gegen Kritik verteidigen. Die Skepsis muss schon vorher bei der Bildung von Hypothesen und Theorien einsetzen, danach sind begründete Zweifel möglich.

    • @ A. Reutlinger
      Ich gebe Ihnenja in fast(!) allem recht .Bezüglich der “Skeptiker” dachte ich eher an die ganzs spezielle Kaste von der GWUP usw. Obwohl ich zumindest mit manchen von denen bezüglich Kirchenkritik übereinstimme. Und obwohl der Streit “Naturalismus” gegen “Realismus” usw. keineswegs trivial oder einfach zu entscheiden ist.

      • Und auch ich gebe Herrn Reutlinger in fast allem recht (wobei ich seinen letzten Absatz oben sehr schön formuliert finde, ich halte eher die Aussage

        Raum und Zeit sind Erfindungen bzw. nichts als Ideen des Menschen. Es gibt sie nicht im Kosmos.

        in ihrer Faktizität für sehr gewagt). Abgesehen davon – wer hätte gedacht, dass wir so einig sein können…

  104. Meine Aussage zu Raum und Zeit ist selbstverständlich meine Überzeugung, für ihre Faktizität kann ich nicht garantieren. Fakt ist, dass man Raum und Zeit nicht selber wahrnehmbar sind, sondern nur über physikalische Objekte bzw. Ereignisse. Mir stellt sich die Frage, ob die Dimensionen eine Eigenschaft des Raumes sind, oder ob die Dimensionen ursächlich den Raum aufspannen. Ich bin der Meinung, dass die Dimensionen ein Merkmal der physikalischen Wechselwirkungen sind und den 3-dimensionalen Raum aufspannen. Aus den besonderen Eigenschaften des Lichtes ergibt sich die 4-dimensionale Raumzeit, die vornehmlich oder sogar ausschließlich einen Rechenoperator darstellt ohne physikalische Realität.

    Unser Denken ist zu sehr unseren Alltagserfahrungen verhaftet. Davon müssen wir uns wohl gründlich lösen, so wie Planck mit den Energiequanten und Einstein mit der Relativität.

    • Nun, der Nachweis von Gravitationswellen und die kosmische Expansion deuten zumindest in die Richtung, dass die Raumzeit selbst dynamisch ist und unabhängig von den in ihr befindlichen Objekten existiert. Aber das Ganze ist eine alte Diskussion, die schon Leibniz und (als Stellvertreter Newtons) Clarke geführt haben und die je nach aktuellem Wissensstand wieder neu bewertet wird. Tatsächlich denke ich aber, dass das Konzept Realität selbst hinterfragt werden muss, denn in der modernen Physik beissen sich zwei fundamentale Eigenschaften der Realität, nämlich dass sie beobachterunabhängig (also objektiv) nachprüfbar sein soll. Konzepte wie die Raumzeit oder noch extremer die quantenmechanische Wellenfunktion sind beobachterunabhängige Abstraktionen, die aber nicht mehr direkt experimentell zugänglich sind, während experimentelle Messungen nicht mehr beobachterunabhängig sind. Da wird es m.E. zu einem gewissen Grad Geschmackssache, was man nun als “Realität” bezeichnet.

      • Nun, Herr Päs
        im Interesse Ihrer eigenen Erkenntnisgewinnung sollten Sie – das kann nur eine Empfehlung von mir sein – nochmals ganz von vorn anfangen, sozusagen einen geistige TÜV durchführen.
        Da ist
        1. das existierende Universum mit dem materiellen Befund
        2. die verbale Beschreibung
        3. die physikalische Verarbeitung und
        4. die mathematische Berechnung.
        Da sollte zwischen 1. bis 4. eine strikte Trennung erfolgen!
        Weiter Folgendes:
        Das Universum nimmt Raum ein. (materieller Befund)
        In der Mathematik hat man, um Berechnungen durchführen zu können, das Koordinatensystem mit 3 Dimensionen entwickelt.
        Dimensionen sind “Werkzeuge” der Mathematik um das Universum welches Raum einnimmt in seinen Ausdehnungen/Bestandteilen je nach Bedarf mathematisch zu beschreiben. (mathematische Berechnung)
        Das sollte man begreifen können, das Verhältnis zwischen Raum einnehmen und mathematischer Berechnung. Oder?
        Also nochmal:
        Wenn etwas Raum einnimmt, kann man es mathematisch mit einem 3-dimensionalen Koordinatensystem beschreiben. Das Universum selbst hat kein Koordinatensystem und damit auch keine Dimensionen hervorgebracht. Es nimmt einfach nur Raum ein.
        Mehr nicht.

        • @Funktionalistiker;
          Mir scheint, Sie drehen sich hier im Kreis. Wenn das Universum Raum einnimmt, bedeutet es nicht automatisch, dass der Raum 3-dimensional sein muss und in 3 Dimensionen beschreibbar ist. Für die relativistische Raumzeit trifft es eben nicht zu. Auf die 4-dimensionale Raumzeit kann zur Beschreibung des Universums nicht verzichtet werden.

          Woher kommen die Dimensionen, wenn nicht vom Universum selbst? Die Dimensionen haben nicht nur mathematische, sondern handfeste physikalische Bedeutung. Es sind die physikalischen Kräfte oder Wechselwirkungen, die den 3-dimensionalen Raum aufspannen.

          Die Frage, warum der Raum 3-dimensional ist, lässt sich m.E. aus menschlicher Perspektive grundsätzlich nicht beantworten, jedenfalls nicht empirisch. Vielleicht gibt es eines Tages ein Genie, das die Frage aus logischen oder naturphilosophischen Überlegungen herleiten kann. Grundsätzlich aber wird es dem Menschen nie möglich sein, das Weltgeschehen vollständig zu verstehen, weil er die Welt nicht von außen betrachten kann.

          • @ Herr Reutlinger und :
            “..Grundsätzlich aber wird es dem Menschen nie möglich sein, das Weltgeschehen vollständig zu verstehen, weil er die Welt nicht von außen betrachten kann…”

            Ich frage halt wieder mal, woher Sie das wissen (können) (wollen)? Haben Sie Zugang zur Zukunft der Menschheit?

  105. > Heinrich Päs, 20. Dezember 2017 @ 17:47
    > Da wird es m.E. zu einem gewissen Grad Geschmackssache, was man nun als “Realität” bezeichnet.

    Der besondere Geschmack kommt mit einem ordentlichen Preisschild:

    Es gibt keine alternativen Fakten

    1. Die Welt besteht aus punktförmigen, eigenschaftslosen Teilchen; die Beziehungen zwischen ihnen machen die ganze Physik aus. Das ist das Weltbild, auf das die moderne Naturwissenschaft sich zubewegt.

    2. Ein physikalisches Gesetz muss nicht nur universell sein, sondern auch dann Aussagen zulassen, wenn wir lediglich begrenzte oder ungenaue Kenntnis vom Zustand des Systems haben.

    3. Die einzige Erkenntnistheorie, die den Erfolg der modernen Wissenschaft nicht wie ein unerklärliches Wunder erscheinen lässt, ist der Realismus.

    http://www.spektrum.de/magazin/wissenschaft-erkenntnis-und-ihre-grenzen/1478201

    • Zu:
      “……1. Die Welt besteht aus punktförmigen, eigenschaftslosen Teilchen;…..”
      Auch hier wurde schon öfter das folgende bonmot zitiert:
      “Aus NICHTS kommt nichts” Oder von mir : Aus nichts folgt nichts. Emergenz ist eine (reine) Hypothese (im freien Fall).
      Kann man durchaus so denken. Nur muss man als “Realist” dann halt auch andere Freifall- Hypothesen als (vorläufige) Forschungsansätze gelten lassen. Kommen solche aber nicht aus dem eigenen Stall, weden sie von der “community” gerne sehr schnell als haltlos- spinnerte Esoterik verunglimpft.

  106. Die Welt besteht aus punktförmigen, eigenschaftslosen Teilchen; die Beziehungen zwischen ihnen machen die ganze Physik aus.

    Ganz so einfach ist es meiner Ansicht nach nicht: Teilchenzahl ist z.B. ein beobachterabhängiges Phänomen, wie man beim
    Unruh-Effekt sieht. Und in der Viele-Welten-Interpretation der Quantenmechanik sind Teilchen emergent: There are no Quantum Jumps, nor are there Particles! – H. D. Zeh.

    Von “alternativen Fakten” würde ich freilich nicht sprechen, sondern von einer Freiheit, welchen Konzepten man den Namen “Realität” geben will. Und es
    wäre ja auch langweilig, wenn wir überall einer Meinung wären…

  107. Egal, ob ich das System Heinz-Dieter Zeh oder Heinrich Päs betrachte: Der Unruh-Effekt spielt keine Rolle. Die Näherung der Atome (die in “Wirklichkeit” gar nicht existieren) ist gut genug für die Beschreibung komplexer Systeme. Da ist noch viel Platz nach oben.

  108. @L.Schaber;
    Nun, die Logik eröffnet einen Zugang zur Zukunft der Menschheit, genauso wie die Naturgesetze, in gewisser Weise selbstverständlich. Der Mensch ist nun mal ein Teil des Universums und besteht aus derselben Materie wie das Universum. Wir können das Universum immer nur aus der Innenperspektive betrachten.

    Als komplexe Systeme können wir nie die Elementarteilchen, aus denen wir selber bestehen, als solche oder als “Ding-an-sich” (I.Kant) wahrnehmen. Auch die wissenschaftlichen Instrumente helfen da nicht weiter, denn auch sie sind komplexe Dinge, die obendrein vom Menschen nach seinen Theorien konstruiert sind. Das heißt, jedes Experiment und jedes Instrument setzt bereits eine wissenschaftliche Theorie voraus!

    Es gibt Dinge, die uns immer verborgen bleiben. Dazu gehören auch die sogenannten Qualia des Bewusstseins; das sind die Inhalte des Bewusstseins als Sinneswahrnehmungen. Es ist doch bemerkenswert, dass unsere Nervensignale unabhängig sind von den Sinnesorganen, von denen sie stammen, aber trotzdem in Farben, Töne, Gerüche oder Empfindungen unterscheiden! Ein naiver Realismus ist schon damit eindeutig widerlegt, abgesehen von den bekannten Sinnestäuschungen, wie sie auch von Magiern eindrucksvoll vorgeführt werden.

    • Fast niemand vertritt heute noch einen “Naiven Realismus”, wie Sie ihn unterstellen. Nicht mehr nach der Kritik durch den “Kritischen Rationalismus” usw…

      • In wissenschaftlichen Kreisen ist der naive Realismus sicher kaum anzutreffen. Aber es gibt ihn und es gibt viele Schattierungen des Realismus, sei es ein ontologischer oder ein epistemologischer Realismus. Genauso gibt es verschiedene Formen des Antirealismus, z.B. den Konstruktivismus. Und es gibt Zwischenformen wie den Pragmatismus, Stichwort konstruktiver Empirismus.

        Man kann es wie Herr Päs als “Geschmackssache” bezeichnen oder man hat gute Gründe für die eigene Einstellung. Allgemein spielt die Kultur, in der man aufgewachsen ist, eine große Rolle. In der Naturwissenschaft geht es nicht um Wahrheit, sondern um empirische Zuverlässigkeit, oder “empirische Adäquatheit”. Die Philosophie untersucht das Verhältnis zwischen Wahrheit und Empirie. Die Physiker tun gut daran, hier mit der Philosophie zu kooperieren.

  109. Hallo Herr Päs,

    da bin ich froh, daß Sie nun Zweifel haben, welchem Verein ich angehöre. Es ist aber sehr einfach: ich bin jemand, der mit Physikern in ein philosophisches Gespräch kommen möchte, nicht weil ich selbst etwas über die philosophischen Aspekte der Physik erfahren will, sondern weil ich glaube, daß ich einen Betrag dazu liefern kann, daß die Aussagen der Physik klarer und verständlicher werden, daß sie dann in der Lage ist, bessere Beschreibungen und Erklärungen der äußeren Welt zu liefern als, sie es derzeit tun.

    So ein Satz mag bei Ihnen Unverständnis auslösen. Was für Beschreibungsprobleme soll die Physik denn da haben ? Und wie kann ein Philosoph Probleme der Physik angehen, wenn er doch von der komplizierten Materie nicht viel versteht. Aber ich bin gar kein Philosoph, nur Hobby Philosoph. Und begrifflich, inhaltlich und mathematisch ist mir die Physik sehr vertraut.

    Trotzdem: Niemand läßt sich dazu herab. Ich verstehe das ja, Vorurteile haben wir alle, ich auch. Und wer will sich auf ein Gespräch auf Augenhöhe einlassen, dessen Notwendigkeit nicht zu sehen ist, mit Gesprächsregeln, die man nicht kennt, mit Themen, die ein anderer bestimmt. Das ist das Problem mit den Philosophen, hier und in den interdiziplinären Arbeitsgemeinschaften.

    Und dann noch mit jemand, der schulmeisterlich und belehrend daherkommt. Nu ja, ich habe weder mit der Schule noch mit der Lehre in meinem Leben irgendwas zu tun gehabt. Ich hab oft versucht, den Sprechstil zu ändern, ich komme trotzdem immer so rüber. Das ist zugegebenermaßen ziemlicher Mist !

    Und ich kann mich natürlich auch von Vorurteilen nicht freisprechen. Vielleicht lese ich zwischen den Zeilen mehr als da steht, und schreibe dann was, was anderen die Lust verdirbt. Vielleicht haben Sie ja zu Ihrer Disziplin doch mehr Distanz, als aus den Beiträgen zum Ausdruck kommt. Vielleicht ist das, was andere Kommentatoren von sich geben, nicht so gemeint, wie es klingt.

    Stummheit, Ablehnung, Fehleinschätzung – ich verstehe das. Ich werbe trotzdem weiter. Was soll ich machen. Ich will ein Paper daraus machen, also muß ich vorher drüber reden. Kritik egal welcher Art macht die Sachen, von denen ich intuitiv überzeugt bin, für mich verständlicher, kommunizierbarer, klärt Positionen, Grenzen, und Begriffe. Ich profitiere, wenn man mir widerspricht ! Mein Vorteil !

    Macht leider keiner wirklich.

    Sie sehen, nicht nur Sie haben Grund zur Beschwerde.

    Auf etwas will ich aber noch eingehen, das ist der Gesprächstil in Ihrem Blog. Dass Sie viele Kommentare nerven, das kann ich verstehen, nerven mich auch. Die Frage ist: wie entwickelt sich eine Gesprächskultur ? Die entwickelt sich nur, wenn sich die Gesprächspartner ernst nehmen. Man kann mal hier und da flapsig, ironisch, heftig oder herablassend werden, klar. Aber nicht immer, es muß auch Antworten geben, die den Kommentatoren das Gefühl geben, sie kriegen eine ernste Antwort und nicht nur Belehrungen.

    Ihre eigenen Kommentare tun das. Meine auch, glauben Sie mir, ich bemühe mich sehr. Aber nicht alle, die fachlich versiert sind, tun das, es tun nur Wenige richtig, sehen Sie mal genau hin. Das können Sie ändern, indem Sie an den entsprechenden Stellen mahnen und einfordern. Wäre jedenfalls eine Idee. Und dann schreiben Sie einfach mal, was Sie mit Ihrem Blog beabsichtigen. Herr Schulz will seinen eigenen Verstand schärfen, er schreibt für sich. Das ist o.k. Für was schreiben Sie?

    Ich will jedenfalls ein konstruktiver Kommentator sein. Mindestens nächstes Jahr !

    Jetzt erst mal ins neue Jahr tanzen, mit kubanischer Salsa furiosa y las chicas locas mas hermosas.

    Guten Rutsch y hasta alla !
    Fossilium

  110. @ fossilium
    Es ärgert und betrübt mich wirklich, weil ich dadurch im Hinblick auf ihre (eigenen) oben erläuterten Ansprüche versage: Trotzdem aber steh ich hier und kann nicht anders und sage:
    Sie haben ein wenig recht .Oder auch ein wenig mehr. Ich muss also ein wenig zustimmen, obwohlich doch eigentlich zwecks Ihres Erkentnisfortschritts gewaltig widersprechen soll. So versage ich eben bezüglich Ihrer Ansprüche. (-: (-:

  111. Hallo L.Schaber,
    Tun Sie nicht.
    Jeder, der nachdenklich ist, ist ein Gesprächspartner.
    Wer zuhört lernt.,
    Grüsse von der Theke !
    Fossilium

  112. Und in der “kausalen dynamischen Triangulation” erscheint die Raumzeit auf sehr kleinen Skalen plötzlich zweidimensional oder fraktal.

    Kausalität, kausal, das sind Begriffe, die mich triggern. Dass ich den Link zur Arbeit von Ambjörn, Jurkiewisz und Loll dennoch erst vor ca. drei Wochen angeklickt habe (weil er mich einfach nicht losließ), liegt an der frustrierenden Erfahrung, bislang nichts Interessantes, nichts Neues darüber gelesen zu haben.

    Umso überraschter war ich, als ich beim Lesen feststellte, dass die CDT enthält, wovon ich seit Jahren ausgehe, es nur anders beschreibe – ich bin ja auch keine Physikerin. Ich habe mir deshalb das paper der drei Wissenschaftler heruntergeladen und mich mühsam durch den Text gekämpft, in der Hoffnung, nicht aus mangelnden Kenntnissen etwas hinein zu interpretieren, was nicht drin steht. Wenn ich es daher richtig verstanden habe, dann besagt die Theorie, dass die Raumzeit aus sogenannten Vier-Simplices besteht, die zum einen nach quantenmechanischen / quantentheoretischen (?) Regeln zusammengesetzt sind (Superposition?) und dabei zum anderen die Kausalität als grundlegendes Prinzip berücksichtigt wurde, indem kausale Regeln dabei als Kleber wirken – d.h., nur Vier-Simplices, die die gleiche zeitlich Richtung haben, können sich zur Raumzeit verbinden, wodurch ein außerordentlich stabiler Raum entsteht. Kausal bzw. gleiche zeitliche Richtung heißt dann ja wohl: die kausale Struktur hat die Richtung Vergangenheit-Gegenwart-Zukunft, in dieser Reihenfolge, nicht anders.
    Soweit, so gut (sofern ich es richtig wiedergegeben habe).
    Das eigentlich Spannende an der Geschichte aber ist dieser Teil:

    erscheint die Raumzeit auf sehr kleinen Skalen plötzlich zweidimensional oder fraktal

    Eine zweidimensionale RaumZEIT? Sehe ich mal von Zahlenwert usw. ab und halte mich an die zweidimensionalen (räumlichen), selbstähnlichen Strukturen … wo ist die Zeit abgeblieben?

    Von dieser Struktur schreiben die Autoren in dem oben verlinkten Spektrums-Beitrag : “Möglicherweise wird das Universum letztlich selbstähnlich und sieht unterhalb einer gewissen Schwelle in jedem Maßstab gleich aus. In diesem Fall besteht die Raumzeit nicht aus Strings oder Raumzeit-Atomen, sondern ist im Kleinsten unendlich langweilig.”

    Ich habe an anderer Stelle gefragt, ob es auch so etwas wie eine zeitliche Überlagerung gibt, aber keine Antwort erhalten. Zugegeben, aus der Frage war nicht zu ersehen, dass ich eben das gemeint habe: Was ist mit der Zeit im Bereich des Allerkleinsten, wenn die Raumzeit auf zwei (räumliche?) Dimensionen schrumpft (was ich damals als ich fragte, noch nicht wusste)?

    Möglichkeit Nr. 1: Es gibt keine Zeit, sie ist abgetrennt, abgekoppelt – wie auch immer – worden. Warum spricht man dann trotzdem von der RaumZEIT? (aus dem paper geht das für mich nicht hervor, entweder weil ich es nicht verstanden habe, oder weil darauf tatsächlich nicht eingegangen wurde).

    Möglichkeit Nr. 2: Es gibt Zeit, aber in Form einer Überlagerung: Vergangenheit als Möglichkeit muss in einer möglichen Zukunft vorkommen, und beide sind (als Möglichkeit) in der Gegenwart enthalten – einer andauernden, unendlich langweiligen Gegenwart.

    Meine Frage: Ist Möglichkeit Nr. 2 physikalisch-mathematisch möglich? Wenn nicht, wie kommen wir ab einer bestimmten Größenordnung zur Zeit?

  113. > Karl Mistelberger, 12. Dezember 2017 @ 07:31
    > Das Verschmelzen zweier ziemlich schwerer Schwarzer Löcher wird durch die Allgemeinen Relativitätstheorie befriedigend beschrieben. Diese basiert auf allgemeinen Überlegungen und korrigiert die Newtonsche Theorie dort, wo auf Grund besonderer Umstände Diskrepanzen zu Beobachtungen bestehen. Die Bedingungen beim Verschmelzen der Schwarzen Löcher unterscheiden sich enorm von denen in der Umlaufbahn des Merkurs um die Sonne und trotzdem funktioniert die Allgemeine Relativitätstheorie befriedigend.

    Zwischen Merkurbahn und Verschmelzung zweier Schwarzer Löcher rangiert die Bewegung von Sternen im Zentrum der Milchstraße rund um ein Schwarzes Loch von 4 Millionen Sonnenmassen.

    Der geschärfte Blick der Astronomen enthüllt was viele vermutet haben: “Beobachtungen mit dem Very Large Telescope der ESO haben zum ersten Mal die von Einsteins Allgemeiner Relativitätstheorie vorhergesagten Auswirkungen auf die Bewegung eines Sterns zeigen können, der das extreme Gravitationsfeld in der Nähe des supermassereichen Schwarzen Lochs in der Mitte der Milchstraße durchwandert. Dieses lang ersehnte Ergebnis ist der Höhepunkt einer 26-jährigen Beobachtungskampagne mit den Teleskopen der ESO in Chile.”

    https://www.eso.org/public/germany/news/eso1825/

Schreibe einen Kommentar


E-Mail-Benachrichtigung bei weiteren Kommentaren.
-- Auch möglich: Abo ohne Kommentar. +